Aap 2009
Aap 2009
Question: 1
You are evaluating a 6-month-old child who has a ventricular septal defect and is scheduled for
cardiac surgery. The childs weight is 6 kg (3rd percentile), length is at the 30th percentile, and
head circumference is at the 50th percentile. His mother states she prepares the formula by
adding 1 scoop of powder to 2 oz of water. She estimates that he drinks 24 oz of formula per
day. You estimate the babys intake is approximately 500 kcal per day of cow milk formula,
which is the recommended dietary allowance (RDA) for his age. According to his mother, he
spits up three times a day and passes two soft stools daily. On physical examination, you hear a
3/6 holosystolic murmur and palpate the liver 1 cm below the right costal margin.
Of the following, the BEST explanation for the childs malnutrition is
page 1
Preferred Response: A
Children who have large ventricular septal defects, such as described for the child in the
vignette, have increased pulmonary blood flow and may have ventricular hypertrophy and heart
failure. Because their hearts have to work harder, their caloric needs often are increased above
the recommended dietary allowance (RDA) for healthy children. The absence of significant
vomiting, diarrhea, or rectal bleeding suggests that the patient does not have significant
gastroesophageal reflux, cow milk protein intolerance, or pancreatic insufficiency. Incorrect
preparation of the formula always should be considered, but the mother's reported preparation is
correct.
The RDA is defined as "a nutrient intake level that is ... sufficient to meet the nutrient
requirements of 97% of healthy individuals" in a given group, categorized by sex and age. RDAs
are developed by the Food and Nutrition Board of the Institute of Medicine in collaboration with
Health Canada. A full list of RDAs may be found in many reference textbooks and at the Food
and Nutrition Information Center on the United States Department of Agriculture web site
(http://fnic.nal.usda.gov). RDAs estimate energy requirements of healthy children and do not
adjust for chronic illness states that increase caloric needs, such as cystic fibrosis or congenital
heart disease. Children who have chronic illnesses often require more calories than the RDA to
grow and develop.
The estimated daily caloric requirement of a patient is the aggregate of the patient's basal
metabolic rate and physical activity. Various mathematic equations can be used to estimate the
daily caloric intake of children and adults of different ages. Such equations usually take into
account the individual's age, sex, physical activity level, and either the weight and height or,
preferably, the body surface area. For children who have chronic illnesses, more accurate
estimates of daily caloric requirements can be obtained by using a laboratory-based technique,
such as indirect calorimetry.
References:
McDaniel NL. Ventricular and atrial septal defects. Pediatr Rev. 2001;22:265-270. Available at:
http://pedsinreview.aappublications.org/cgi/content/full/22/8/265
Sonneville K. Nutritional requirements: dietary reference intakes. In: Hendricks KM, Duggan C.
Manual of Pediatric Nutrition. 4th ed. Hamilton, Ontario, Canada: BC Decker; 2005:83-100
page 2
You are admitting a 750-g female infant to the neonatal intensive care unit (NICU) for treatment
of respiratory distress and presumed sepsis. The pregnancy was complicated by
chorioamnionitis and preterm labor. The infants trachea was intubated, a single dose of
exogenous surfactant administered, and both an umbilical venous catheter and umbilical arterial
catheter were placed successfully in the delivery room. In the NICU, the infant is placed on a
radiant warmer. The nurse caring for the infant asks if the infant will need to be transferred to an
isolette incubator.
Of the following, the MOST likely reason for this infant to be relocated into an isolette incubator is
page 3
Preferred Response: E
The extremely low gestational-age newborn (ELGAN) who has a birthweight of less than 1,000 g
is at great risk for transcutaneous evaporative water loss in an open air, nonhumidified
environment such as a radiant warmer. This is most concerning in the first 24 to 72 hours of
postnatal life, diminishing thereafter as the infant's skin becomes cornified. The use of a
contained, convectively heated, and humidified environment can reduce transcutaneous
evaporative water loss best in the first few days of the ELGAN's postnatal life.
Ambient light is not a contributing factor to retinopathy of prematurity, for which the ELGAN
is also at risk. Although excessive thermal warming can occur under the radiant warmer,
bronzing of the skin is related to the use of phototherapy lights in newborns who have
conjugated hyperbilirubinemia. The ELGAN's core temperature can be maintained using a
radiant warmer, but evaporative and convective heat losses remain a concern over lengthy
periods of time, and differences in extremity, head, and core temperatures may affect perfusion
and acid-base status. There is no increased risk for infection on the radiant warmer compared
with the isolette incubator.
References:
Dollberg S, Hoath SB. Temperature regulation in preterm infants: role of the skin-environment
interface. NeoReviews. 2001;2:e282-e291. Available for subscription at:
http://neoreviews.aappublications.org/cgi/content/full/2/12/e282
Korones SB. An encapsulated history of thermoregulation in the neonate. NeoReviews.
2004;5:e78-e85. Available for subscription at:
http://neoreviews.aappublications.org/cgi/content/full/5/3/e78
Sedin G. The thermal environment of the newborn infant. In: Martin RJ, Fanaroff AA, Walsh MC,
eds. Fanaroff and Martin's Neonatal-Perinatal Medicine. 8th ed. Philadelphia, Pa: Mosby
Elsevier; 2006:585-596
page 4
You are seeing a 1-month-old girl for follow-up after a hospitalization for acute gastroenteritis
caused by rotavirus. Her diarrhea had decreased in the hospital while taking oral rehydration
solution, but when her mother resumed her usual cow milk formula, the girl began to have an
increased number of very watery stools. She appears well hydrated, and findings on her
abdominal examination are normal.
Of the following, the MOST appropriate approach to managing this infants diarrhea is to
page 5
Preferred Response: A
The infant described in the vignette most likely has lactase deficiency due to rotavirus infection.
Lactase is an enzyme found in the most superficial villous portion of the intestinal brush border,
which hydrolyzes lactose to glucose and galactose. Lactase deficiency may have several
causes in children and adults. Primary lactase deficiency, the most common type, is a
genetically determined condition that affects children and adults at different ages but is unusual
before 5 years. Symptoms include abdominal distention, bloating, flatulence, or nausea after the
ingestion of lactose, with the amount of lactose needed to cause such symptoms varying from
person to person. The diagnosis is made by breath hydrogen testing, and management consists
of removing some or all lactose from the diet. Congenital lactase deficiency is extremely rare.
Secondary lactase deficiency may develop after an infectious gastroenteritis, such as
rotavirus, giardiasis, or cryptosporidiosis. Other causes include celiac disease and enteropathy
related to immunodeficiency. Secondary lactase deficiency is suggested when a child who has a
recent diarrheal illness experiences worsening diarrhea or bloating after the reintroduction of
lactose into the diet, as described for the girl in the vignette. Most children who have
gastroenteritis do not develop lactase deficiency. For this reason, most infants can tolerate and
should continue taking human milk or standard lactose-containing formula throughout a diarrheal
illness. For very young infants (eg, <3 months old), such as the one described in the vignette, or
those who have significant fluid losses, a lactose-free formula may be attempted until the
diarrhea resolves. Infants who are breastfed should be encouraged to continue breastfeeding,
even if secondary lactase deficiency is suspected.
Giving full-strength formula or human milk is recommended to supply the child with sufficient
calories during the recovery phase of a diarrheal illness; therefore, diluting the formula or
providing only oral rehydration solution is inappropriate. If the child is not vomiting, oral hydration
is optimal, and intravenous hydration is not necessary. There is no need to confirm the
diagnosis of rotavirus infection; doing so would not alter management plans.
References:
Dalby-Payne J, Elliott E. Gastroenteritis in children. BMJ Clinical Evidence. 2007. Available for
subscription at: http://clinicalevidence.bmj.com/ceweb/conditions/chd/0314/0314.jsp
Heyman MB; Committee on Nutrition. Lactose intolerance in infants, children, and adolescents.
Pediatrics. 2006;118:1279-1286. Available at:
http://pediatrics.aappublications.org/cgi/content/full/118/3/1279
King CK, Glass R, Bresee, Duggan C. Managing acute gastroenteritis among children: oral
rehydration, maintenance, and nutritional therapy. MMWR Morbid Mortal Wkly Rep Recomm
Rep. 2003;52(RR-16):1-16. Available at:
http://www.cdc.gov/mmwr/preview/mmwrhtml/rr5216a1.htm
page 6
During a prenatal visit with expectant parents, they report that they are strict vegans. They ask
you to advise them on a healthy diet and any required supplements. The mother plans to
breastfeed the newborn exclusively for the first 6 months.
Of the following, you are MOST likely to tell them that their newborn may require supplemental
A. calcium
B. folate
C. iron
D. vitamin B6
E. vitamin B12
page 7
Preferred Response: E
A vegan diet, by definition, excludes all foods derived from animal products. A lacto-ovovegetarian diet may include milk and eggs. Although a vegan diet may be healthy, there is a risk
for vitamin B12 deficiency because vitamin B12 is only found in foods of animal origin.
Breastfeeding vegan mothers may produce milk that is deficient in this vitamin and require
supplementation that generally is achieved by continuing the consumption of prenatal vitamins
containing vitamin B12.
The recommended supplementation for breastfed vegan infants to prevent vitamin B12
deficiency is 0.4 mcg/day during the first 6 postnatal months and 0.5 mcg/day from 6 months to
1 year of age. Vegan infants who are not breastfed should receive iron-fortified soy infant
formula until 1 year of age to avoid deficiencies in iron. Vegan infants require no other mineral or
vitamin supplementation.
Vegan diets in older children and adolescents may be low in calcium (similar to the typical
American "teenage diet" that contains less than the recommended intake of dairy products), and
the zinc consumption may be relatively low due to the absence of phytate, which renders zinc
more bioavailable. Children who follow vegan diets may have relatively diminished overall energy
intake because such diets commonly are low in fat and high in fiber.
Review of nutrient intake and energy intake in conjunction with growth curves of children
eating vegan diets in both the United States and the United Kingdom demonstrate no significant
health issues. Height and weight measured in vegan populations may be slightly lower than
average but not in the range of failure to thrive or short stature. Adolescents eating vegan diets
are more likely than adolescents eating a typical American diet to meet nutritional goals, including
recommended intake of fruits and vegetables. Vegan adolescents are less likely to be obese
because they consume fewer foods high in fat. However, they remain at risk for vitamin B12
deficiency and should consume at least a daily multivitamin. They are less likely to have anemia
but just as likely to have low calcium intake as their non-vegan peers. Adolescents who follow a
lacto-ovo-vegetarian diet are less likely to have deficiencies in vitamin B12, calcium, and iron.
Folate and vitamin B6 are not likely to be deficient in persons who consume vegan diets
because those nutrients are found in many legumes, fruits, and vegetables that are the
mainstays of the diet.
References:
Kleinman RE. Nutritional aspects of vegetarian diets. In: Pediatric Nutrition Handbook. 5th ed. Elk
Grove Village, Ill: American Academy of Pediatrics; 2003:191-208
Mangels AR, Messina V. Considerations in planning vegan diets: infants. J Am Diet Assoc.
2001:101:670-677. Abstract available at: http://www.ncbi.nlm.nih.gov/pubmed/11424546
Messina V, Mangels AR. Considerations in planning vegan diets: children. J Am Diet Assoc.
2001:101:661-669. Abstract available at: http://www.ncbi.nlm.nih.gov/pubmed/11424545
Moilanen BC. Vegan diets in infants, children and adolescents. Pediatr Rev. 2004:25:174-176.
Available at: http://pedsinreview.aappublications.org/cgi/content/full/25/5/174
Perry CL, McGuire MT, Neumark-Sztainer D, Story M. Adolescent vegetarians: how well do
their dietary patterns meet the Healthy People 2010 objectives? Arch Pediatr Adolesc Med.
2002; 156:431-437. Available at: http://archpedi.ama-assn.org/cgi/content/full/156/5/431
page 8
You are treating a child who has suffered a splenic injury and is being transfused with large
volumes of packed red blood cells for severe anemia. He weighs 10 kg and has received 4 units
thus far.
Of the following, the finding on electrocardiography that is MOST likely to represent a serious
complication of his therapy is
A. atrial flutter
B. delta waves
C. prominent U waves
D. supraventricular tachycardia
E. tall-peaked T waves
page 9
Preferred Response: E
Administration of fluids and blood products can be essential for resuscitation of the trauma
patient but may lead to potentially dangerous electrolyte imbalances. Recognition of these
abnormalities, which may be subtle, can be important and even lifesaving. When large volumes
of blood are transfused, as reported for the child in the vignette, hyperkalemia may occur, which
is believed to be the result of extravasation of potassium from the red blood cells that have been
irradiated and stored over time. Hyperkalemia can be associated with paresthesias, weakness,
and tingling, although cardiac toxicity typically precedes such symptoms. Severe cardiac
rhythym changes may begin abruptly. The classic electrocardiographic sign of hyperkalemia is
tall, peaked T waves (Item C5A), particularly as the serum potassium concentration approaches
or exceeds 5.0 to 6.0 mEq/L (5.0 to 6.0 mmol/L). As hyperkalemia progresses, other changes in
the ECG, such as widening of the QRS complex, may be noted. The rhythm changes that occur
as a result of hyperkalemia, including ventricular arrhythmias, may develop abruptly, leading to
sudden changes in the patient's clinical condition.
Another potential electrolyte complication of fluid and blood administration is hypocalcemia,
which results from the citrate-containing anticoagulants that bind free calcium. Hypocalcemia
may manifest on ECG as a prolonged QT interval, which tends to widen as the ionized calcium
concentrations decrease.
Atrial flutter, a primary disease of the atrial tissue, would not be expected in the patient
described in the vignette. The delta wave refers to the ECG finding of pre-excitation, seen in
conditions such as the Wolff-Parkinson-White syndrome that have an associated "bypass" tract
allowing for excitement of the His-Purkinje system without passage through the atrioventricular
node (Item C5B).
The U wave may be seen in hypokalemia (Item C5C), particularly as concentrations
decrease below 2.7 mEq/L (2.7 mmol/L), or hypercalcemia (calcium concentrations exceeding
12.0 mg/dL [3.0 mmol/L]), which would not be expected in the patient described in the vignette.
Hypercalcemia also may lead to a diminished QT interval, and with more severe hypercalcemia,
second- or third-degree heart block (Item C5D) may develop. Supraventricular tachycardia
(Item C5E) would not be expected to occur as a result of a large-volume transfusion process.
References:
Galel SA, Naiman JL. Use of blood and blood products. In: Rudolph CD, Rudolph AM, eds.
Rudolph's Pediatrics. 21st ed. New York, NY: McGraw-Hill Medical Publishing Division;
2003:1576-1581
Vetter V. Arrhythmias. In: Moller JH, Hoffman JIE, eds. Pediatric Cardiovascular Medicine.
Philadelphia, Pa: Churchill Livingstone; 2000:833-884
page 10
A 15-year-old girl presents to the emergency department with a 4-week history of nasal
drainage and face pain and a 2-week history of frontal headaches and fatigue. Her mother
complains that her daughter has an "attitude" and has not been respectful or seemed to care
about anything for the past 2 weeks. The daughter awoke this morning with a headache and
vomited. On physical examination, the adolescent is afebrile and has normal vital signs. She
responds slowly to questions and is not oriented to the date. She complains of pain to palpation
of her cheeks and forehead. She has no nuchal rigidity and no focal weakness. The remainder
of the physical examination findings are normal.
Of the following, the BEST initial diagnostic procedure is
page 11
Preferred Response: A
The subacute onset of mental status changes described for the adolescent in the vignette
warrants an emergency evaluation. In most cases, neuroimaging is indicated, along with
appropriate laboratory testing.
The relatively nonspecific pain and what her mother perceives as common emotional
problems (apathy in a teenager) probably represent early frontal lobe symptoms. The headache
on awakening and vomiting are concerning for increased intracranial pressure (ICP). Confusion
and psychomotor retardation on the mental status examination indicate involvement of the
central nervous system. A focal, ischemic, ictal, infectious/inflammatory, or toxic/metabolic
process must be identified urgently. A brain abscess is suggested by the prominent facial pain in
this setting; the sinuses are a common source of brain abscesses (Item C6). Brain abscesses
often present only with nonspecific pain and not with fever.
Head computed tomography (CT) scan is preferred for this patient because the
constellation of pain, confusion, and morning vomiting makes a focal intracranial mass a
possibility. Increased ICP is associated with morning vomiting because ICP is highest in the
morning. Contrast is recommended because of the insidious onset, which could indicate either a
neoplasm or infectious process. Intravenous contrast is not needed for all neuroimaging
procedures. However, it increases the diagnostic yield of imaging studies where either neoplasm
or infection is suspected because both typically involve some degradation of the blood-brain
barrier or hypervascularity, resulting in contrast enhancement at the site of the lesion. Magnetic
resonance imaging (MRI) with contrast also is a good choice. The advantage of MRI is higher
spatial and soft-tissue resolution. Disadvantages of MRI compared with CT include: 1) less
availability for emergency department studies; 2) need for pharmacologic sedation in agitated
patients because sedation affects mental status, thereby masking disease-related mental
status; 3) longer time in the scanner, which could delay treatment decisions; and 4) cost. Thus,
in most cases, a head CT scan with contrast is preferred as the initial study in the emergency
department.
Electroencephalography (EEG) is an important test for assessment of a patient who has
encephalopathy of unclear cause to rule out nonconvulsive status epilepticus (NCSE),
particularly if the patient is known to have epilepsy. If an EEG cannot be obtained rapidly,
intravenous administration of 0.1 mg/kg lorazepam can treat NCSE immediately, although this
would not clear confusion about other causes. For this adolescent, the facial pain makes the
diagnosis of NCSE less likely than a brain abscess.
A lumbar puncture may be needed to rule out meningitis or encephalitis, but the pain and
morning vomiting more strongly suggest the possibility of an intracranial mass. Lumbar puncture
prior to head CT is not advised in this case because it could reduce pressure below the foramen
magnum and result in herniation from the supratentorial mass. Toxicology screening and nasal
swabs are reasonable but not the preferred initial diagnostic tests because they will not affect
emergency management of the increased intracranial pressure.
References:
Goodkin HP, Harper MB, Pomeroy SL. Intracerebral abscess in children: historical trends at
Children's Hospital Boston. Pediatrics. 2004;113:1765-1770. Available at:
http://pediatrics.aappublications.org/cgi/content/full/113/6/1765
Haslam RHA. Brain abscess. In: Kliegman RM, Behrman RE, Jenson HB, Stanton BF, eds.
Nelson Textbook of Pediatrics. 18th ed. Philadelphia, Pa: Saunders Elsevier; 2007:2524-2525
Kan L, Nagelberg J, Maytal J. Headaches in a pediatric emergency department: etiology,
imaging, and treatment. Headache. 2000;40:25-29. Abstract available at:
http://www.ncbi.nlm.nih.gov/pubmed/10759899
page 12
A 5-year-old girl who is new to your practice presents to the clinic for a prekindergarten physical
examination. Her primary caretaker, the maternal grandmother, reports that the childs mother
used multiple street drugs throughout her pregnancy as well as medications prescribed for
seizure and bipolar disorders. The grandmother is concerned that this childs speech
development is delayed. On physical examination, you note that the girl has wide-spaced eyes,
a short nose, and midface hypoplasia.
Of the following, the substance that is MOST likely to be associated with this childs dysmorphic
features is
A. lithium
B. lysergic acid diethylamide (LSD)
C. marijuana
D. methamphetamine
E. phenobarbital
page 13
Preferred Response: E
The features described for the child in the vignette are most consistent with fetal anticonvulsant
syndrome, which can occur following exposure to numerous medications, including phenytoin,
carbamazepine, valproate, and phenobarbital. Multiple authors have observed a 10% to 20%
incidence of birth defects in infants exposed to phenobarbital in utero. Anomalies include midface
hypoplasia, ocular hypertelorism (Item C7A), nail hypoplasia (Item C7B), cleft lip+/-cleft palate,
and heart defects as well as developmental delay and pre- and postnatal growth failure.
Phenobarbital-exposed newborns may exhibit a withdrawal syndrome that is evidenced by
tremulousness and increased activity. Of interest, such infants are likely to have lower serum
bilirubin concentrations than nonexposed neonates.
Prenatal exposure to lithium is associated with an increased risk of cardiac malformations
(eg, Ebstein anomaly) in the fetus. If the mother takes lithium near term, the exposed neonate
may have cyanosis, hypotonia, abnormalities of cardiac rhythm, goiter, hypothyroidism, and
nephrogenic diabetes insipidus. Lithium exposure is not associated with dysmorphic features or
developmental delays.
Despite popular belief, lysergic acid diethylamide (LSD) generally is not associated with birth
defects or withdrawal symptoms in prenatally exposed infants. Although there are isolated case
reports of birth defects in exposed neonates, an increased risk for anomalies is not borne out by
epidemiologic studies. It is important to note, however, that LSD users often abuse other
substances, underscoring the importance of taking an in-depth drug/substance abuse history in
pregnant women.
Marijuana use during pregnancy is not known to be associated with an increased incidence
of birth defects, dysmorphic features, or developmental delay in exposed offspring, although
further study is needed in this regard. Some investigations have shown reduced fetal growth in
exposed pregnancies, but this is not confirmed. Prenatally exposed newborns may have
tremulousness, increased irritability, and abnormal visual response to light stimulus.
Although there does not appear to be an increase in congenital anomalies associated with
methamphetamine use during pregnancy, further study is necessary. There are reports of
decreased birthweight in exposed neonates. A neonatal withdrawal syndrome consisting of
abnormal sleep patterns, tremulousness, poor feeding, and increased tone has been observed
commonly. Once again, it is important to consider polydrug abuse in these instances.
As with all teratogens, the timing of exposure is critical, with the most vulnerable period of
embryonic development occurring between 18 and 60 days after conception, during
organogenesis. Dosage of the offending agent also is important, as are route of administration,
modifying environmental factors, and genetic background of the mother and fetus.
References:
Gallagher RC, Kingham K, Hoyme HE. Fetal anticonvulsant syndrome. In: Cassidy SB, Allanson
JE, eds. Management of Genetic Syndromes. 2nd ed. Hoboken, NJ: Wiley-Liss; 2005:239-250
Phenobarbital, lithium, LSD, marijuana, methamphetamine. Reprotox. Available for subscription
at: http://www.reprotox.org
Phenobarbital, lithium, LSD, marijuana, methamphetamine. Teris. Available for subscription at:
http://depts.washington.edu/terisweb/teris
page 14
A 15-year-old girl comes to the urgent care clinic complaining of lower abdominal pain for 48
hours. She is nauseated but has had no fever, vomiting, or diarrhea. She is afebrile and denies
abdominal trauma. She localizes the pain to the left lower quadrant and describes it as
intermittent, stabbing pain episodes separated by intervals of more continuous dull pain. She has
never been sexually active. Her last menstrual period was 1 week ago. She has had no vaginal
discharge or itching. On physical examination, she has left lower quadrant guarding and rebound
tenderness. Her pelvic examination shows no vaginal discharge or uterine tenderness, although
there is an exquisitely tender mass in the left adnexal area.
Of the following, the MOST likely diagnosis is
A. appendicitis
B. endometritis
C. ovarian torsion
D. sacroiliitis
E. splenic rupture
page 15
Preferred Response: C
Torsion of any adnexal structure, including the ovary, can result in an acute pelvic mass (Item
C8). The most common causes of adnexal torsion in young women are cysts and neoplasms,
with about 6% of torsions in one series occurring in the setting of normal-appearing ovaries.
Histopathology was benign in more than 90% of cases in this series. Torsion occurs when
masses cause the ovary to swing on its vascular pedicle, and larger masses generally are
associated with a greater potential for torsion until the size of the mass impedes movement. The
onset of pain associated with torsion can be abrupt, sharp, and very severe. However, with
intermittent or partial torsion, intense periods of intermittent pain may be separated by
generalized aching during detorsion, as described for the teenager in the vignette. Pain can
occur for several days or weeks prior to a complete torsion. Nausea or vomiting can
accompany severe pain. If adnexal torsion is diagnosed, prompt intervention (untwisting the
adnexa usually via laparoscopy) is indicated to preserve ovarian function.
Appendicitis is in the differential diagnosis for lower abdominal pain, but is less likely when
pain occurs in the left lower quadrant. Splenic rupture is a surgical emergency, like ovarian
torsion, but the finding of an adnexal mass on examination is unlikely. Endometritis is not very
likely in a young woman who is not sexually active and more commonly presents with midline
uterine pain. An orthopedic cause for this patient's pain (eg, sacroiliitis) is unlikely, but it remains
in the differential diagnosis of pelvic pain. Sacroiliitis usually presents with low back or hip pain.
References:
Adams Hillard PJ. Pelvic masses. In: Neinstein LS, eds. Adolescent Health Care: A Practical
Guide. 5th ed. Philadelphia, Pa: Lippincott Williams & Wilkins; 2008:706-713
Growdon WB, Laufer MR. Ovarian torsion. UpToDate Online 15.3. 2008. Available for
subscription at: http://www.utdol.com/utd/content/topic.do?topicKey=gyn_surg/5273
Laufer MR, Goldstein DP. Gynecologic pain: dysmenorrhea, acute and chronic pelvic pain,
endometriosis, and premenstrual syndrome. In: Emans SJH, Laufer MR, Goldstein DP, eds.
Pediatric and Adolescent Gynecology. 5th ed. Philadelphia, Pa: Lippincott, Williams & Wilkins;
2005:417-476
Varras M, Tsikini A, Polyzos D, Samara Ch, Hadjopoulos G, Akrivis Ch. Uterine adnexal torsion:
pathologic and gray-scale ultrasonographic findings. Clin Exp Obstet Gynecol. 2004;31:34-38.
Abstract available at: http://www.ncbi.nlm.nih.gov/pubmed/14998184
page 16
A 2-year-old boy who has chronic renal failure is brought to the emergency department for
evaluation of nausea, fatigue, and muscle weakness. On physical examination, the boy has a
heart rate of 140 beats/min, decreased perfusion, and palpable pulses. You obtain
electrocardiography (Item Q9). Electrolyte measurements include a potassium concentration of
7.5 mEq/L (7.5 mmol/L) and a glucose value of 72.0 mg/dL (4.0 mmol/L).
Of the following, the MOST appropriate initial treatment is administration of
A. calcium chloride
B. insulin
C. normal saline 20 mL/kg
D. sodium polystyrene sulfonate
E. verapamil
page 17
Preferred Response: A
Hyperkalemia can be asymptomatic or produce symptoms such as nausea, fatigue, and muscle
weakness, as reported for the boy in the vignette. The most concerning effect of hyperkalemia
is on cardiac membrane polarization. Electrocardiographic changes typically consist of peaked
T waves and progress to widening of the QRS complexes with dampening of P waves as the
hyperkalemia increases. Without treatment, the patient eventually develops ventricular fibrillation
or asystole.
The electrocardiographic tracing of the boy described in the vignette demonstrates peaked
T waves (Item C9), and he has decreased perfusion. The immediate priority is to stabilize his
cardiac membrane potential and decrease his risk of arrhythmias, which is accomplished best
by administration of intravenous calcium chloride, an agent that has a rapid onset of action.
Insulin causes potassium to shift to the intracellular spaces, thereby decreasing serum
concentrations, but it always should be administered in conjunction with glucose to avoid
hypoglycemia. Sodium bicarbonate administered intravenously also produces intracellular shifts
of potassium and may be useful in patients who have metabolic acidosis.
Treatments to enhance the elimination of potassium often are indicated in the treatment of
hyperkalemia, but they have a slower onset of action and, therefore, should not be the first line
of treatment in life-threatening hyperkalemia. Sodium polystyrene sulfonate, which can be
administered orally or rectally, exchanges sodium for potassium, which subsequently is
eliminated from the body. Loop diuretics also can enhance potassium elimination. Dialysis may
be indicated when ongoing elevated potassium concentrations are expected, such as in renal
failure, or with very high acute concentrations, as seen with tumor lysis syndrome or
rhabdomyolysis.
Administration of verapamil is not indicated in the treatment of hyperkalemia; it has been
reported to be associated with the development of complete heart block in this setting. Normal
saline has a minimal effect on hyperkalemia.
References:
Greenbaum LA. Electrolyte and acid-base disorders: potassium. In: Kliegman RM, Behrman RE,
Jenson HB, Stanton BF, eds. Nelson Textbook of Pediatrics. 18th ed. Philadelphia, Pa:
Saunders Elsevier; 2007:279-284
Hauser GJ, Kulick AF. Electrolyte disorders in the pediatric intensive care unit. In: Wheeler DS,
Wong HR, Shanley TP, eds. Pediatric Critical Care Medicine: Basic Science and Clinical
Evidence. New York, NY: Springer-Verlag; 2007:1156-1175
page 18
A 13-year-old girl who has just moved to the United States from Brazil comes to your office
because her mother is worried that she is not "developing yet." On physical examination, her
height is 50 inches, and she has a triangular face, a low hairline, high-arched palate, and a shieldshaped chest (Item Q10). Breast tissue is not visible or palpable, but there is Sexual Maturity
Rating 3 pubic hair. You obtain bone age radiography and a karyotype and measure serum
luteinizing hormone and follicle-stimulating hormone.
Of the following, the MOST appropriate additional laboratory measurement is
A. adrenocorticotropic hormone
B. prolactin
C. 17-hydroxyprogesterone
D. testosterone
E. thyroid-stimulating hormone
page 19
Preferred Response: E
The clinical findings described for the girl in the vignette are characteristic of Turner syndrome
(gonadal dysgenesis) associated with an abnormality of one X chromosome. Girls who have this
disorder usually are short (mean adult height, approximately 55 inches without growth hormone
treatment); have poorly developed ovaries; and often have dysmorphisms, including a triangular
facies, low hairline, high-arched palate, hypoplastic nipples, and an increased carrying angle.
They may have left heart disorders such as coarctation of the aorta as well as horseshoe
kidney or other renal malformations.
Initial screening studies to diagnose Turner syndrome include a karyotype and
measurement of luteinizing hormone (LH) and follicle-stimulating hormone (FSH). Most girls who
have Turner syndrome do not initiate normal puberty. Concentrations of LH and FSH rise as
they reach pubertal age range because they have ovarian failure. Although concentrations of
estradiol and other estrogens are low, clinical estradiol assays are not designed to provide
accurate values in the low-normal range expected in early puberty. Therefore, physical findings
such as breast development are a better marker of estrogen effect than measurements of
estrogen.
Adolescents who have Turner syndrome are at higher risk of developing chronic
lymphocytic thyroiditis and hypothyroidism than the general population. Approximately 20% of
affected adolescent girls have antibody-positive autoimmune chronic lymphocytic thyroiditis, and
5% to 10% develop overt hypothyroidism. Accordingly, measurement of thyroid-stimulating
hormone is an appropriate laboratory test for patients such as the girl described in the vignette.
An elevated value indicates primary hypothyroidism and the need for confirmatory assessment
of free thyroxine and antithyroid antibodies (thyroperoxidase, antimicrosomal, or
antithyroglobulin).
Abnormalities of the hypothalamic-pituitary-adrenal axis are unusual in patients who have
Turner syndrome. Therefore, measurement of adrenocorticotropic hormone is not useful.
Measurement of prolactin would be useful if the girl had a pituitary or hypothalamic problem, but
her clinical findings strongly point to Turner syndrome. A 17-hydroxyprogesterone value would
be elevated in the presence of an adrenal biosynthetic defect leading to the development of the
most common form of congenital adrenal hyperplasia (cyp21 or 21-hydroxylase deficiency) as
well as some of the less common disorders of adrenal biosynthesis. Measuring testosterone
would be reasonable if there were evidence of inappropriate masculinization, such as
clitoromegaly and a growth spurt. Some girls who have Turner syndrome have functioning Y
chromosomal DNA and could have androgenization, but this is unusual. The presence of Y
chromosomal DNA does increase the risk of gonadal malignancy, and girls who have significant
Y chromosomal DNA on testing often require prophylactic gonadectomy.
References:
Chiovato L, Larizza D, Bendinelli G, et al. Autoimmune hypothyroidism and hyperthyroidism in
patients with Turner's syndrome. Eur J Endocrinol. 1996;134:568-575. Abstract available at:
http://www.ncbi.nlm.nih.gov/pubmed/8664977
Doswell BH, Visootsak J, Brady AN, Graham JM Jr. Turner syndrome: an update and review for
the primary pediatrician. Clin Pediatr. 2006;45:301-313. Abstract available at:
http://www.ncbi.nlm.nih.gov/pubmed/16703153
Frias JL, Davenport ML, Committee on Genetics and Section on Endocrinology. Health
supervision for children with Turner syndrome. Pediatrics. 2003;111:692-702. Available at:
http://pediatrics.aappublications.org/cgi/content/full/111/3/692
Matura LA, Ho VB, Rosing DR, Bondy CA. Aortic dilatation and dissection in Turner syndrome.
Circulation. 2007;116:1663-1670. Abstract available at:
http://www.ncbi.nlm.nih.gov/pubmed/17875973
page 20
page 21
A 2-month-old infant has lost the vision in both of his eyes due to bilateral retinoblastoma. His
distressed parents ask how the infants blindness will affect his behavior and development.
Of the following, the child MOST likely will
page 22
Preferred Response: B
Legal blindness is defined as central visual acuity with corrective lenses of 20/200 or less in the
strongest eye or a limited visual field that extends to an angle of 20 degrees. Congenital
blindness occurs in 30 per 100,000 births. More than 50% of children who have visual
impairment also have developmental disabilities, such as cognitive-adaptive disability, seizures,
hearing impairments, and learning disorders. In many of these cases, the disabilities result from
central nervous system pathology. Postnatal blindness, which accounts for approximately 8% to
11% of all childhood blindness, can be caused by infections, trauma, or tumors. Retinoblastoma
is the most common primary malignant intraocular tumor of childhood. The initial finding in most
cases is a white pupillary reflex (leukokoria) (Item C11). Advanced tumors may be treated with
enucleation.
Children who have congenital or acquired (eg, due to retinoblastoma) blindness without
associated neurologic abnormalities should not be at increased risk for motor or cognitive
impairment. They are not at increased risk for language-based learning disabilities or autism
spectrum disorders. However, children who have significant visual impairment may begin to
walk at an older age (18 to 22 months) than sighted children due to different exposure to motor
exploration. They typically develop language skills at the same time (12 months) as sighted
children. Children who have visual impairments should be provided with much physical contact
that includes hugging and comforting. They should be encouraged to partake in self-help skills
and exploration of their environment.
References:
Davidson PW, Burns CM. Visual impairment and blindness. In: Levine MD, Carey WB, Crocker
AC, eds. Developmental- Behavioral Pediatrics. 3rd ed. Philadelphia, Pa: WB Saunders
Company; 1999:571-578
Msall ME. Visual impairment. In: Parker S, Zukerman B, Augustyn M. Developmental and
Behavioral Pediatrics: A Handbook for Primary Care. 2nd ed. Philadelphia, Pa: Lippincott
Williams & Wilkins; 2005:366-369
Olitsky SE, Hug D, Smith LP. Disorders of vision. In: Kleigman RM, Behrman RE, Jenson HB,
Stanton BF, eds. Textbook of Pediatrics. 18th ed. Philadelphia, Pa: Saunders Elsevier;
2007:2573-2576
page 23
A 15-year-old boy presents to the clinic because of a persistent cough. According to his mother,
his cough has been present for approximately 2 weeks, but it seems to be getting worse. He
does not cough all the time, but the coughing episodes tend to come in bursts. This morning she
became very worried because he passed out during a coughing spell. Physical examination
reveals a healthy-appearing male in no apparent distress. He is afebrile, and his vital signs are
normal. He has petechiae on his face but no other skin lesions. His lungs are clear.
Of the following, the MOST appropriate antimicrobial agent to prescribe for this patient is
A. azithromycin
B. clarithromycin
C. doxycycline
D. erythromycin
E. trimethoprim-sulfamethoxazole
page 24
Preferred Response: A
Any patient who has episodic coughing episodes that conclude in syncope or vomiting, such as
the boy described in the vignette, should be considered to have pertussis. Other complications
from pertussis described in adolescents include urinary incontinence, sleep interruption, rib
fractures, and pneumonia. Despite universal immunization of children against pertussis, a
marked increase in disease incidence has been demonstrated among adolescents of 11 to 18
years of age. In an attempt to address this problem, the American Academy of Pediatrics
recommends that adolescents in this age group receive a single dose of tetanus toxoid, reduced
diphtheria toxoid, and acellular pertussis (Tdap) for booster immunization. For those who
received only a tetanus toxoid and reduced diphtheria toxoid (Td) booster, a booster with Tdap
is suggested if the interval since the Td administration is at least 2 years or if the adolescent is
living in a setting of increased disease, has a risk of a complicated course if he or she acquires
the disease, or possibly can transmit infection to a vulnerable contact.
Treatment of pertussis with antimicrobial agents once the cough has started does not affect
the course of the illness but is recommended to limit the spread of disease to others. In the past,
the treatment of choice was erythromycin, but recent data have demonstrated that the
effectiveness of other macrolides such as azithromycin or clarithromycin is similar to that of
erythromycin, and they have fewer adverse effects. The most common complaints of patients
who take erythromycin preparations are gastrointestinal irritation, including epigastric distress,
abdominal cramping, nausea, vomiting, and diarrhea. The occurrence of these adverse effects
can result in poor adherence to treatment regimens. In addition, erythromycin cannot be used in
children younger than 1 month of age due to its association with the development of hypertrophic
pyloric stenosis. Azithromycin and clarithromycin attain higher tissue concentrations than
erythromycin, have longer half-lives, and can be administered less often and for a shorter total
duration. With the advent of these newer macrolides, erythromycin rarely is indicated as a firstline agent for any illness.
Azithromycin has become the agent of choice for treatment of pertussis because of the
ease of administration (once daily for 5 days) and the fact that it does not inhibit cytochrome P450, as erythromycin and clarithromycin do. Therefore, it does not interact with other
medications that are metabolized by this system (eg, digoxin, carbamazepine). Trimethoprimsulfamethoxazole also is effective against pertussis and is considered an alternative for patients
who cannot tolerate a macrolide or have a macrolide-resistant isolate if they are older than 2
months of age. Doxycycline is not recommended for the treatment of pertussis.
References:
American Academy of Pediatrics. Pertussis (whooping cough). In: Pickering LK, Baker CJ, Long
SS, McMillan JA, eds. Red Book: 2006 Report of the Committee on Infectious Diseases. 27th ed.
Elk Grove Village, Ill: American Academy of Pediatrics; 2006:498-520
Committee on Infectious Diseases. Prevention of pertussis among adolescents:
recommendations for use of tetanus toxoid, reduced diphtheria toxoid, and acellular pertussis
(Tdap) vaccine. Pediatrics. 2006;117:965-978. Available at:
http://pediatrics.aappublications.org/cgi/content/full/117/3/965
Tiwari T, Murphy TV, Moran J. Recommended antimicrobial agents for the treatment and
postexposure prophylaxis of pertussis: 2005 CDC guidelines. MMWR Recomm Rep.
2005;54(RR14):1-16. Available at: http://www.cdc.gov/mmwr/preview/mmwrhtml/rr5414a1.htm
page 25
A 15-year-old male presents for evaluation of a progressively enlarging lesion on his left forearm
that began 5 days ago. He explains that the lesion initially looked like a "spider bite" with a blister,
but over the last several days, a black scab has developed in the center of the lesion, and there
is a large area of redness around the scab. The lesion has been pruritic but not painful. Except
for low-grade fevers for the last 2 days, he has had no other systemic symptoms. He returned 1
week ago from a school trip to Morocco, where he visited a leather tannery, went shopping in
the large outdoor marketplace, visited some historic sites, and took a camel ride in the desert.
He states that the students stayed in a hostel in Morocco, but there were no screens on the
windows, and spiders, ants, and other insects were visible in the rooms. On physical
examination, the boy is afebrile, and his left forearm is edematous, with a 3x3-cm black eschar
surrounded by a 5-cm area of erythema and induration (Item Q13). The lesion is not tender to
palpation, and there is no drainage. There are several 1.5-cm tender lymph nodes in his left
axilla. Findings on the remainder of his examination are within normal limits.
Of the following, the MOST likely cause of this patients lesion is
A. Bacillus anthracis
B. Francisella tularensis
C. Loxosceles laeta
D. methicillin-resistant Staphylococcus aureus
E. Yersinia pestis
page 26
Preferred Response: A
page 27
Swanson DL, Vetter RS. Bites of brown recluse spiders and suspected necrotic arachnidism. N
Engl J Med. 2005;352:700-707. Extract available at:
http://content.nejm.org/cgi/content/extract/352/7/700
page 28
A 3-month-old infant who has a history of renal dysplasia associated with obstructive uropathy
has marked polyuria. He is breastfeeding and receiving supplemental cow milk-based formula.
In an effort to reduce the high urine output, you consider reducing the renal solute load by
changing feedings from the milk-based formula currently being used.
Of the following, the MOST appropriate change is to
page 29
Preferred Response: C
The infant described in the vignette has polyuria caused by a urinary concentrating defect. The
concentrating defect is the result of tubular damage due to the obstructive uropathy. The inability
to concentrate the urine causes the kidneys to create an "excessive" volume of urine to excrete
the solute load presented to them.
One strategy to reduce polyuria is to reduce the solute burden placed on the kidneys.
Potential renal solute load is affected by intake of protein, sodium, potassium, chloride, and
phosphorus. The protein and phosphorus content are the most important variables when
comparing infant feeding regimens.
Human milk possesses a lower potential renal solute load than cow milk or cow milk-based
formulas. Accordingly, the most appropriate change in feeding for the infant in the vignette is to
recommend that the mother stop cow milk formula supplementation and exclusively breastfeed.
If human milk is not available, a "low-solute" cow milk-based formula can be used. A low calciumphosphorus formula has the next lowest potential renal solute load compared with human milk.
Cow milk, soy milk-based formula, hydrolyzed formula with medium-chain triglycerides, and 24kcal milk-based formula all have greater renal solute loads than human milk.
Renal solute load should also be considered in nephrogenic diabetes insipidus.
References:
Fiorino KN, Cox J. Nutrition and growth. In: Robertson J, Shilkofski N, eds. Harriet Lane
Handbook: A Manual for Pediatric House Officers. 17th ed. Philadelphia, Pa: Elsevier Mosby;
2005:525-608
Hall RT, Carroll RE. Infant feeding. Pediatr Rev. 2000;21:191-200. Available at:
http://pedsinreview.aappublications.org/cgi/content/full/21/6/191
Linshaw MA. Congenital nephrogenic diabetes insipidus. Pediatr Rev. 2007;28:372-380.
Available at: http://pedsinreview.aappublications.org/cgi/content/full/28/10/372
Ziegler EE, Fomon SJ. Potential renal solute load of infant formulas. J Nutr. 1989;119 (12
suppl):1785-1788. Available at: http://jn.nutrition.org/cgi/reprint/119/12_Suppl/1785
page 30
A mother brings in her 13-month-old daughter for evaluation because her girl developed a
perioral rash and "hives" on two occasions last week. One episode occurred while eating yogurt
and another happened immediately after eating a bagel with cream cheese. She states that her
daughter has eaten other foods such as eggs and bread without problems but is breastfeeding
and never has been given milk-based formulas or cow milk. The infant has been given rice milk,
but she became fussy and seems to prefer breastfeeding. The mother is concerned that her
daughter may be allergic to milk but would like to stop breastfeeding.
Of the following, the BEST advice is to recommend
page 31
Preferred Response: E
Milk protein allergy is an immunoglobulin (Ig) E-mediated food reaction that affects 2% to 3% of
infants within the first postnatal year. Typical symptoms include urticaria, angioedema, atopic
dermatitis, and anaphylaxis. With IgE-mediated reactions, the quantity of milk required to result in
a reaction often is minimal (eg, milk touching the face, a taste of ice cream). Taking a detailed
history about the specific food(s) involved, timing of the onset of symptoms, and type of
symptoms is important to distinguish IgE-mediated reactions, as described for the child in the
vignette, from other adverse milk reactions, such as milk protein enterocolitis and lactose
intolerance. Once an IgE-mediated food allergy is suspected, the clinician should consider
allergy skin testing or serum IgE testing for the suspected food.
While awaiting results from either blood testing or allergy consultation for skin testing, the
first reasonable action is to switch to a soy-based formula. Approximately 10% to 15% of infants
and children who have IgE-mediated milk protein allergies may not tolerate soy formula, but this
risk applies to infants younger than 6 months of age. For infants older than 6 months, the risk is
closer to 5%. Nonetheless, the initial soy formula trial should be performed in the clinic. Other
acceptable formula options in this scenario include an extensively hydrolyzed or an amino acidbased formula, although the unpleasant taste and significantly higher cost can be limiting for
many families. Also, because almost all affected infants can be fed successfully with a soy or
extensively hydrolyzed formula, switching initially to an amino acid-based formula is not required.
Food challenges often are used to assess adverse food reactions, but they generally are
reserved for foods that are unlikely allergens or if the clinical history is inconsistent or vague (eg,
a patient who eats a particular food and does not always have a reaction). Food challenges may
result in anaphylaxis and generally are avoided when the history and testing results support an
IgE-mediated reaction.
Infants who have a specific food allergy sometimes are placed incorrectly on restricted diets
that avoid multiple foods. Without a specific history of other adverse food reactions, avoidance of
other foods such as egg or wheat is not recommended. However, parents should be counseled
that children can develop other food allergies and should monitor their children during ingestion
of other common food allergens.
Breastfeeding until age 3 years old is an option that is not preferred by the mother in the
vignette. Although most IgE-mediated cow milk allergies resolve by 3 years of age, milk protein
allergy can persist past 5 years of age in up to 20% of affected children.
References:
American Academy of Pediatrics Committee on Nutrition. Soy protein-based formulas:
recommendations for use in infant feeding. Pediatrics. 1998;101:148-153. Available at:
http://pediatrics.aappublications.org/cgi/content/full/101/1/148
Bhatia J, Greer F, and the Committee on Nutrition. The use of soy protein-based formulas in
infant feeding. Pediatrics. 2008;121:1062-1068. Available at:
http://pediatrics.aappublications.org/cgi/content/full/121/5/1062
Klemola T, Vanto T, Juntunen-Backman K, Kalimo K, Korpela R, Varjonen E. Allergy to soy
formula and to extensively hydrolyzed whey formula in infants with cow's milk allergy: a
prospective, randomized study with a follow-up to the age of 2 years. J Pediatr. 2002;140:219224. Abstract available at: http://www.ncbi.nlm.nih.gov/ pubmed/11865274
Saarinen KM, Pelkonen AS, Mkel MJ, Savilahti E. Clinical course and prognosis of cow's milk
allergy are dependent on milk-specific IgE status. J Allergy Clin Immunol. 2005;116:869-875.
Abstract available at: http://www.ncbi.nlm.nih.gov/pubmed/16210063
Sampson HA, Leung DYM. Adverse reactions to foods. In: Kleigman RM, Behrman RE, Jenson
HB, Stanton BF, eds. Nelson Textbook of Pediatrics. 18th ed. Philadelphia, Pa: Saunders
Elsevier; 2007:986-989
page 32
A 16-year-old girl is brought to the emergency department after being found unresponsive in her
bedroom. Her parents report finding a note in which she wrote of "wanting to end the pain." In
addition, they found several empty, unlabeled pill vials on her dresser. On physical examination,
the girl is responsive only to painful stimuli. Her heart rate is 60 beats/min, respiratory rate is 16
breaths/min, blood pressure is 90/60 mm Hg, and oxygen saturation is 92%. Her pupils are 3
mm, equal in size, and sluggishly reactive. The remainder of findings on her physical
examination are normal.
Of the following, the MOST important diagnostic test to obtain when evaluating this patient is a
A. carboxyhemoglobin concentration
B. complete blood count
C. serum acetaminophen concentration
D. serum ammonia concentration
E. serum osmolality
page 33
Preferred Response: C
Treatment of a patient who has ingested an unknown substance or substances, such as the
one described in the vignette, should focus initially on stabilization of vital functions; treatment of
the patient's symptoms; and identification of agents that are potentially fatal, have delayed clinical
toxicity, or for which antidotal therapy is indicated. The evaluation should include a screening
history to identify circumstances surrounding the event, potential environmental exposures,
available medications or toxins, and previous medical or psychiatric history. The physical
examination should focus on vital sign abnormalities, pupillary size and reactivity, skin findings,
and mental status. These components are likely to yield the most useful clues to toxins that
have identifiable symptom complexes (Item C16A).
Laboratory testing should assess the patient's acid/base status, oxygenation and ventilation,
glucose concentration, and anion gap. Qualitative urine drug testing may be obtained but has
significant limitations, including the small number of drugs tested (typically drugs of abuse) and
screening thresholds that may produce false-negative results. Therefore, the results of urine
toxicologic screening rarely affect treatment plans. Quantitative drug concentrations should be
measured based on the information gleaned from the initial history, physical examination, and
screening laboratory testing, although acetaminophen and salicylate concentrations should be
measured for every patient who has an unknown, mixed, or intentional ingestion. Acetaminophen
is of particular concern because it causes few initial symptoms, is a common agent in
adolescent and adult intentional ingestions, and may require antidotal therapy to prevent potential
fatal liver damage. Abdominal radiographs may be considered to examine the patient for radioopaque substances (Item C16B).
A complete blood count is not likely to provide information leading to the identification of a
toxin. Measurement of serum ammonia may be indicated if acetaminophen is identified as the
ingested agent because toxicity can lead to hepatic damage. Carboxyhemoglobin should be
measured in the patient whose history is suggestive of carbon monoxide exposure. Increased
serum osmolality may provide indirect evidence of alcohol ingestion, although a quantitative
serum test for alcohols is more useful in this setting.
References:
Erikson TB, Thompson TM, Lu JJ. The approach to the patient with an unknown overdose.
Emerg Med Clin North Am. 2007;25:249-281. Abstract available at:
http://www.ncbi.nlm.nih.gov/pubmed/17482020
Lavallee M, Olsson J Jr, Cheng TL. In brief: unknown poison. Pediatr Rev. 2004;25:370-371.
Available at: http://pedsinreview.aappublications.org/cgi/content/full/25/10/370
McKay CA Jr. Can the laboratory help me? Toxicology laboratory testing in the possibly
poisoned pediatric patient. Clin Pediatr Emerg Med. 2005;6:116-122
Valez LI, Shepherd JG, Goto CS. Approach to the child with occult toxic exposure. UpToDate
Online 15.3. 2008. Available for subscription at:
http://www.utdol.com/utd/content/topic.do?topicKey=ped_tox/3023&selectedTitle=4~150&source
=search_result
page 34
You are following a 3-month-old infant who was born at 30 weeks gestation, underwent a distal
ileal resection for necrotizing enterocolitis at 2 weeks of age, and subsequently was placed on
parenteral nutrition for 2 months. The baby has residual cholestasis from the parenteral nutrition
(total bilirubin, 5.0 mg/dL [85.5 mcmol/L]; direct bilirubin, 3.0 mg/dL [51.3 mcmol/L]). Currently,
she is receiving a cow milk protein hydrolysate formula concentrated to 24 kcal/oz (0.8
kcal/mL). You are considering adding a dietary supplement to increase the caloric density of the
formula.
Of the following, the supplement that is the MOST likely to be tolerated and cause less diarrhea
in this infant is
A. flaxseed oil
B. medium-chain triglyceride oil
C. olive oil
D. omega-3 polyunsaturated fatty acid (fish oil)
E. soybean oil
page 35
Preferred Response: B
Infants who have chronic illnesses may have specialized nutritional requirements and often do
not tolerate the standard 20-kcal/oz formula given to healthy term infants. For example, children
who have some forms of congenital heart disease or renal disease may require a more
concentrated formula because the standard formula may lead to volume overload. Children who
have intestinal disease or malabsorption, such as the child described in the vignette, also may
require a more concentrated formula to decrease the likelihood of feeding intolerance or
diarrhea.
Formula may be concentrated by increasing the concentration of protein, carbohydrate, or
fat. Perhaps the easiest method of increasing the caloric density of a formula is to mix more
powder with the same amount of water. For example, four scoops of most commercially
available formulas mixed with 8 oz of water yields standard 0.67-kcal/mL (20-kcal/oz) formula,
but mixing five scoops in 8 oz results in 0.83-kcal/mL (25-kcal/oz) formula. However, exceeding
25-kcal/oz formula by increasing the amount of powder may yield too high a concentration of
protein, which could result in an excessive renal solute load. For this reason, carbohydrate and
fat supplements are available to concentrate infant and toddler feedings further. The most
common adverse effect of carbohydrate supplements is diarrhea, and lipid supplements may
cause either diarrhea or delayed gastric emptying. For these reasons, caution is recommended
when increasing the caloric density of a formula, especially when the caloric density is increased
to greater than 1 kcal/mL (30 kcal/oz).
Because the patient in the vignette has had an ileal resection and cholestasis, the best fat
supplement for him is oil composed of medium-chain triglycerides (MCT oil). MCT oil can be
absorbed directly across the enterocyte and does not require intraluminal digestion by bile acids.
In contrast, soy, olive, flaxseed, and fish oils are long-chain fatty acids that require bile acids for
digestion and might cause diarrhea in a child who has cholestasis and ileal resection.
References:
Courtney E, Grunko A, McCarthy T. Enteral nutrition. In: Hendricks KM, Duggan C. Manual of
Pediatric Nutrition. 4th ed. Hamilton, Ontario, Canada: BC Decker; 2005:252-316
Suchy FJ. Neonatal cholestasis. Pediatr Rev. 2004;25:388-396. Available at:
http://pedsinreview.aappublications.org/cgi/content/full/25/11/388
page 36
You have admitted a 750-g male infant to the neonatal intensive care unit (NICU) for treatment of
respiratory distress and presumed sepsis. The Apgar scores were 1, 5, and 7 at 1, 5, and 10
minutes, respectively. The infant received one dose of exogenous surfactant in the delivery
room. In the NICU, the infant is being cared for on a radiant warmer. At 4 hours after birth,
physical examination reveals a temperature of 97.0F (36.1C), heart rate of 180 beats/min,
respiratory rate of 40 breaths/min (assisted breaths on the ventilator), blood pressure of 45/27
mm Hg, mean arterial blood pressure of 30 mm Hg, and pulse oximetry of 92%. The infant is
receiving synchronized intermittent mechanical ventilation with a peak inflation pressure of 18
cm H2O over a positive end-expiratory pressure of 4 cm H2O at a rate of 40 breaths/min and an
FiO2 of 0.40. Umbilical catheters are present in the umbilical artery and vein. On physical
examination, you note a soft, flat anterior fontanelle. You auscultate equal mechanical breath
sounds bilaterally over the chest and note minimal subcostal retractions. The skin is thin and
somewhat moist, and many veins are visible through it. The ears are flattened against the
cranium and lack any cartilage or recoil. There is a small phallus and an empty scrotum. The
infant is laying on the warmer with legs and arms extended. The neuromotor tone is decreased,
and the infant does not appear to be very active, but he does respond to tactile stimuli with
movement of the arms and legs in seemingly random and purposeless activity.
Of the following, the MOST important next step is to
page 37
Preferred Response: D
The extremely low gestational-age newborn (ELGAN) whose birthweight is less than 1,000 g
may present with physical examination findings indicative of marked physical and
neurodevelopmental immaturity. These findings have been studied and correlated with
gestational age in scoring systems such as those published by Dubowitz in 1980 and Ballard in
1979 and (revised) 1991. Even the best efforts to estimate the gestational age of very lowbirthweight and immature infants at dates less than 28 completed weeks of gestation by
maternal last menstrual period and obstetric ultrasonography may be imprecise and tend to
overestimate maturity (gestational age) by 2 weeks. Nevertheless, the thin, moist, transparent
skin; flattened thin ears without cartilage or recoil; and small phallus and empty scrotum
described for the infant in the vignette indicate a degree of physical immaturity consistent with 28
weeks' gestation or less. Similarly, the low neuromotor tone, absence of flexed posture, relative
inactivity, and random purposeless flailing of extremities with tactile stimulation indicate
significant neurologic immaturity.
Moving the infant into an isolette incubator is an important next step in the care of this
ELGAN. The absolute benefit of using a radiant warmer bed (access to the infant to conduct
early examinations, admission nursing care, and procedures such as placement of umbilical
catheters) is eclipsed by its associated increased ambient convection, evaporative heat loss,
and transcutaneous water loss compared with the contained, convectively heated, and
humidified environment of the isolette. The isolette environment still allows access, visibility, and
treatment with phototherapy, but with less insensible water loss or variability in patient
temperature.
This ELGAN is being treated for respiratory distress syndrome and has had a good
response to exogenous surfactant administration, oxygen, and assisted ventilation. At 4 hours of
postnatal age, it is too early to administer a second dose of surfactant. Developmentally
cryptorchid testes may yet descend into the scrotal sac in coming weeks. An oxygen saturation
of 92% is good for this infant, and increasing the inspired oxygen concentration may be harmful,
contributing to oxidative injury of pulmonary tissues or retinopathy of prematurity. Cranial
ultrasonography is indicated in this sick and preterm infant, but without clinical indications of
hemorrhage, shock, and seizure or encephalopathy, such a study is performed best at 3 to 7
days of postnatal life.
References:
Donovan EF, Tyson JE, Ehrenkranz RA, et al. Inaccuracy of Ballard scores before 28 weeks'
gestation. National Institute of Child Health and Human Development Neonatal Research
Network. J Pediatr. 1999;135:147-152. Abstract available at:
http://www.ncbi.nlm.nih.gov/pubmed/10431107
Marn GMA, Martn Moreiras J, Llitera Fleixas G, et al. Assessment of the new Ballard score to
estimate gestational age [in Spanish]. An Pediatr (Barc). 2006;64:140-145. English abstract
available at: http://www.ncbi.nlm.nih.gov/pubmed/16527066
Sedin G. The thermal environment of the newborn infant. In: Martin RJ, Fanaroff AA, Walsh MC,
eds. Fanaroff and Martin's Neonatal-Perinatal Medicine. 8th ed. Philadelphia, Pa: Mosby
Elsevier; 2006:585-596
Thilo EH, Rosenberg AA. The newborn infant. In: Hay WW Jr, Levin MJ, Sondheimer JM,
Deterding RR, eds. Current Pediatric Diagnosis & Treatment. 18th ed. New York, NY: The
McGraw-Hill Companies, Inc; 2007:chap 1
page 38
You are called to the newborn nursery to evaluate a 1-day-old girl whose hands and feet are
blue. She was born at term via a cesarean section, and there were no complications. Apgar
scores were 9 at both 1 and 5 minutes. Her respiratory rate is 40 breaths/min, heart rate is 140
beats/min, and blood pressure is normal. Pulse oximetry is 98% on room air. Her lungs are
clear, and there is no murmur. Her lips are pink, but her hands and feet are cyanotic (Item Q19),
and capillary refill is less than 2 seconds.
Of the following, the MOST likely cause of her peripheral cyanosis is
A. cold environment
B. polycythemia
C. retained fetal lung fluid
D. sepsis
E. transposition of the great arteries
page 39
Preferred Response: A
Cyanosis is a common finding in newborns. The first step in the evaluation of an infant who
exhibits cyanosis is to determine whether it is central or peripheral (acrocyanosis). The infant
described in the vignette has clinical features suggestive of acrocyanosis, a bluish discoloration
of the hands and feet (Item C19A) in response to vasomotor instability or cold environment. In
this condition, the lips and mucous membranes are spared, although the perioral area may be
affected. The extremities may be cool to touch. Acrocyanosis is believed to be due to
vasoconstriction of small arterioles and resolves in the first few postnatal months. Infants who
have acrocyanosis require no further evaluation; parental reassurance is all that is needed.
Central cyanosis is the bluish discoloration of the tissues best observed in the lips, tongue,
mucous membranes, and occasionally nail beds (Item C19B). It occurs when the concentration
of deoxygenated (reduced) hemoglobin exceeds 4.0 to 6.0 g/dL (40.0 to 60.0 g/L) within the
capillary bed. In an individual who has normal hemoglobin, cyanosis usually becomes apparent
when the oxygen saturation drops below 75%-80%. Causes of central cyanosis in the neonatal
period typically are related to cyanotic heart disease, such as transposition of the great arteries,
or respiratory disease, such as pneumonia and retained fetal lung fluid (transient tachypnea of
the newborn). The absence of central cyanosis, a cardiac murmur, and respiratory distress for
the infant in the vignette makes these diagnoses less likely than acrocyanosis of infancy.
Polycythemia, defined as a venous hematocrit of at least 65% (0.65), causes a deep
reddish-purple discoloration of the skin and mucous membranes. Infants may be otherwise
asymptomatic or may exhibit lethargy, anorexia, or respiratory distress. Polycythemia does not
cause either peripheral or central cyanosis per se, but it does affect the percent saturation at
which cyanosis becomes clinically apparent. Cyanosis in infants who have polycythemia occurs
at higher oxygen saturations than in those who have anemia. For example, if an infant has a
hemoglobin of 24.0 g/dL (240.0 g/L), he is likely to appear cyanotic when his oxygen saturation
reaches 87%. In an infant who has a hemoglobin of 12.0 g/dL (120.0 g/L), cyanosis may not be
seen until the oxygen saturation falls to 75%. Newborns who have sepsis may develop cyanosis
as a component of multisystem involvement, but other signs, such as respiratory distress, poor
peripheral perfusion, lethargy, or poor feeding, also are evident.
References:
Bernstein D. Evaluation of the cardiovascular system: history and physical examination. In:
Kliegman RM, Behrman RE, Jenson HB, Stanton BF, eds. Nelson Textbook of Pediatrics. 18th
ed. Philadelphia, Pa: Saunders Elsevier; 2007:1857-1863
Sasidharan P. An approach to diagnosis and management of cyanosis and tachypnea in term
infants. Pediatr Clin North Am. 2004;51:999-1021. Abstract available at:
http://www.ncbi.nlm.nih.gov/pubmed/15275985
page 40
A frustrated mother requests referral of her 15-month-old child to an allergy and asthma
specialist because the boy never seems to have stopped coughing and wheezing over the 6
months of the past respiratory virus season. During the history taking, the mother states that the
baby only occasionally is exposed to wood smoke at the familys barbecue restaurant and to
cosmetic chemicals used at the grandmothers hair salon. Both the father and grandfather
smoke cigarettes in the home.
Of the following, the environmental exposure that is MOST likely to be causing the childs
respiratory symptoms is exposure to
A. cigarette smoke
B. cleaning fluids
C. dust mites
D. hairspray
E. wood smoke
page 41
Preferred Response: A
The child described in the vignette may have multiple triggers for his wheezing exacerbations,
although tobacco smoke probably is the one to which he is exposed most often that should be
diminished. Well-known tobacco toxins include nicotine, carbon monoxide, formaldehyde,
hydrogen cyanide, sulfur dioxide, nitrogen oxide, ammonia, polycyclic aromatic hydrocarbons,
and the nitrosamines. These substances produce both irritant and immunologic effects on the
respiratory tract. It is important to note that smoking cessation is difficult, and most pediatricians
have little training in the process. However, there is evidence that a brief discussion of smoking
cessation in the context of medical visits is effective.
Wood smoke is another significant irritant to the respiratory tract; in addition to chemical
irritants, wood smoke contains particulate matter. Assuming that the family's restaurant kitchen
meets current standards for air filtration/exhaust, the child's exposure should be minimal.
However, outdoor barbecue pits that discharge smoke directly into the environment should be
avoided.
Exposure to cooking spray/oils at the restaurant and cosmetic chemicals such as hairspray
at the salon also should be avoided, although the use of volatile organic compounds, including
chlorofluorocarbons in hairspray, permanent wave solution, cleaning solutions, and cooking
sprays, has decreased sharply since passage of the Clean Air Act and other Environmental
Protection Agency measures in the 1990s. Material Safety Data Sheets for most hairsprays, as
well as other cosmetics and cleaning fluids, reveal little inhalation toxicity, except in the case of
prolonged exposure under improper ventilation.
Although dust mite exposure may contribute to asthma exacerbation, there is no evidence
that environmental control can reduce the concentration of mites or exacerbations related to
them. Finally, serial respiratory virus infections themselves may contribute significantly to this
child's history of wheezing and cough.
The child in the vignette may benefit from therapeutic advances in asthma management
such as those contained in the National Heart, Lung and Blood Institute Guidelines for asthma,
including attention to number of symptomatic days, use of inhaled corticosteroids, and use of a
metered dose inhaler with spacer for proper delivery of medication, which may be more effective
than nebulization in older infants and children.
References:
Brunnhuber K, Cummings KM, Feit S, Sherman S, Woodcock J.Putting evidence into practice:
smoking cessation. BMJ Clinical Evidence. 2007. Available for subscription at:
http://clinicalevidence.bmj.com/ceweb/resources/index.jsp
Kum-Nji P, Meloy L, Herrod HG. Environmental tobacco smoke exposure: prevalence and
mechanisms of causation of infections in children. Pediatrics. 2006;117:5:1745-1754. Available
at: http://pediatrics.aappublications.org/cgi/content/full/117/5/1745
Roseby R, Waters E, Polnay A, Campbell R, Webster P, Spencer N. Family and carer smoking
control programmes for reducing children's exposure to environmental tobacco smoke.
Cochrane Database Syst Rev. 2003;3:CD001746. Available at:
http://www.cochrane.org/reviews/en/ab001746.html
Stein RT, Holberg CJ, Sherrill D, et al. Influence of parental smoking on respiratory symptoms
during the first decade of life: The Tucson Children's Respiratory Study. Am J Epidemiol.
1999;149:1030-1037. Available at: http://aje.oxfordjournals.org/cgi/reprint/149/11/1030
U.S. Environmental Protection Agency. Health effects of wood smoke. Available at:
http://www.epa.gov/woodstoves/healtheffects.html
U.S. Environmental Protection Agency. National volatile organic compound emission standards
for consumer products. Available at: http://www.epa.gov/fedrgstr/EPA-AIR/1998/September/Day-
page 42
11/a22660.htm
page 43
You are evaluating a newborn 6 hours after his birth. Labor and delivery were uncomplicated,
but amniocentesis performed during the pregnancy revealed trisomy 21. Fetal
echocardiography at 20 weeks gestation showed normal findings. The infant currently is
sleeping and is well-perfused, with a heart rate of 140 beats/min and no audible murmurs. His
physical features are consistent with Down syndrome.
Of the following, the MOST appropriate diagnostic study to perform is
A. barium swallow
B. cervical spine radiography
C. echocardiography
D. head ultrasonography
E. radiography of the abdomen
page 44
Preferred Response: C
Congenital heart defects (CHDs) are the most common of the congenital anomalies, occurring
with an incidence of approximately 5 to 8 per 1,000 live births (0.5% to 0.8%). The incidence of
CHD is greater in stillbirths and there is an increased incidence of CHD in those who have
aneuploidy. It is believed by some that the abnormal chromosomal composition rather than the
cardiac abnormality is responsible for the fetal demise of those who have CHDs. The strong
association between chromosomal abnormality and CHD has been shown in a number of
studies that focus on identification of fetal cardiac abnormality. A number of well-defined
chromosomal anomalies are associated with CHD (Item C21).
Early and accurate diagnosis of CHD is important in counseling parents of children in whom
a chromosomal abnormality or syndrome is suspected. Such diagnoses may have significant
effects on the health and well-being of the newborns because some CHDs may require ductal
patency for perfusion of either the systemic or pulmonary circulation. Therefore,
echocardiography now is considered an important component of the routine health supervision
of infants who have Down syndrome and other syndromes predisposing to structural heart
defects.
Although results of fetal echocardiography for the infant in the vignette were interpreted as
normal, not all CHDs can be diagnosed routinely with fetal echocardiography because of the
shunting pathways of the fetal circulation. For example, persistent patency of the ductus
arteriosus and secundum atrial septal defects are diagnosed postnatally. Small ventricular septal
defects frequently are not seen during fetal echocardiography because the pressure in the right
and left ventricles are equal due to the ductus arteriosus, resulting in minimal flow across the
defect prenatally.
Given the strong association between Down syndrome and CHD, echocardiography should
be performed in the newborn described in the vignette. Although children born with Down
syndrome have an increased incidence of duodenal atresia and other types of gastrointestinal
obstruction, a barium swallow is not an appropriate initial test for an asymptomatic newborn.
Cervical radiography, beginning at age 3 years, is important because of the risk of atlantoaxial
(C1-C2) subluxation. There is no indication for routine head ultrasonography or abdominal
radiography in an otherwise asymptomatic newborn who has Down syndrome.
References:
Committee on Genetics. Health supervision for children with Down syndrome. Pediatrics.
2001;107:442-449. Available at: http://pediatrics.aappublications.org/cgi/content/full/107/2/442
Silberbach M, Hannon D. Presentation of congenital heart disease in the neonate and young
infant. Pediatr Rev. 2007;28:123-131. Available at:
http://pedsinreview.aappublications.org/cgi/content/full/28/4/123
Tennstedt C, Chaoui R, Krner H, Dietel M. Spectrum of congenital heart defects and
extracardiac malformations associated with chromosomal abnormalities: results of a seven year
necropsy study. Heart. 1999;82:34-39. Abstract available at:
http://www.ncbi.nlm.nih.gov/pubmed/10377306
page 45
A 4-year-old boy presents with headache and difficulty walking. On physical examination, he is
afebrile, all growth parameters are within normal limits, and his mentation appears normal. The
optic discs are clearly visible and appear normal. He has normal eye position in primary gaze
but cannot abduct his right eye fully. He has normal tone, strength, and reflexes in his upper
limbs, but has bilateral hyperreflexia at the knees and ankle clonus. On gait examination, he toewalks.
Of the following, the MOST important next step is to obtain
page 46
Preferred Response: A
The boy described in the vignette presents with a very concerning constellation of symptoms
and signs requiring urgent evaluation of the central nervous system. The first step in the
diagnostic process is to localize the problem to the proper level of the nervous system: brain,
brainstem/cerebellum, spinal cord, nerve, junction, or muscle. Such a determination not only
allows for urgent appropriate diagnosis and treatment but also reduces unnecessary discomfort,
risks, and costs of inappropriate diagnostic testing. Consultation with a neurologist can be useful
before ordering testing. Headache, gait disturbance (toe-walking), and acquired ocular
misalignment (right eye cannot move to the right) localizes the problem to the central nervous
system and raises concerns for both hydrocephalus and a brainstem lesion.
Computed tomography (CT) scan of the head is the preferred neuroimaging technique for
this patient. Although brain magnetic resonance imaging (MRI) can document the
brainstem/posterior fossa better, it may not be readily available. Moreover, a head CT scan in
the emergency department is adequate to rule out hydrocephalus that requires emergent
neurosurgical consultation. Unfortunately, a common cause for hydrocephalus in a child of this
age is a brainstem or cerebellar neoplasm such as an astrocytoma, glioma, medulloblastoma, or
ependymoma. When these tumors enlarge in the posterior fossa, they can obstruct the flow of
cerebrospinal fluid and cause acute hydrocephalus, which is a neurosurgical emergency.
Primary headache disorders such as migraine do not often present before the age of 6
years and are not accompanied by the neurologic findings described in the vignette. Toe-walking
is a nonspecific finding that can occur in the context of relatively benign delayed development,
although if this represents a change for a child, an upper motor neuron (brain/spinal cord) lesion
must be ruled out. The ankle clonus described for the boy in the vignette mandates this
approach. Similarly, inability to abduct one eye can be congenital, but if this is a new finding, a
brain or brainstem lesion must be ruled out. In this case, headache and the oculomotor findings
make a brain or brainstem lesion likely.
Gait impairment and hyperreflexia only in the legs can result from hydrocephalus or can
localize to the mid- to lower spinal cord. A spinal cord lesion would not explain the headache, and
bilateral motor findings without sensory, bowel, or bladder involvement are uncommon for a
spinal cord lesion. Therefore, MRI of the spine is not needed. Similarly, the hyperreflexia is not
evidence of a nerve or muscle problem, obviating the need for electromyography and nerve
conduction studies. Lumbar puncture is therapeutic for headache due to pseudotumor cerebri,
which could present with this constellation of symptoms, but this degree of gait abnormality
would be uncommon in pseudotumor, and the presence of true hydrocephalus or an intracranial
mass must be excluded with a head CT or brain MRI prior to lumbar puncture. Visual evoked
potentials occasionally are used to determine whether the axons from eye to occipital cortex
function normally, but such a test generally is performed when demyelinating disorders are
suspected.
References:
Avellino AM. Hydrocephalus. In: Singer HS, Kossoff EH, Hartman AL, Crawford TO, eds.
Treatment of Pediatric Neurologic Disorders. Boca Raton, Fla: Taylor & Francis; 2005:25-36
Garton HJ, Piatt JH Jr. Hydrocephalus. Pediatr Clin North Am. 2004;51:305-325. Abstract
available at: http://www.ncbi.nlm.nih.gov/pubmed/15062673
Kestle JR. Pediatric hydrocephalus: current management. Neurol Clin. 2003;21:883-895.
Abstract available at: http://www.ncbi.nlm.nih.gov/pubmed/14743654
Kuttesch J Jr, Ater JL. Brain tumors in childhood. In: Behrman RE, Kliegman RM, Jenson HB,
Stanton BF, eds. Nelson Textbook of Pediatrics. 18th ed. Philadelphia, Pa: Saunders Elsevier;
2007:2128-2136
Piatt JH Jr. Recognizing neurosurgical conditions in the pediatrician's office. Pediatr Clin North
page 47
page 48
Parents who are new to your area bring in their 3-year-old daughter for evaluation because they
are concerned about her delayed speech. They say that she uses about 50 single words. The
girl has had tetralogy of Fallot repaired surgically and recurrent upper respiratory tract infections
with otitis media, for which tympanostomy tubes have been placed. Findings on physical
examination include microcephaly, underfolded pinnae, a broad nasal bridge, cleft uvula, and a
small chin. In addition, the childs speech has a hypernasal quality. The family history is
negative for birth defects and developmental delays.
Of the following, the contiguous gene deletion syndrome that BEST fits this childs features is
A. Angelman
B. Beckwith-Wiedemann
C. 4pD. Prader-Willi
E. 22q11
page 49
Preferred Response: E
page 50
Battaglia A, Carey JC, Wright TJ. Wolf-Hirschhorn syndrome. GeneReviews. 2006. Available at:
http://www.geneclinics.org/servlet/access?db=geneclinics&site=gt&id=8888891&key=OvKiicpzcf
vnc&gry=&fcn=y&fw=vH7o&filename=/profiles/whs/index.html
Bishara N, Clericuzio CL. Common dysmorphic syndromes in the NICU. NeoReviews.
2008;9:e29-e38. Available for subscription at:
http://neoreviews.aappublications.org/cgi/content/full/9/1/e29
Cassidy SB, Schwartz S. Prader-Willi syndrome. GeneReviews. 2006. Available at:
http://www.geneclinics.org/servlet/access?db=geneclinics&site=gt&id=8888891&key=OvKiicpzcf
vnc&gry=&fcn=y&fw=vWzr&filename=/profiles/pws/index.html
Lin RJ, Cherry AM, Bangs CD, Hoyme HE. FISHing for answers: the use of molecular
cytogenetic techniques in neonatology. NeoReviews. 2003;4:e94-e98. Available for subscription
at: http://neoreviews.aappublications.org/cgi/content/full/4/4/e94
McDonald-McGinn DM, Emanuel BS, Zackai EH. 22q11.2 deletion syndrome. GeneReviews.
2005. Available at:
http://www.ncbi.nlm.nih.gov/bookshelf/br.fcgi?book=gene&part=gr_22q11deletion
Shprintzen RJ. Velo-cardio-facial syndrome. In: Cassidy SB, Allanson JE, eds. Management of
Genetic Syndromes. 2nd ed. Hoboken, NJ: Wiley-Liss; 2005:615-632
Shuman C, Amith AC, Weksberg R. Beckwith-Wiedemann syndrome. GeneReviews. 2005.
Available at:
http://www.geneclinics.org/servlet/access?db=geneclinics&site=gt&id=8888891&key=OvKiicpzcf
vnc&gry=&fcn=y&fw=0F2J&filename=/profiles/bws/index.html
Weksberg R, Shuman C. Beckwith-Wiedemann syndrome and hemihyperplasia. In: Cassidy SB,
Allanson JE, eds. Management of Genetic Syndromes. 2nd ed. Hoboken, NJ: Wiley-Liss;
2005:101-116
Williams CA. Angelman syndrome. In: Cassidy SB, Allanson JE, eds. Management of Genetic
Syndromes. 2nd ed. Hoboken, NJ: Wiley-Liss; 2005:53-62
Williams CA, Driscoll DJ. Angelman syndrome. GeneReviews. 2007. Available at:
http://www.geneclinics.org/servlet/access?db=geneclinics&site=gt&id=8888891&key=OvKiicpzcf
vnc&gry=&fcn=y&fw=tkPG&filename=/profiles/angelman/index.html
page 51
A 13-year-old girl presents with severe lower abdominal pain of 24 hours duration. She states
that the pain is sharp and constant and that she has had similar pain for several days
approximately monthly over the past 4 months. She has no vomiting or diarrhea with the pain,
but she is constipated frequently, having a bowel movement about every 3 to 4 days. She feels
that her jeans are getting tighter around the waist, although she remains active, playing soccer
daily. She has never had a menstrual period and denies ever being sexually active. On physical
examination, she is afebrile, her heart rate is 85 beats/min, and her blood pressure is 110/70 mm
Hg. Her weight is at the 60th percentile and her height at the 50th percentile for age. Her breasts
and genitalia are at Sexual Maturity Rating 5. Abdominal examination reveals a firm and tender
midline mass that is inferior to the umbilicus.
Of the following, the MOST likely diagnosis is
A. bladder obstruction
B. endometriosis
C. hematocolpos
D. megacolon
E. ovarian cyst
page 52
Preferred Response: C
The adolescent described in the vignette has a clinical history and physical examination findings
compatible with an imperforate hymen, which probably is the most common obstructive anomaly
of the female reproductive tract. An adolescent patient who has an imperforate hymen may be
asymptomatic or may have a history of cyclic abdominal pain that may occur for several years
before the diagnosis is made. A bluish, bulging hymen may be seen on genital inspection (Item
C24), and a distended vagina may be palpated on rectoabdominal or abdominal examination. If
the vagina becomes substantially enlarged with accumulated blood, the patient may experience
back pain, pain with defecation that can result in constipation, nausea and vomiting, or difficulty
in urinating.
Bladder outlet obstruction occurs rarely, and although it produces a suprapubic mass, it
does not cause cyclic abdominal pain. Megacolon also is unlikely and does not cause cyclic
pain, although colonic irritation may develop from the pressure produced by the mass. An
ovarian cyst typically causes a right- or left-sided (not midline) mass, and endometriosis is an
unlikely cause of a palpable mass, although it can cause cyclic and acyclic pain in adolescents.
References:
Adams Hillard PJ, Deitch HF. Gynecologic disorders. In: Osborn LM, DeWitt TG, First LR, Zenel
JA, eds. Pediatrics. Philadelphia, Pa: Elsevier Mosby;2005:1461-1471
Laufer MR, Goldstein DP, Hendren WH. Structural abnormalities of the female reproductive
tract. In: Emans SJH, Laufer MR, Goldstein DP, eds. Pediatric and Adolescent Gynecology. 5th
ed. Philadelphia, Pa: Lippincott, Williams & Wilkins; 2005:334-416
page 53
During teaching rounds, the pediatric ward resident reports on a 4-month-old circumcised male
infant who was admitted to the pediatric ward for fever that morning. The infant is now afebrile
and has had respiratory rates of 40 breaths/min while sleeping and greater than 60 breaths/min
when awake. The infant has a soft, flat fontanelle on physical examination and is not irritable.
The only diagnostic studies obtained on admission were a urinalysis and complete blood count,
the results of which were normal, except for a white blood cell count of 16.0x103/mcL
(16.0x109/L).
Of the following, the MOST appropriate next step is
page 54
Preferred Response: B
Respiratory rates vary across a relatively wide range in pediatric patients, depending on factors
such as age and activity status. Therefore, strict definitions of tachypnea and bradypnea are
difficult to determine and always must be considered in association with other factors such as
current clinical status and individual history. Because tachypnea is a sensitive indicator of lower
airway disease, patients who have elevated respiratory rates deserve a clinical evaluation in the
context of other associated symptoms.
The tachypnea, history of fever, and elevated white blood cell count described for the boy in
the vignette warrant chest radiography. Although lumbar puncture and a urine culture often are
indicated to evaluate infants who have fever, the elevated respiratory rate combined with
reassuring neurologic examination results and normal urinalysis make pneumonia a more likely
diagnosis. Administration of a normal saline bolus would not be expected to improve the
abnormal respiratory rate.
References:
Bloomfield D. In brief: tachypnea. Pediatr Rev. 2002;23:294-295. Available at:
http://pedsinreview.aappublications.org/cgi/content/full/23/8/294
Sectish TC, Prober CG. Pneumonia. In: Kliegman RM, Behrman RE, Jenson HB, Stanton BF,
eds. Nelson Textbook of Pediatrics. 18th ed. Philadelphia, Pa: Saunders Elsevier;2007:17951799
page 55
A 6-year-old boy who has severe vomiting and dehydration is admitted to the hospital. Initial
laboratory studies demonstrate a serum sodium concentration of 126.0 mEq/L (126.0 mmol/L),
potassium of 5.3 mEq/L (5.3 mmol/L), and pH of 7.26. After 24 hours of rehydration with 0.9%
saline, his serum sodium concentration is 129.0 mEq/L (129.0 mmol/L) and potassium is 4.9
mEq/L (4.9 mmol/L). On physical re-examination, you note that his knees, elbows, dorsal
fingers, and tongue are somewhat pigmented (Item Q26), and his skin is darker than that of
other family members.
Of the following, the MOST useful diagnostic laboratory study at this time is measurement of
serum
page 56
Preferred Response: B
Acute or chronic vomiting and dehydration associated with hyponatremia and elevated
potassium concentrations, as described for the boy in the vignette, suggest adrenocortical
insufficiency. Primary adrenal insufficiency is associated with skin pigmentation (Item C26)
because of ACTH overproduction. ACTH acts directly on the melanocortin receptors of skin to
activate melanin production. Normally, ACTH controls the amount of cortisol produced by the
adrenal cortex by stimulating adrenal steroidogenesis. Cortisol then feeds back both to the
pituitary and hypothalamus to inhibit pituitary ACTH release. Measurement of low serum cortisol
and markedly elevated serum ACTH concentrations at any time of the day usually confirms the
diagnosis of primary adrenal insufficiency. The presence of skin pigmentation in the boy
described in the vignette suggests that identifying an elevated ACTH value will be easy, but
because there is diurnal variation in ACTH and cortisol, with highest concentrations during the
early morning hours and lowest in the late afternoon and evening, children who have less severe
adrenal insufficiency should have these hormones assessed in the early morning. In addition, an
ACTH stimulation test might be necessary for diagnostic confirmation. In this test, synthetic
ACTH1-24 is administered intravenously, and the adrenal cortisol response is measured before
injection and at 1 hour postinjection. An adequate cortisol response at 1 hour rules out adrenal
insufficiency.
Although ADH concentrations might be elevated in primary adrenal insufficiency because of
loss of intravascular fluid volume, measurement of this hormone does not help in the diagnosis
of a child who has low serum sodium and somewhat elevated potassium values. Low serum
sodium is associated with inappropriate ADH release, but the potassium would not be elevated.
ACTH and melanocyte-stimulating hormone (a melanocortin) both are produced from enzymatic
cleavage of a larger molecule, proopiomelanocortin. Although melanocyte-stimulating hormone
might be overproduced to some extent in the presence of excess ACTH, elevated
concentrations of this hormone are not necessary for skin pigmentation with ACTH excess.
Dehydroepiandrosterone (DHEA) is a weak androgen precursor produced by the fetal
adrenal initially; production increases again in mid-childhood with the onset of adrenarche. DHEA
values rise slowly from age 4 to 6 years. Elevated values may be found with some adrenal
tumors and some relatively rare types of congenital adrenal hyperplasia, but these disorders are
associated with some degree of early puberty. DHEA-S, the sulfated product of DHEA, usually
has stable serum values without diurnal variation. Therefore, normal DHEA or DHEA-S values in
an adolescent or adult confirm the presence of a functioning adrenal gland. This test is less
useful in a young child because onset of adrenarche is variable. Cortisol is low in primary or
secondary adrenal insufficiency but tends toward low ranges in most people after the early
morning hours.
Insulin-like growth factor 1 is a marker for growth hormone sufficiency, and normal
concentrations vary with age and sex. Because this child does not seem to have a growth
problem, the result of this assay should be normal for age.
References:
Auchus RJ, Rainey WE. Adrenarche-physiology, biochemistry and human disease. Clin
Endocrinol. 2004;60:288-296. Available at: http://www.blackwellsynergy.com/doi/full/10.1046/j.1365-2265.2003.01858.x
Coco G, Dal Pra XC, Presotto F, et al. Estimated risk for developing autoimmune Addison's
disease in patients with adrenal cortex antibodies. J Clin Endocrinol Metab. 2006;91:1637-1645.
Available at: http://jcem.endojournals.org/cgi/content/full/91/5/1637
Donohoue PA. Diagnosis of adrenal insufficiency in children. UpToDate Online 15.3. 2008.
Available for subscription at:
http://www.uptodateonline.com/utd/content/topic.do?topicKey=pediendo/20697
Perry R, Kecha O, Paquette J, Huot C, van Vliet G, Deal C. Primary adrenal insufficiency in
Copyright 2009 by the American Academy of Pediatrics
page 57
children: twenty years experience at the Sainte-Justine Hospital, Montreal. J Clin Endocrinol
Metab. 2005;90:3243-3250. Available at: http://jcem.endojournals.org/cgi/content/full/90/6/3243
Wilson TA, Speiser P. Adrenal insufficiency. eMedicine Specialties, Pediatrics: General Medicine,
Endocrinology. 2007. Available at: http://www.emedicine.com/ped/TOPIC47.HTM
page 58
A mother of a 6-year-old boy in your practice is concerned that her son may have dyslexia. She
has brought a sample of his printing to the visit in which the boy wrote "ded" instead of "bed" and
"dad" instead of "bad." She wants your advice on what she should do to help her son learn how
to write properly.
Of the following, the MOST appropriate response is to
A. reassure the mother that letter reversal can be normal through 7 years of age
B. recommend a comprehensive psychoeducational evaluation for a learning disability
C. recommend neurologic evaluation
D. refer the child for an occupational therapy evaluation and services to improve his writing skills
E. refer the child for vision therapy
page 59
Preferred Response: A
Letter reversal in writing can be normal in children through 7 years of age. Dyslexia, a word
recognition defect, is a specific learning disability that is neurobiologically based. It is
characterized by problems with the ability to recognize words accurately and poor spelling and
decoding skills. Its prevalence is as high as 17.4% of the school-age population. Affected
children have problems attaching the correct labels or names to letters and words. They may
call a "b" a "d" or read "saw" as "was." Because the problem is linguistic, not visual, affected
children do not have problems copying letters.
Backward writing and letter reversal occur commonly in early development for all children
whether or not they have learning disabilities. All children should receive routine vision
screening, but a visual acuity problem would not be the cause of the letter reversal for the boy
described in the vignette. There is no scientific evidence that vision therapy (eye exercise) is
effective in the remediation of language-based learning disorders. Because letter reversal still
can be considered in the normal range of development at 6 years of age, psychoeducational
evaluation, neurologic evaluation, and occupational therapy are not indicated for this child.
References:
Committee on Children With Disabilities, American Academy of Pediatrics (AAP) and American
Academy of Ophthalmology (AAO), and American Association for Pediatric Ophthalmology and
Strabismus (APOS). Learning disabilities, dyslexia, and vision: a subject review. Pediatrics.
1998;102:1217-1219. Available at:
http://pediatrics.aappublications.org/cgi/content/full/102/5/1217
Fletcher JM, Lyon GR, Fuchs LS, Barnes MA. Reading disabilities: word recognition. In:
Learning Disabilities: From Identification to Intervention. New York, NY: The Guilford Press:
2007:85-163
Shaywitz SE, Shawitz BA. Dyslexia (specific reading disability). Pediatr Rev. 2003;24:147-153.
Available at: http://pedsinreview.aappublications.org/cgi/content/full/24/5/147
page 60
A 14-year-old girl presents to the emergency department with a 2-day history of fever and a
rash. The rash has been progressive, and now her mouth and eyes hurt. Upon further
questioning, she reports that she was started on an antibiotic 7 days ago for some complaints of
dysuria, but she does not remember its name. Physical examination reveals a moderately toxicappearing female whose temperature is 102.6F (39.2C), respiratory rate is 25 breaths/min,
heart rate is 105 beats/min, and blood pressure is 105/70 mm Hg. Her bulbar conjunctivae are
erythematous (Item Q28A), and she has some early bullous lesions developing in her mouth.
She has right upper quadrant tenderness and multiple target lesions (Item Q28B) on her chest,
abdomen, arm, back, upper thighs, buttocks, and face.
Of the following, the antimicrobial agent that is MOST likely to be associated with these clinical
findings is
A. amoxicillin
B. azithromycin
C. cefdinir
D. clindamycin
E. trimethoprim-sulfamethoxazole
page 61
Preferred Response: E
page 62
A 14-year-old girl presents for evaluation after 4 days of a temperature to 103.0F (39.5C),
nausea, abdominal cramping, and profuse bloody diarrhea. She reports that she has not
traveled anywhere, has no pets, and has had no ill contacts or unusual food exposures. One
week ago, she was diagnosed with a methicillin-sensitive Staphylococcus aureus chronic
osteomyelitis of her distal radius and has been receiving intravenous cefazolin therapy via a
peripherally inserted central catheter line. Physical examination reveals an uncomfortable
teenager who complains of severe abdominal pain and has a temperature of 102.8F (39.4C)
and moist mucous membranes. Her abdomen is diffusely tender, with voluntary guarding but no
rebound tenderness on palpation. Rectal examination demonstrates normal sphincter tone with
no fissures or other lesions. Laboratory findings include a peripheral white blood cell count of
15.0x103/mcL (15.0x109/L); hemoglobin of 13.0 g/dL (130.0 g/L); platelet count of
300.0x103/mcL (300.0x109/L); and a differential count of 65% neutrophils, 25% lymphocytes,
and 10% monocytes. Her stool appears watery and grossly bloody.
Of the following, the MOST appropriate treatment for this patients condition is
A. ceftriaxone
B. clindamycin
C. metronidazole
D. trimethoprim-sulfamethoxazole
E. vancomycin
page 63
Preferred Response: C
The patient described in the vignette has findings of antibiotic-associated Clostridium difficile
disease due to her cefazolin therapy. C difficile accounts for approximately 20% to 30% of
cases of antibiotic-associated diarrhea, 50% to 70% of antibiotic-associated colitis cases, and
more than 90% of antibiotic-associated pseudomembranous colitis cases. The incidence of
antibiotic-associated diarrhea and colitis varies greatly, depending on the antibiotic and its
spectrum of activity and pharmacokinetic properties. Almost all antibiotic classes have been
associated with disease, but clindamycin, the penicillins, and the cephalosporins are implicated
most commonly.
The pathogenesis of C difficile-mediated diarrhea and colitis includes: 1) disruption and
eradication of normal colonic organisms by antibacterial agents; 2) colonization with toxigenic C
difficile; and 3) production of toxin A or toxin B by C difficile, both of which mediate cytoskeletal
damage of target cells, resulting in mucosal injury and inflammation.
Infection with toxigenic C difficile causes a spectrum of disease, ranging from asymptomatic
carriage to a fulminant, relapsing, and occasionally fatal colitis. Signs and symptoms may
develop as early as the first day of therapy to as late as 10 weeks after therapy has ended, with
a typical range of 5 to 10 days after the initiation of antibiotic therapy. The gastrointestinal
manifestations of C difficile are variable, ranging from watery diarrhea to colitis with blood. Other
findings include fever (seen in 30% to 50% of patients), leukocytosis (mean peripheral leukocyte
count of more than 15.0 x 103/mcL [15.0 x 109/L), nausea, malaise, abdominal pain, and
cramping. Oral metronidazole and oral vancomycin are equally effective for the treatment of C
difficile-associated diarrhea and colitis, but metronidazole is considered the drug of choice
because of cost and concerns regarding the emergence of vancomycin-resistant enterococci.
Ceftriaxone, clindamycin, and trimethoprim-sulfamethoxazole are not effective in the treatment of
this infection.
References:
American Academy of Pediatrics. Clostridium difficile. In: Pickering LK, Baker CJ, Long SS,
McMillan JA, eds. Red Book: 2006 Report of the Committee on Infectious Diseases. 27th ed. Elk
Grove Village, Ill: American Academy of Pediatrics; 2006:261-263
Benson L, Song X, Campos J, Singh N. Changing epidemiology of Clostridium difficile-associated
disease in children. Infect Control Hosp Epidemiol. 2007;28:1233-1235. Abstract available at:
http://www.ncbi.nlm.nih.gov/pubmed/17926272
Klein EJ, Boster DR, Stapp JR, et al. Diarrhea etiology in a children's hospital emergency
department: a prospective cohort study. Clin Infect Dis. 2006;43:807-813. Abstract available at:
http://www.ncbi.nlm.nih.gov/pubmed/16941358
Thielman NM, Wilson KH. Antibiotic-associated colitis. In: Mandell GL, Bennett JE, Dolin R, eds.
Mandell, Douglas and Bennett's Principles and Practice of Infectious Diseases. 6th ed. New
York, NY: Elsevier Churchill Livingstone; 2005:1249-1263
page 64
A 14-year-old girl who has a history of insulin-dependent diabetes mellitus (IDDM) presents with
vomiting, increased urination, and decreased energy. Physical examination reveals Kussmaul
breathing and delayed capillary refill. Laboratory findings include:
Sodium, 136.0 mEq/L (136.0 mmol/L)
Potassium, 5.2 mEq/L (5.2 mmol/L)
Chloride, 100.0 mEq/L (100.0 mmol/L)
Bicarbonate, 10.0 mEq/L (10.0 mmol/L)
Blood urea nitrogen, 24.0 mg/dL (8.6 mmol/L)
Creatinine, 0.9 mg/dL (79.6 mcmol/L)
Glucose, 550.0 mg/dL (30.5 mmol/L)
The patient receives initial hydration with 20 mL/kg of normal saline.
Of the following, a TRUE statement regarding this patient is that the
page 65
Preferred Response: E
Sodium and potassium are cations that often are accompanied by chloride as an anion when
consumed in the diet. Within the body, these cations play very different roles. The cell
membrane is permeable to both sodium and potassium, but due to Na+-K+-ATPase, they are
contained primarily on opposite sides of the cell membrane, with sodium distributed almost
exclusively extracellularly and potassium contained intracellularly. Sodium is the principal
extracellular cation and acts as the primary osmole to maintain extracellular volume-including
plasma volume, which is important for tissue perfusion.
Sodium balance is regulated in the kidney. Excessive sodium intake results in reduced
sodium conservation within the nephron and increased excretion to keep balance with the
increased intake. One tissue in which sodium is contained is the bone, and growing children who
have normal kidney function require 2 to 3 mEq/kg per day of sodium for growth.
Potassium is important for maintaining the resting membrane potential of cells. The typical
intracellular concentration of potassium (140.0 mEq/L [140.0 mmol/L]) is in marked contrast to
the 3.5 to 5.5 mEq/L (3.5 to 5.5 mmol/L) of potassium in the extracellular compartment. The ratio
of extracellular-to-intracellular potassium determines the resting membrane potential and
subsequent action potentials in tissues such as skeletal/cardiac muscle and neuronal tissues.
Accordingly, hypokalemia and hyperkalemia can result in muscle paralysis or cardiac
dysrhythmias such as ventricular tachycardia, fibrillation, and ultimately cardiac arrest. The
growing child requires 1 to 2 mEq/kg per day of potassium for cellular growth and to avoid
potassium deficiency.
Certain conditions alter the measured extracellular potassium concentrations, often
providing misleading information about intracellular stores. In the setting of acidosis, potassium
shifts from intracellular to extracellular locations, increasing measured extracellular potassium
values. Conversely, alkalosis or correction of metabolic acidosis results in a lowering of
measured potassium values. Therapies that result in intracellular shifting of potassium include:
insulin, beta-2 agonists such as albuterol or epinephrine, and sodium bicarbonate (due to
quenching of acidosis). The patient described in the vignette has severe metabolic acidosis with
increased urine output, and the serum potassium value of 5.2 mEq/L (5.2 mmol/L) is misleading.
In fact, potassium should be added to the intravenous fluids because this girl's total body
potassium is depleted, which will become very apparent upon correction of the acidosis and of
the hyperglycemia (with insulin).
References:
Plotnick L. Insulin-dependent diabetes mellitus. Pediatr Rev. 1994;15:137-148. Available at:
http://pedsinreview.aappublications.org/cgi/reprint/15/4/137
Rose BD, Post TW. Potassium homeostasis. In: Clinical Physiology of Acid-base and Electrolyte
Disorders. 5th ed. New York, NY: McGraw-Hill Medical Publishing Division; 2001:372-375
Rose BD, Post TW. The total body water and the plasma sodium concentration. In: Clinical
Physiology of Acid-base and Electrolyte Disorders. 5th ed. New York, NY: McGraw-Hill Medical
Publishing Division; 2001:241-243
page 66
You have just assisted in the delivery of a 38-week gestational age male infant who was born via
cesarean section to a 25-year-old woman. As you are completing the infants initial physical
examination, the father mentions that he and his wife have allergic rhinitis and asthma. He asks
whether his son is at increased risk for allergies and how they can reduce the boys chance for
developing such allergic disorders.
Of the following, the MOST appropriate next step is to
A. explain that because both parents have asthma, breastfeeding will not reduce the risk of
eczema
B. explain that breastfeeding or formula choices do not matter now because the mother did not
restrict her diet during pregnancy
C. measure the cord blood immunoglobulin E concentration to help establish the newborns risk
for atopic disorders
D. recommend exclusive breastfeeding for 4 months with the addition of a hypoallergenic formula
if needed
E. start the newborn on a cow milk formula for the first month, then switch to strict breastfeeding
if he develops eczema
page 67
Preferred Response: D
The incidence of atopy (allergic rhinitis, asthma, eczema) has increased significantly over the
past few decades. The ability to intervene and either delay or prevent atopic disease in infants
born to atopic parents has been the subject of numerous studies. Application of these studies to
the population as a whole is difficult because the specific interventions and endpoints for each
study often differ. However, one aspect that is agreed on is that atopy risk for infants increases
significantly when both parents have a history of atopy (30% to 60%) compared with a history
for just one parent (20% to 40%) or neither parent (10% to 15%).
Prior to delivery, two prevention strategies have been studied: maternal diet restriction and
supplementation with probiotics. Currently, no evidence supports maternal dietary restriction to
common allergenic foods. Some studies have supported administration of probiotics (eg,
Lactobacillus rhamnosus) to the mother 2 to 4 weeks before delivery and to the infant for 6
months after birth. One study demonstrated a reduction in eczema at 2 years but no reduction in
asthma, immunoglobulin (Ig) E concentrations, or allergen sensitization. Further, the dose and
type of probiotic has differed in various investigations, making generalized recommendations
difficult.
Even if both parents have atopy, as described in the vignette, breastfeeding or formula
choices may affect atopy outcomes for the infant. In "high-risk" newborns (ie, both parents have
atopy or one parent and one sibling have atopy), the American Academy of Pediatrics
Committee on Nutrition recommends exclusive breastfeeding for at least 4 months, with
supplementation of a hypoallergenic formula if needed. Although it is difficult to compare studies
because the duration of breastfeeding and atopic outcome (ie, eczema, allergic rhinitis, asthma)
differ, breastfeeding for at least 3 months reduces the risk for eczema. The protective benefit
becomes more complex when controlling for the specific maternal atopic condition. For "highrisk" infants born to women who choose not to breastfeed, most studies and experts support
starting an extensively hydrolyzed formula. Starting a cow or soy milk formula, compared with
an extensively hydrolyzed formula, increases the risk for early eczema. Or note, interventions
resulting in decreased atopy early in life may not predict later atopic outcomes.
Cord IgE concentrations can be used to assess a newborn's risk for atopy, but its
measurement currently is not recommended as a routine screening tool. Furthermore, because
both parents in the vignette have a history of atopy, the child already is considered "high risk."
The ability to predict atopy based on cord IgE concentrations also depends on the cutoff value
used. In one study, 80% of newborns whose cord IgE concentrations were greater than 0.9
kU/L subsequently developed atopy by 5 years of age, but the specific IgE value did not
correlate with atopy severity.
References:
American Academy of Pediatrics Committee on Nutrition. Hypoallergenic infant formulas.
Pediatrics. 2000;106: 346-349. Available at:
http://pediatrics.aappublications.org/cgi/content/full/106/2/346
Greer FR, Sicherer SH, Burks AW, Committee on Nutrition and Section on Allergy and
Immunology. Effects of early nutritional interventions on the development of atopic disease in
infants and children: the role of maternal dietary restriction, breastfeeding, timing of introduction
of complementary foods and hydrolyzed formulas. Pediatrics. 2008;121:183-191. Available at:
http://pediatrics.aappublications.org/cgi/content/full/121/1/183
Mihrshahi S, Ampon R, Webb K, et al for the CAPS Team. The association between infant
feeding practices and subsequent atopy among children with a family history of asthma. Clin Exp
Allergy. 2007;37:671-679. Abstract available at: http://www.ncbi.nlm.nih.gov/pubmed/17456214
Prescott SL, Bjrkstn B. Probiotics for the prevention or treatment of allergic disease. J Allergy
Clin Immunol. 2007;120:255-262. Abstract available at:
http://www.ncbi.nlm.nih.gov/pubmed/17544096
page 68
Snijders BEP, Thijs C, Dagnelie PC, et al. Breast-feeding duration and infant atopic
manifestations, by maternal allergic status, in the first two years of life (KOALA study). J Pediatr.
2007;151:347-351. Abstract available at: http://www.ncbi.nlm.nih.gov/pubmed/17889066
page 69
A 2-year-old girl who has a 4-day history of varicella presents to the office with agitation. Her
mother reports that she treated the fever, rash, and pruritus with acetaminophen and
diphenhydramine regularly, which provided some relief. This morning her daughter seemed
more irritable, had a higher fever than yesterday, and "seemed delirious." On physical
examination, the agitated and inconsolable child has a temperature of 104.2F (40.1C), heart
rate of 160 beats/min, respiratory rate of 36 beats/min, and blood pressure of 135/87 mm Hg.
Her pupils are dilated and sluggishly reactive. Examination of the skin reveals numerous small,
crusted erosions without surrounding erythema. Neurologic examination demonstrates no focal
findings, and the patient is not ataxic.
Of the following, the MOST likely explanation for these symptoms is
A. diphenhydramine overdose
B. hypoglycemia
C. intracranial hemorrhage
D. Reye syndrome
E. varicella cerebellitis
page 70
Preferred Response: A
The patient described in the vignette is exhibiting symptoms of anticholinergic toxicity. The
classic mnemonic "hot as a hare, dry as a bone, blind as a bat, red as a beet, and mad as a
hatter" aptly describes a number of the commonly seen signs and symptoms, including
hyperpyrexia, dry skin, dilated pupils, flushing, and delirium. In addition, affected patients
typically exhibit tachycardia and hypertension. The clinical syndrome caused by anticholinergic
toxicity is due to competitive inhibition of acetylcholine binding to the postganglionic
parasympathetic muscarinic receptors by the offending agent. Many commonly used
medications have anticholinergic properties, including antihistamines, tricyclic antidepressants,
antispasmodics, and mydriatics. For this patient, diphenhydramine is the most likely culprit.
Treatment of anticholinergic poisoning is primarily supportive, with stabilization of vital
functions (the ABCs) of greatest importance, followed by appropriate decontamination (eg,
activated charcoal for ingestions, skin cleansing and removal of transdermal delivery devices for
dermal absorption). Agitation and seizures may be treated with benzodiazepines.
Physostigmine, a cholinergic agent that competitively inhibits acetylcholinesterase, may be more
effective for the treatment of severe agitation than benzodiazepines, but significant toxicity risks
make its use controversial.
Hypoglycemia can cause altered mental status but typically produces bradycardia,
hypotension, and coma, rather than agitation. Intracranial hemorrhage can be seen in patients
who have varicella-related thrombocytopenia, although affected patients frequently present with
hemorrhagic skin lesions (Item C32). In addition, patients who have intracranial hemorrhage
typically present with focal neurologic findings and bradycardia with hypertension, if intracranial
pressure is increased.
The classic presentation of Reye syndrome includes sudden onset of protracted vomiting
without fever in association with lethargy that progresses to delirium, seizures, stupor, and
coma. Reye syndrome typically occurs 5 to 7 days after the onset of a viral illness, following
apparent improvement in the child's initial symptoms. It results from mitochondrial dysfunction
leading to liver failure and cerebral edema. Ninety percent of cases have been reported to follow
an upper respiratory tract infection; varicella has been associated with 5% of cases.
Central nervous system complications of varicella are uncommon and may include
cerebellitis, transverse myelitis, peripheral neuritis, and optic neuritis. Cerebellitis is the most
common presentation of encephalitis and typically presents on the third to eighth day of illness
with ataxia. Meningoencephalitis presents with fever, meningeal signs, seizures, and altered
mental status.
References:
Burns JJ Jr. Toxicity, anticholinergic. eMedicine Specialties, Emergency Medicine, Toxicology.
2006. Available at: http://www.emedicine.com/EMERG/topic36.htm
Burns MJ, Linden CH, Graudins A, Brown RM, Fletcher KE. A comparison of physostigmine and
benzodiazepines for the treatment of anticholinergic poisoning. Ann Emerg Med. 2000;35:374381. Abstract available at: http://www.ncbi.nlm.nih.gov/pubmed/10736125
Carey RG, Balisteri WF. Mitochondrial hepatopathies. In: Kleigman RM, Behrman RE, Jenson
HB, Stanton BF, eds. Nelson Textbook of Pediatrics. 18th ed. Philadelphia, Pa: Saunders
Elsevier; 2007:1696-1697
Gershon AA, LaRussa P. Varicella-zoster virus infections. In: Gershon AA, Hotez PJ, Katz SL,
eds. Krugman's Infectious Diseases of Children. 11th ed. Philadelphia, Pa: Mosby; 2004:785-816
Su M, Goldman M. Anticholinergic poisoning. UpToDate Online 15.3. 2008. Available for
subscription at:
http://www.utdol.com/utd/content/topic.do?topicKey=ad_tox/13958&selectedTitle=1~376&source
=search_result
page 71
A 7-month-old child presents for a follow-up office visit after undergoing a Kasai procedure for
biliary atresia at 6 weeks of age. The mother states that the boy is irritable when his right arm is
moved. On physical examination, the infant is jaundiced. You detect tenderness in the anterior
radial head. Radiography of the affected region demonstrates metaphyseal fraying (Item Q33)
and a fracture.
Of the following, the MOST appropriate laboratory studies to obtain next are
page 72
Preferred Response: C
Chronic cholestasis due to biliary atresia results in decreased bile flow into the intestine. The
absence of intraluminal bile acids, in turn, causes decreased digestion of lipids, leading to fat
malabsorption. In addition, absorption of fat-soluble vitamins (A, D, E, and K) is impaired, which
may lead to clinical sequelae of fat-soluble vitamin deficiency. Finally, steatorrhea may impair
calcium absorption because intraluminal free fatty acids may bind calcium.
The clinical presentation of the patient in the vignette strongly suggests the presence of
rickets from vitamin D deficiency. Therefore, the most helpful initial laboratory testing is
determination of calcium, phosphorus, and 25-hydroxyvitamin D concentrations. The 25hydroxyvitamin D assay is the best measure of hepatic stores of vitamin D and is a better
marker of vitamin D status than either serum vitamin D or 1,25-dihydroxyvitamin D. Although
bone density testing, measurement of serum magnesium and parathyroid hormone, and
determination of the urinary calcium-to-creatinine ratio may provide useful additional information,
they will not help establish the diagnosis of vitamin D-deficient rickets.
Rickets is a potentially preventable complication of biliary atresia, but requires monitoring of
calcium, phosphorus, and 25-hydroxyvitamin D concentrations two to four times a year. Infants
who have biliary atresia routinely receive supplementation with approximately 8,000 IU of
ergocalciferol (vitamin D2) daily. This dose of vitamin D is approximately 20 times the
recommended dietary allowance for a healthy toddler. If rickets develops or the vitamin D
concentration cannot be maintained within the normal range, the patient should receive either
calcitriol (1,25-dihydroxyvitamin D3) or intramuscular vitamin D.
References:
Campbell KM, Bezerra JA. Biliary atresia. In: Walker WA, Goulet O, Kleinman RE, Sherman PM,
Shneider BL, Sanderson IR, eds. Pediatric Gastrointestinal Disease. 4th ed. Hamilton, Ontario,
Canada: BC Decker; 2004:1122-1138
Suchy FJ. Neonatal cholestasis. Pediatr Rev. 2004;25:388-396. Available at:
http://pedsinreview.aappublications.org/cgi/content/full/25/11/388
page 73
You are examining a 3.5-kg term infant 48 hours after his birth. Results of the physical
examination are normal, and you are considering discharging him from the hospital. He is being
fed formula from a bottle, and the nurses report intakes of 30 mL every 3 hours. He has wet at
least six diapers daily for the past 2 days, but he has not passed any meconium or expressed
any stool since birth.
Of the following, the MOST likely diagnosis is
A. ileal atresia
B. imperforate anus
C. meconium ileus
D. meconium plug syndrome
E. neonatal small left colon syndrome
page 74
Preferred Response: D
Ninety-five percent of term infants express meconium or pass a stool in the first 24 hours of
postnatal life. The infant described in the vignette is term and appropriately grown and has been
feeding and voiding well, but he has failed to pass meconium or any stool in the first 2 days of
postnatal life. Although he does not have any abdominal distention, emesis, or systemic illness,
the pediatric clinician should be concerned about potential bowel, particularly colonic,
obstruction.
The most likely explanation for the symptoms described for the infant in the vignette is
meconium plug syndrome, which typically is an isolated phenomenon that is not associated with
anatomic obstruction (eg, atresia). It occurs commonly in term and preterm infants and may be
associated with maternal magnesium sulfate treatment for pre-eclampsia/eclampsia. Meconium
plug obstruction generally is related to hypomotility. Clinically, there may be no abdominal
findings or a gradual increase in girth but no other signs of illness. Plain radiographs of the
abdomen generally provide nonspecific findings, but may show a paucity of gas in the
rectosigmoid. A contrast enema characteristically illuminates the plugs of meconium and
facilitates their evacuation. On occasion, a firm, paraffin-like formed plug may be expressed
spontaneously by affected infants during the second postnatal day. Although some infants who
have retained meconium may exhibit a small left colon on contrast enema, colonic motility
usually is normal upon evacuation of the meconium plug(s).
Although meconium plug syndrome is the most common cause of delayed passage of stool,
the clinician also should consider Hirschsprung disease, a congenital absence of ganglion cells.
A failure to pass meconium in the first 24 hours of postnatal life characterizes 95% of affected
infants. The area of affected bowel typically is in the rectosigmoid, where a transition zone may
be observed on contrast enema, although this finding is less common in neonates. If
Hirschsprung disease is considered, diagnostic rectal biopsy should be performed.
The neonatal small left colon syndrome is seen in infants of diabetic mothers and is
diagnosed using a contrast enema. The enema may be both diagnostic and therapeutic, as seen
in meconium plug syndrome. Gradual feeding and monitoring of the stooling pattern generally
results in resolution of the condition over the early weeks of postnatal life.
Imperforate anus occurs in about 1 in 4,000 to 5,000 births, and typically is apparent on
physical examination. In some cases, a fistulous tract may exist, and the expression of
meconium may occur anywhere along the perineal-scrotal-urethral line. Imperforate anus may
be an isolated finding or seen in conjunction with other anomalies such as vertebral
malformations, cardiac malformation, tracheoesophageal fistula/esophageal atresia, renal
anomalies, and limb malformation (VACTERL association).
Ileal atresia (proximal or distal) occurs as part of a spectrum of jejunal-ileal bowel atresia
that likely reflects a mesenteric vascular defect or interruption in development. Its absolute
frequency is not well reported, although it commonly is diagnosed prenatally (dilated bowel,
polyhydramnios) on obstetric ultrasonography. With a distal obstruction, the newborn may take
early feedings well, but becomes ill, with bile-stained emesis and abdominal distention, in the first
24 to 48 postnatal hours. Plain radiographic findings may include multiple stacked loops of airfilled bowel and air-fluid levels.
Meconium ileus is a condition of intestinal obstruction related to thickened, inspissated
mucus mixed with meconium that is characteristic of cystic fibrosis and is related to altered
chloride and water balance in mucus. The meconium may be beadlike, in small, dense pellets,
and even visible on prenatal obstetric ultrasonography. A microcolon may exist distal to the
small bowel obstruction. Affected infants may have visible and palpable loops of bowel on
examination, in addition to abdominal distention, bilious emesis, and failure to pass meconium in
the first 24 to 48 hours of postnatal life. Plain radiographs may reveal a soap-bubble appearance
characteristic of meconium stool. A contrast enema may reveal a microcolon and failure to see
contrast reflux past the ileocecal valve.
References:
Albanese CT, Sylvester KG. Pediatric surgery. In: Doherty GM, Way LW, eds. Current Surgical
page 75
Diagnosis and Treatment. 12th ed. New York, NY: The McGraw-Hill Companies, Inc; 2006:chap
45
Burge D, Drewett M. Meconium plug obstruction. Pediatr Surg Int. 2004;20:108-110. Abstract
available at: http://www.ncbi.nlm.nih.gov/pubmed/14760494
Casaccia G, Trucchi A, Spirydakis I, et al. Congenital intestinal anomalies, neonatal short bowel
syndrome, and prenatal/neonatal counseling. J Pediatr Surg. 2006;41:804-807. Abstract
available at: http://www.ncbi.nlm.nih.gov/pubmed/16567197
Hajivassiliou CA. Intestinal obstruction in neonatal/pediatric surgery. Semin Pediatr Surg.
2003;12:241-253. Abstract available at: http://www.ncbi.nlm.nih.gov/pubmed/14655163
Magnuson DK, Parry RL, Chwals WJ. Selected abdominal gastrointestinal anomalies. In: Martin
RJ, Fanaroff AA, Walsh MC, eds: Fanaroff and Martin's Neonatal-Perinatal Medicine. 8th ed.
Philadelphia, Pa: Mosby Elsevier; 2006:1381-1402
Nurko S. Motility of the colon and anorectum. NeoReviews. 2006;7:e34-e48. Available for
subscription at: http://neoreviews.aappublications.org/cgi/content/full/7/1/e34
Sutton TL. Index of suspicion in the nursery. NeoReviews. 2006;7:e269-e271. Available for
subscription at: http://neoreviews.aappublications.org/cgi/content/full/7/5/e269
Thilo EH, Rosenberg AA. The newborn infant. In: Hay WW Jr, Levin M, Sondheimer JM,
Deterding RR, eds. Current Pediatric Diagnosis & Treatment. 18th ed. New York, NY: The
McGraw-Hill Companies, Inc; 2007:chap 1
page 76
A 4-year-old boy who recently emigrated from Central America is brought to your clinic because
of 2 weeks of colicky abdominal pain that recently has worsened. His vital signs are normal, and
he is afebrile. Physical examination reveals mild diffuse tenderness, but there is no rebound or
guarding. After your examination, he has an episode of vomiting. Examination of the vomitus
reveals long, slim objects that resemble worms (Item Q35).
Of the following, the BEST treatment choice is
A. albendazole
B. iodoquinol
C. metronidazole
D. praziquantel
E. voriconazole
page 77
Preferred Response: A
Ascariasis is caused by infestation with the roundworm Ascaris lumbricoides. It occurs most
commonly in tropical regions and areas that have poor sanitation, although many cases also
occur in the United States each year. Adult worms (Item C35A) live in the small intestine and
produce eggs (Item C35B) that are excreted in the stool into the soil. Infestation occurs when
the eggs in the contaminated soil are ingested. Larvae pass from the small intestine into the
bloodstream, traveling to the liver and lungs. They migrate from the lung to the pharynx, where
they are swallowed. The worms mature in the small intestine, where they produce their eggs,
completing the cycle.
Many patients who have ascariasis are asymptomatic, but symptoms such as nonspecific
gastrointestinal complaints and abdominal pain can occur, as described for the boy in the
vignette. Intestinal obstruction and symptoms related to the migration of the larvae, such as
obstructive jaundice and peritonitis, also may be seen. Adult worms may pass from the rectum,
nose, or mouth if there is heavy worm burden. The diagnosis is made by seeing either ova on
microscopic stool examination or the adult worm itself, as described for the boy in the vignette.
Treatment of ascariasis that is not associated with intestinal obstruction consists of a single
dose of either albendazole or pyrantel pamoate or a 3-day course of mebendazole. Intestinal
obstruction may require treatment with piperazine citrate solution or, rarely, surgical intervention.
Iodoquinol is a luminal amebicide that is effective for the treatment of asymptomatic amebic
cyst excreters, and metronidazole is the drug of choice for patients who have symptomatic
amebiasis as well as giardiasis. Neither is appropriate for the treatment of ascariasis.
Praziquantel is useful for the treatment of liver fluke infestations, schistosomiasis, and tapeworm
infestations. Voriconazole is an intravenous or oral medication used in the treatment of fungal
infections.
References:
American Academy of Pediatrics. Amebiasis. In: Pickering LK, Baker CJ, Long SS, McMillan JA,
eds. Red Book: 2006 Report of the Committee on Infectious Diseases. 27th ed. Elk Grove
Village, Ill: American Academy of Pediatrics; 2006:204-208
American Academy of Pediatrics. Ascaris lumbricoides infections. In: Pickering LK, Baker CJ,
Long SS, McMillan JA, eds. Red Book: 2006 Report of the Committee on Infectious Diseases.
27th ed. Elk Grove Village, Ill: American Academy of Pediatrics; 2006:218-219
Dent AE, Kazura JW. Ascariasis (Ascaris lumbricoides). In: Kliegman RM, Behrman RE,
Jenson HB, Stanton BF, eds. Nelson Textbook of Pediatrics. 18th ed. Philadelphia, Pa:
Saunders Elsevier; 2007:1495
page 78
An 8-month-old boy who has Down syndrome and a large ventriculoseptal defect has had
recurrent otitis media and sinusitis during the respiratory virus season that required four
separate courses of antibiotics in 4 months. At todays visit, his mother states that his rhinitis
and otitis media symptoms have resolved, but she is concerned about a recurrent diaper rash
that is unresponsive to both barrier creams and repeated use of the nystatin cream prescribed
last month. Examination reveals white plaques (Item Q36A) on the buccal mucosa just inside the
lips and a diaper rash (Item Q36B).
Of the following, the BEST therapeutic option for this child is
A. oral fluconazole
B. oral griseofulvin
C. oral itraconazole
D. topical clotrimazole alone
E. topical clotrimazole combined with triamcinolone
page 79
Preferred Response: A
The child described in the vignette has recurrent Candida diaper dermatitis and thrush that is
probably due to his four courses of antibiotics. Candida infections are a common problem in
infants, toddlers, and children who have underlying medical conditions that require frequent
administration of antibiotics. The administration of antibiotics disrupts normal skin and mucous
membrane microbiologic flora. The boy described in the vignette has been prescribed topical
antifungal preparations without sustained improvement.
Prescribing alternative topical antifungal preparations such as clotrimazole for the diaper
rash might treat Candida sp that are resistant to nystatin, but it will not treat thrush. Once-daily
oral fluconazole has been proven superior to oral nystatin for resistant thrush and effective for
Candida diaper dermatitis. Recurrence rates are similar for both fluconazole and nystatin. Oral
itraconazole is another effective antifungal, but it may be associated with more gastrointestinal
adverse effects, including hepatotoxicity, and is dosed twice daily. Griseofulvin is not the drug of
choice for Candida infection, although it is useful in fungal hair and nail infections.
Combining antifungal agents with steroids may diminish inflammation temporarily, but
steroids also delay the resolution of fungal infection by decreasing T-cell response in the skin,
resulting in worsening of the dermatitis after a few days. Further, high systemic absorption of
steroids may occur in areas of thin and occluded skin, such as the diaper area.
References:
American Academy of Pediatrics. Candidiasis (moniliasis, thrush). In: Pickering LK, Baker CJ,
Long SS, McMillan JA, eds. Red Book: 2006 Report of the Committee on Infectious Diseases.
27th ed. Elk Grove Village, Ill: American Academy of Pediatrics; 2006:242-246
American Academy of Pediatrics. Drugs for invasive and other serious fungal infections in
children. In: Pickering LK, Baker CJ, Long SS, McMillan JA, eds. Red Book: 2006 Report of the
Committee on Infectious Diseases. 27th ed. Elk Grove Village, Ill: American Academy of
Pediatrics; 2006:780
American Academy of Pediatrics. Recommended doses of parenteral and oral antifungal drugs.
In: Pickering LK, Baker CJ, Long SS, McMillan JA, eds. Red Book: 2006 Report of the
Committee on Infectious Diseases. 27th ed. Elk Grove Village, Ill: American Academy of
Pediatrics; 2006:777-779
American Academy of Pediatrics. Topical drugs for superficial fungal infections. In: Pickering LK,
Baker CJ, Long SS, McMillan JA, eds. Red Book: 2006 Report of the Committee on Infectious
Diseases. 27th ed. Elk Grove Village, Ill: American Academy of Pediatrics; 2006:781-784
Goins RA, Ascher D, Waecker N, Arnold J, Moorefield E. Comparison of fluconazole and
nystatin oral suspensions for treatment of oral candidiasis in infants. Pediatr Infect Dis J.
2002;21:1165-1167. Abstract available at: http://www.ncbi.nlm.nih.gov/pubmed/12506950
Pankhurst CL. Antifungal treatment in immunocompetent or immunocompromised infants and
children. BMJ Clinical Evidence. 2007. Available for subscription at:
http://clinicalevidence.bmj.com/ceweb/conditions/orh/1304/1304_I4.jsp538REF22
Weisse ME, Aronoff SC. Candida. In: Kliegman RM, Behrman RE, Jenson HB, Stanton BF, eds.
Nelson's Textbook of Pediatrics. 18th ed. Philadelphia, Pa: Saunders Elsevier; 2007:1207-1310
page 80
You are called to the newborn nursery to evaluate a 2-hour-old male who was born at term. The
pregnancy was uncomplicated, but meconium staining was noted at delivery. The baby weighs
3.8 kg, is afebrile, and has a heart rate of 165 beats/min and a respiratory rate of 70
breaths/min. You note tachypnea and hyperpnea with clear breath sounds, no murmurs, and
strong distal pulses. His oxygen saturation in room air is 68%. You place a nonrebreather mask
to deliver an Fio2 of 1.0. After 5 minutes, the oxygen saturation is 72%.
Of the following, the BEST explanation for the findings of the hyperoxia test is
page 81
Preferred Response: E
Most commonly, the practitioner is alerted to hypoxemia in the newborn by the finding of a low
oxygen saturation value. Among the various causes of abnormal oxygenation in the newborn
are pulmonary pathologies, congenital cardiovascular malformations, persistent pulmonary
hypertension of the newborn, and disturbances of the hematologic and metabolic systems. Rightto-left shunting can be thought of as a diversion of desaturated blood away from the lungs and to
the systemic circulation. This can occur because blood does not perfuse the ventilated portions
of the lung (intrapulmonary right-to-left shunting). Intrapulmonary shunting resulting from
infection such as pneumonia, pneumothorax, retained fetal lung liquid, and pulmonary
prematurity is the most frequently encountered reason for desaturation in a newborn.
Conversely, abnormal oxygenation can result from the situations in which the desaturated blood
does not perfuse the pulmonary artery from the heart (intracardiac right-to-left shunting) or is
diverted from the pulmonary circuit through the ductus arteriosus (extracardiac right-to-left
shunting). Examples of these pathologies include pulmonary atresia, transposition of the great
arteries, tricuspid atresia, and pulmonary hypertension.
Whether the cause of the desaturation is intrapulmonary, intracardiac, or extracardiac rightto-left shunting, cyanosis (blue, maroon, or purple discoloration of the skin) is likely to be
present. Typically, clinicians discern cyanosis in patients who have oxygen saturations of less
than 85%, although it may be apparent to some when the saturation is 90% or less. If cyanosis
is suspected in the newborn, cyanotic heart disease must be considered. It is reasonable to
place the patient in a high-oxygen atmosphere (near FiO2 of 1.0) to determine if the high-dose
oxygen can overcome the shunting. If the degree of cyanosis improves and oxygen saturations
become normal, the problem likely is intrapulmonary shunting rather than cyanotic heart
disease. If hyperoxia does not lead to increased oxygen saturation and higher PaO2 (=150 torr),
cyanotic congenital heart disease should be considered and the infant should undergo further
cardiac evaluation.
The newborn described in the vignette has cyanosis but no murmurs. His tachypnea and
hyperpnea (deep breathing) represent the physiologic response to hypoxemia. His saturation
improves slightly with the delivery of high-dose oxygen. Transposition of the great arteries is the
best explanation for the infant's hypoxemia (Item C37). No amount of oxygen delivered to the
patient's alveoli can improve oxygenation of the pulmonary blood flow because the pulmonary
blood flow in transposition already is well saturated. The neonate remains desaturated until
oxygenated blood from the left atrium adequately crosses the atrial septum to be delivered to the
system through the aorta. In contrast, the oxygenation defect associated with meconium
aspiration syndrome, persistent pulmonary hypertension of the newborn, pneumonia, and
retained fetal lung liquid syndrome is improved with high-dose oxygen delivery to the alveoli.
References:
Driscoll D, Allen HD, Atkins DL, et al. Guidelines for evaluation and management of common
congenital cardiac problems in infants, children, and adolescents. A statement for healthcare
professionals from the Committee on Congenital Cardiac Defects of the Council on
Cardiovascular Disease in the Young, American Heart Association. Circulation. 1994;90:21802188. Available at: http://circ.ahajournals.org/cgi/reprint/90/4/2180
Ranjit MS. Common congenital cyanotic heart defects--diagnosis and management. J Indian
Med Assoc. 2003;101:71-72, 74. Abstract available at:
http://www.ncbi.nlm.nih.gov/pubmed/12841486
Silberbach M, Hannon D. Presentation of congenital heart disease in the neonate and young
infant. Pediatr Rev. 2007;28:123-131. Available at:
http://pedsinreview.aappublications.org/cgi/content/full/28/4/123
page 82
The mother of a 10-month-old child who has mild hypotonia brings him to the office after he has
an unprovoked seizure. On physical examination, you note several hypopigmented macules on
the trunk (Item Q38A). Magnetic resonance imaging of the brain reveals several thickened
areas of cerebral cortex (Item Q38B), with abnormal signal and abnormalities along the walls of
the lateral ventricles (Item Q38C).
Of the following, the MOST likely diagnosis is
A. incontinentia pigmenti
B. neurofibromatosis type 1
C. Sturge-Weber syndrome
D. tuberous sclerosis
E. von Hippel-Lindau syndrome
page 83
Preferred Response: D
Low tone and seizures are relatively common neurologic problems. Low tone is a nonspecific
finding that may be due to disease in the central or peripheral nervous system, but the
occurrence of a seizure suggests a central cerebral cause. Neuroimaging with brain magnetic
resonance imaging (MRI) generally is recommended for any infant who has a seizure because
congenital brain malformations are more common at this age. The combination of such cerebral
symptoms and pigmentary abnormalities of the skin are an indication to obtain brain MRI to
assess for a possible neurocutaneous disorder. The seizures, hypotonia, hypopigmented
macules, and MRI findings described for the child in the vignette are most suggestive of
tuberous sclerosis complex (TSC) (Item C38A).
Incontinentia pigmenti is characterized by spasticity rather than hypotonicity, and skin
findings include swirled hyperpigmentation following the line Blaschko (Item C38B). Skin findings
associated with neurofibromatosis type 1 include caf au lait macules (Item C38C), axillary
freckling, and iris Lisch nodules (Item C38D). Although Sturge-Weber syndrome can be
associated with seizures in the first postnatal year, the primary skin finding is a port wine stain
(Item C38E). Neither seizures nor skin abnormalities are seen with von Hippel-Lindau
syndrome.
Initial management of this patient centers on the chief complaint, which is the seizure. In
most cases, no treatment is recommended in a child after a first unprovoked seizure. However,
the recurrence risk is much greater in TSC.
Subsequently, diagnostic assessment should be directed toward confirming whether this
child has TSC. In many cases, this is a clinical diagnosis based on the characteristic findings of
skin examination and the cerebral complications. However, many other organs may be involved
at presentation or during the child's lifetime, including the eyes, kidneys, lungs, and heart. In the
future, this child is at risk for developmental learning difficulties, behavior problems that can
include features of autistic spectrum disorders, and malignancies. Given the autosomal
dominant inheritance, proper management involves assessment of the parents and genetic
counseling. Commercial genetic testing is available and is helpful in cases where clinical
features, particularly early in the disease, do not confirm the diagnosis. Genetic testing may
have false-negative results due to mosaicism, ie, some organs may be affected due to TSC1 or
TSC2 mutations that are not present in blood. Given the complexity of this diagnosis, many
experts recommend that children who have TSC be cared for in multidisciplinary specialty
clinics.
References:
Ferner RE. Neurofibromatosis 1 and neurofibromatosis 2: a twenty first century perspective.
Lancet Neurol. 2007;6:340-351. Abstract available at:
http://www.ncbi.nlm.nih.gov/pubmed/17362838
Haslam RHA. Neurocutaneous syndromes. In: Kliegman RM, Behrman RE, Jenson HB, Stanton
BF, eds. Nelson Textbook of Pediatrics. 18th ed. Philadelphia, Pa: Saunders Elsevier; 2007:24832488
Kandt RS. Tuberous sclerosis complex. In: Singer HS, Kossoff EH, Hartman AL, Crawford TO,
eds. Treatment of Pediatric Neurologic Disorders. Boca Raton, Fla: Taylor & Francis; 2005:553560
Northrup H, Au K-S. Tuberous sclerosis complex. GeneReviews. 2005. Available at:
http://www.ncbi.nlm.nih.gov/bookshelf/br.fcgi?book=gene&part=tuberous-sclerosis
page 84
You are called to the emergency department to evaluate a 5-month-old boy who has new-onset
seizures. On physical examination, you note that he is thin and has marked hepatomegaly. The
mother tells you that he has been irritable the past several mornings when he awakened from a
full nights sleep. This morning, she found him seizing in his crib and called 911. Laboratory tests
performed on specimens taken prior to starting intravenous fluids reveal hypoglycemia, lactic
acidosis, hyperuricemia, and hyperlipidemia. You suspect a diagnosis of glycogen storage
disease.
Of the following, the MOST appropriate long-term management of this disorder includes
page 85
Preferred Response: B
page 86
A 13-year-old girl comes to your office because her menstrual periods are irregular. She
attained menarche at 12 years of age and states that she has had only four menstrual periods
over the past year. The periods last for 5 to 7 days and require the use of four pads per day.
She has never been sexually active. She plays no sports, but she swims in the summer for fun.
On physical examination, her weight and height are at the 50th percentile for age. She has
minimal facial acne and no hirsutism or other skin lesions. Her breast and genital development is
at Sexual Maturity Rating 5.
Of the following, the MOST appropriate management strategy for this patient is to
page 87
Preferred Response: C
Young women of low gynecologic age (ie, a few years after the onset of menstruation) often
have anovulatory cycles due to immaturity of the hormonal feedback system of the
hypothalamic-pituitary-ovarian axis. During the adolescent years, when ovulation does not occur
with every cycle, both frequent and infrequent menstruation can result. By 2 years after
menarche, 55% to 82% of cycles are ovulatory, and by 5 years after menarche, 80% to 90%
are ovulatory, resulting in more regular menses. The girl described in the vignette has no signs
of androgen excess or evidence of weight loss that may be associated with an eating disorder
but does have scant irregular periods without cramps or heavy prolonged menses. Because
these findings suggest physiologic anovulatory cycles, no further laboratory studies, such as
measurement of follicle-stimulating hormone, luteinizing hormone, and prolactin, are necessary.
Her continued normal linear growth obviates the need for a bone age measurement. The
girl's normal growth coupled with no evidence of fatigue or skin dryness make thyroid disease
unlikely. Pelvic ultrasonography is not indicated in the absence of signs of androgen excess or
other symptoms suggestive of ovarian pathology. The possibility of pregnancy always should be
assessed in the presence of amenorrhea.
References:
Emans SJ. Amenorrhea in the adolescent. In: Emans SJH, Laufer MR, Goldstein DP, eds.
Pediatric and Adolescent Gynecology. 5th ed. Philadelphia, Pa: Lippincott, Williams & Wilkins;
2005:214-269
Ohlemeyer CL. Menstrual disorders. In Osborn LM, DeWitt TG, First LR, Zenel JA eds.
Pediatrics. Philadelphia, Pa: Elsevier Mosby; 2005:1455-1460
page 88
You are evaluating a 20-month-old boy who has a rectal temperature of 106F (41.1C) and a
history of coughing. His mother reports that the child has had a decrease in activity and eating
over the past 2 days. On physical examination, the boy appears moderately ill but is alert and
easily interacts with you. He occasionally grunts, has a heart rate of 140 beats/min, and has a
respiratory rate of 55 breaths/min. His neck is supple, he is circumcised, and he has no
evidence of otitis media.
Of the following, the BEST initial test in the evaluation of this child is
A. chest radiography
B. C-reactive protein measurement
C. erythrocyte sedimentation rate
D. lumbar puncture
E. urinalysis
page 89
Preferred Response: A
Physical examination, a detailed history, and direct observation can help the clinician recognize
up to 90% of children who have a serious illness. Carefully selected laboratory and diagnostic
tests can enhance the detection of invasive bacterial infections in children who have fever.
Infants younger than 3 months of age who present with a fever are at high risk of invasive
bacterial infection due to their still-developing immune system. Specific high-risk factors,
including a history of prematurity, evidence of an abnormally elevated or depressed white blood
cell count, a focal source of infection such as otitis media or soft-tissue infection, or abnormal
chest findings on radiography are associated with serious bacterial infection in almost 25% of
cases. Infants younger than 3 months of age who do not have such high-risk factors still have a
nearly 3% incidence of a serious bacterial infection. Infants between the ages of 3 months and 3
years remain at risk for serious bacterial infections, with reports of a 3% incidence of bacteremia
associated with a temperature of 102.2F (39.0C) and 7% with a temperature of 104.0F
(40.0C). A temperature higher than 105.8F (41.0C) frequently is associated with invasive
bacterial infection.
The child described in the vignette has both an elevated temperature and signs and
symptoms suggestive of a primary respiratory infection that make chest radiography the best
initial diagnostic test. A lumbar puncture can be deferred because of the patient's age and
appropriate interaction with the examiner. The patient's age, sex, and circumcised status make
a urinary tract infection unlikely; as a result, urinalysis also could be deferred pending results of
chest radiography. Positive urinalysis can be suggestive of a urinary tract infection, but a urine
culture remains the definitive test when a urinary tract infection is suspected. Elevated
erythrocyte sedimentation rates and C-reactive protein measurements are indicative of the
presence of acute inflammatory processes, but they are nonspecific findings. Their elevation in
young children who have fever may be useful in guiding the clinician toward additional studies in
selected children.
References:
Brook I. Unexplained fever in young children: how to manage severe bacterial infection. BMJ.
2003;327:1094-1097. Available at: http://www.bmj.com/cgi/content/full/327/7423/1094
McCarthy PL. Evaluation of the sick child in the office and clinic. In: Kliegman RM, Behrman RE,
Jenson HB, Stanton BF, eds. Nelson Textbook of Pediatrics. 18th ed. Philadelphia, Pa:
Saunders Elsevier; 2007:363-365
page 90
You are called to the emergency department to see an 8-year-old girl in whom congenital
adrenal hyperplasia was diagnosed at birth. She is being treated with oral mineralocorticoid daily
(9-alpha-fludrocortisone 0.1 mg) and hydrocortisone 5 mg orally every 8 hours. She is febrile
(temperature of 102.0F [38.9C]) and has vomited twice. According to her mother, other family
members recently recovered from a gastrointestinal illness that started with fever and vomiting.
Of the following, the MOST appropriate treatment for this child is to
page 91
Preferred Response: B
The girl described in the vignette has adrenal insufficiency and is vomiting and febrile. She needs
stress doses of glucocorticoids but cannot keep down orally administered agents. No parenteral
preparation of mineralocorticoid is available. In contrast to synthetic glucocorticoids,
hydrocortisone has about 1% of the mineralocorticoid effect of aldosterone (the most important
natural mineralocorticoid), and a large dose of hydrocortisone can act as a mineralocorticoid.
Accordingly, this child needs a rapid-acting parenteral hydrocortisone preparation.
Hydrocortisone hemisuccinate administered subcutaneously, unless she has poor perfusion,
which would necessitate intramuscular or intravenous administration, can stabilize her course.
The girl's usual oral medication is not sufficient for the increased need of stress. Increasing
the oral medication may not be sufficient because she probably will not be able to keep it down
or absorb it if she is vomiting. Mineralocorticoid need generally does not increase during stress.
If she is vomiting and receiving parenteral hydrocortisone, she might require intravenous 0.9%
saline with glucose to maintain her sodium stores and her blood glucose in the normal range.
For children who have usual febrile illnesses or require surgery, administration of
glucocorticoid in doses that supply three to six times the usual cortisol secretion rate of 3 to 7
mg/M2 per day is reasonable treatment. However, children who have adrenal insufficiency and
develop varicella because of failure or lack of immunization must be managed very carefully.
There are no controlled studies, but some anecdotal data suggest that infection with varicella
virus can become devastating in glucocorticoid-treated individuals. For such children, careful
titration of hydrocortisone dose to only twice the usual need may provide the small amount of
needed additional glucocorticoid without compromising the children's immune responses.
References:
Donohoue PA. Treatment of adrenal insufficiency in children. UptoDate Online 15.3. 2008.
Available for subscription at:
http://www.uptodateonline.com/utd/content/topic.do?topicKey=pediendo/19876
Shulman DI, Palmert MR, Kemp SF, for the Lawson Wilkins Drug and Therapeutics Committee.
Adrenal insufficiency: still a cause of morbidity and death in childhood. Pediatrics. 2007;119:e484e494. Available at: http://pediatrics.aappublications.org/cgi/content/full/119/2/e484
Wilson TA, Speiser P. Adrenal insufficiency. eMedicine Specialties, Pediatrics: General Medicine,
Endocrinology. 2007. Available at:
http://www.emedicine.com/ped/TOPIC47.HTM
page 92
A 9-year-old girl has been evaluated by a learning consultant and found to have a slow reading
rate, weakness in short-term memory, and problems with reading comprehension. Her parents
ask you what subjects other than reading will be most challenging for her due to these learning
difficulties.
Of the following, the subject that this child should find MOST challenging is
A. art
B. creative writing
C. mathematics
D. music
E. social studies
page 93
Preferred Response: E
A student who has a slow reading rate, reduced reading comprehension, and impaired shortterm memory, such as the girl described in the vignette, will encounter problems in "content"
classes, which include subjects such as science, history, and social studies. When children
read texts in these subjects, they need to read factual information and use the material in the
text to obtain knowledge about the subject. This requires comprehension of the text, which
involves identifying and understanding the words.
To determine the meaning of a word, a reader first must decode and identify the word on the
page. A slow reader takes much longer to complete assignments and test questions. Further,
individuals who have weakness in understanding and remembering the text will have much more
difficulty with homework assignments and on examinations. The girl in the vignette will not have
the same difficulty with art, music, creative writing, or mathematics because these subjects do
not demand the same emphasis on reading a text and recalling facts.
References:
Fletcher JM, Lyon GR, Fuchs LS, Barnes MA. Reading disabilities: comprehension. In: Learning
Disabilities: From Identification to Intervention. New York, NY: The Guilford Press; 2007:184-206
Shaywitz SE, Shaywitz BA. Dyslexia (specific reading disability). Pediatr Rev. 2003;24:147-153.
Available at: http://pedsinreview.aappublications.org/cgi/content/full/24/5/147
page 94
As you are leaving the supermarket, the cashier tells you that she is worried because her child
recently had a positive tuberculin skin test. She had to take him to the health department for skin
testing because he had been in contact with her father, who recently was diagnosed with active
pulmonary tuberculosis. They told her that the boys skin test was positive at "25," but his chest
radiograph was normal. She is concerned because the doctor told her that the case is a little
unusual because of the type of tuberculosis her father has. She asked the physician at the
health department to write it down, and she hands you a piece of paper that says "INH
resistant." The mother asks you what type of medication her boy should receive.
Of the following, the MOST appropriate antituberculous agent to prescribe for this boy is
A. ciprofloxacin
B. ethambutol
C. isoniazid
D. pyrazinamide
E. rifampin
page 95
Preferred Response: E
With a positive tuberculin skin test, a negative chest radiograph, and no indications of active
disease, the patient described in the vignette meets the classification of a latent tuberculosis
infection (LTBI). LTBI usually is treated with isoniazid (INH) once daily for 9 months, but when
the source case is known to have INH resistance, this agent should not be used. Instead, a 6month course of rifampin is recommended. An exception to this approach would be if the source
case was known to be resistant to both INH and rifampin, in which case a tuberculosis expert
should be consulted to determine the best course of treatment. Ciprofloxacin, ethambutol, and
pyrazinamide are used in combination with other antituberculous agents for the treatment of
active tuberculous disease; they are not indicated for monotherapy in treating LTBI.
Although usually very well tolerated, patients who receive rifampin should be aware of
possible adverse effects. Urine, tears, and saliva change to a reddish-orange color, which may
stain clothes or contact lenses. Rifampin therapy also may be associated with mild "flulike"
symptoms (eg, myalgias) that resolve with continued therapy. Rifampin also induces
cytochrome P-450 activity and, therefore, decreases the half-life of medications such as
warfarin, digoxin, thyroxine, oral contraceptives, and some antimicrobial agents (eg,
chloramphenicol), making them less effective. When rifampin is used in combination with INH,
patients are twice as likely to develop hepatitis as are patients treated with rifampin in
combination with other antituberculous medications. Thrombocytopenia and leukopenia both
have been associated with rifampin therapy exceeding 1 month's duration.
References:
Alsayyed B, Adam HM. In brief: rifampin. Pediatr Rev. 2004;25:216-217. Available at:
http://pedsinreview.aappublications.org/cgi/content/full/25/6/216
American Academy of Pediatrics. Tuberculosis. In: Pickering LK, Baker CJ, Long SS, McMillan
JA, eds. Red Book: 2006 Report of the Committee on Infectious Diseases. 27th ed. Elk Grove
Village, Ill: American Academy of Pediatrics; 2006:678-698
Bliziotis IA, Ntziora F, Lawrence KR, Falagas ME. Rifampin as adjuvant treatment of Grampositive bacterial infections: a systemic review of comparative clinical trials. Eur J Clin Microbiol
Infect Dis. 2007;26:849-856. Abstract available at:
http://www.ncbi.nlm.nih.gov/pubmed/17712583
page 96
You are speaking to a group of medical students about different antibiotic classes that can be
used in the treatment of meningitis. One student asks you about chloramphenicol, a drug with
which he is not familiar.
Of the following, the MOST common adverse effect associated with chloramphenicol therapy is
page 97
Preferred Response: A
Chloramphenicol is an antibiotic that inhibits protein synthesis by reversibly binding to the 50S
subunit of the 70S ribosome. Against most sensitive organisms, chloramphenicol produces a
static effect by blocking protein synthesis. However, it is bactericidal against some meningeal
pathogens, such as Haemophilus influenzae, Streptococcus pneumoniae, and Neisseria
meningitidis. Chloramphenicol is not bactericidal against group B streptococci or enteric gramnegative bacilli because drug concentrations in the cerebrospinal fluid sufficient for effective
eradication of these organisms cannot be achieved at therapeutic doses. With the possible
exception of typhoid fever in some developing areas of the world where cost and availability
make it the primary therapy, chloramphenicol no longer is the drug of choice for any specific
infection.
Chloramphenicol is extremely active against a variety of organisms, including bacteria,
spirochetes, rickettsiae, chlamydiae, and mycoplasmas. Most gram-positive and gram-negative
aerobic bacteria are inhibited by concentrations achievable in the serum, and chloramphenicol is
one of the most effective antibiotics against anaerobic bacteria, including Bacteroides strains.
Salmonellae, including S typhi, generally are susceptible, although chloramphenicol-resistant
strains have emerged rapidly since the late 1980s in India, Korea, Vietnam, Peru, Mexico, and
Thailand.
Plasma concentrations of chloramphenicol vary, depending on the time and route of
administration. Oral chloramphenicol is well absorbed from the gastrointestinal tract and
produces peak serum concentrations of 12 mcg/mL of active antibiotic after a 1-g dose. The
intravenous preparation of the drug produces active chloramphenicol concentrations in the
serum that are 70% of those obtained after an oral dose. Intramuscular administration is well
tolerated and produces peak serum values that are similar to those seen with intravenous
administration. However, for patients who have enteric fever, peak serum concentrations of the
drug after intramuscular administration are only one half to two thirds of those obtained by
intravenous administration due to delayed drug absorption from the injection site.
Metabolism and excretion of chloramphenicol vary widely in the pediatric population and are
age- and patient-dependent. Therefore, monitoring of serum concentrations is critical. Significant
drug-drug interactions occur between chloramphenicol and other agents. Chloramphenicol
prolongs the half-life of tolbutamide, chlorpropamide, phenytoin, cyclophosphamide, and warfarin
by inhibiting hepatic microsomal enzymes. Drugs such as phenytoin, rifampin, and phenobarbital
decrease the serum concentration and increase the total body clearance of chloramphenicol by
inducing hepatic microsomal enzymes. Chloramphenicol may delay the response of anemias to
iron, folic acid, and vitamin B12, and in vitro it antagonizes the bactericidal activity of the
penicillins, cephalosporins, and aminoglycoside antibiotics.
The most common adverse effect associated with chloramphenicol is reversible bone
marrow depression that is characterized by a combination of reticulocytopenia, anemia,
leucopenia, and thrombocytopenia. Other adverse effects include idiosyncratic aplastic anemia
that is rare but may be fatal; hemolytic anemia in patients who have the Mediterranean form of
glucose-6-phosphate dehydrogenase deficiency; gray baby syndrome in neonates, which is
characterized by abdominal distention, vomiting, flaccidity, cyanosis, circulatory collapse, and
death; and optic neuritis that may occur in patients receiving prolonged therapy. Hypersensitivity
reactions (eg, rashes, drug fevers, anaphylaxis) are rare.
References:
Kauffman RE, Miceti JN, Strebel L, Buckley JA, Done AK, Dajani AS. Pharmacokinetics of
chloramphenicol and chloramphenicol succinate in infants and children. J Pediatr. 1981;98:315320. Abstract available at: http://www.ncbi.nlm.nih.gov/pubmed/7463235
Myers B, Salvatore M. Tetracyclines and chloramphenicol. In: Mandell GL, Bennett JE, Dolin R,
eds. Mandell, Douglas and Bennett's Principles and Practice of Infectious Diseases. 6th ed. New
York, NY: Elsevier Churchill Livingstone, 2005:356-373
page 98
Pickering LK, Hoecker JL, Kramer WG, Kohl S, Cleary TG. Clinical pharmacology of two
chloramphenicol preparations in children: sodium succinate (IV) and palmitate (oral) esters. J
Pediatr. 1980;96:757-761. Abstract available at: http://www.ncbi.nlm.nih.gov/pubmed/6987361
Rahal JJ Jr, Simberkoff MS. Bactericidal and bacteriostatic action of chloramphenicol against
meningeal pathogens. Antimicrob Agents Chemother. 1979;16:13-18. Available at:
http://aac.asm.org/cgi/reprint/16/1/13?view=long&pmid=38742
Yunis AA. Chloramphenicol-induced bone marrow suppression. Semin Hematol. 1973;10:225234
page 99
A 2-year-old boy presents with fever and abdominal pain. Urinalysis reveals a specific gravity of
1.010, pH of 5.5, 2+ protein, no blood, and negative leukocyte esterase and nitrite tests.
Microscopy findings are negative.
Of the following, the MOST appropriate diagnostic test to assess the severity of proteinuria in
this child is
A. a random urine sample measurement for quantitative protein and creatinine concentrations
B. measurement of serum albumin concentration and correlation with urine protein concentration
measured by dipstick
page 100
Preferred Response: A
The 2+ protein measurement on a dilute urine sample (urine specific gravity <1.015) described
for the boy in the vignette is abnormal and requires quantitation of the proteinuria. Although
traditional teaching favored 24-hour urine testing for quantitative assessment of proteinuria in the
past, this technique no longer is necessary. Spot/random urine samples for proteinuria correlate
closely with 24-hour urine collections and are much easier to collect via a urine bag, especially in
the child who is not yet toilet trained. A normal spot urine protein-to-creatinine ratio is less than
0.2.
The urine dipstick is a good screening test for proteinuria, but because it is a qualitative test,
it is a suboptimal indicator of the severity of proteinuria. Similarly, sulfosalicylic acid treatment of
freshly voided urine is a qualitative, not quantitative test. The sulfosalicylic acid induces
precipitation of proteins. Urine microalbumin is an ultrasensitive technique (using a
radioimmunoassay, enzyme-linked immunoassay, or nephelometry) to detect albumin excretion
in the urine that falls below that measured on a urine dipstick. Assessment of microalbuminuria is
used commonly in patients who have diabetes as a screening test for renal disease before overt
proteinuria occurs. Microalbuminuria screens also have been used in those who have
hypertension. Although this technique is quantitative, it has no role when overt proteinuria is
present, as measured on a urine dipstick. Measurement of serum albumin should be deferred
until the severity of the proteinuria can be quantified.
Although 24-hour urine testing for proteinuria no longer is necessary, the test remains useful
for evaluating children for risk factors for nephrolithiasis/urolithiasis or to screen for
catecholamine metabolites in children who have suspected neural crest tumors (neuroblastoma
or pheochromocytoma).
References:
Abitbol C, Zilleruelo G, Freundlich M, Strauss J. Quantitation of proteinuria with urinary
protein/creatinine ratios and random testing with dipsticks in nephrotic children. J Pediatr.
1990;116:243-247. Abstract available at: http://www.ncbi.nlm.nih.gov/pubmed/2299494
Chahar OP, Bundella B, Chahar CK, Purohit M. Quantitation of proteinuria by use of single
random spot urine collection. J Indian Med Assoc. 1993;91:86-87. Abstract available at:
http://www.ncbi.nlm.nih.gov/pubmed/8409488
Gregianin LJ, McGill AC, Pinheiro CM, Brunetto AL. Vanilmandelic acid and homovanillic acid
levels in patients with neural crest tumor: 24-hour urine collection versus random sample.
Pediatr Hematol Oncol. 1997;14:259-265. Abstract available at:
http://www.ncbi.nlm.nih.gov/pubmed/9185210
Pontremoli R, Leoncini G, Ravera M, et al. Microalbuminuria, cardiovascular, and renal risk in
primary hypertension. J Am Soc Nephrol. 2002;13:S169-S172. Available at:
http://jasn.asnjournals.org/cgi/content/full/13/suppl_3/S169
page 101
A 12-month-old girl presents with a 3-month history of a pruritic rash that involves her cheeks,
neck, anterior trunk, and antecubital and popliteal areas. The rash improves after use of an overthe-counter topical steroid cream but still is present most days, and the infant often wakes up at
night scratching. On physical examination, you observe a raised erythematous rash that has
areas of lichenification (Item Q47).
Of the following, the MOST helpful intervention is to
page 102
Preferred Response: D
Some 30% to 40% of infants who have moderate-to-severe atopic dermatitis (AD), such as
described for the infant in the vignette, may have an underlying immunoglobulin (Ig) E-mediated
food allergy exacerbating the AD. For some infants, food ingestion may result in immediate
worsening of AD severity, although most infants do not demonstrate this immediate reaction.
Many foods have been implicated in AD, but five (milk, eggs, soy, wheat, and peanut) account
for 90% of the causative allergens.
Both allergy skin testing and measurement of serum IgE concentrations to these foods can
help to identify and eliminate likely triggers. Either a negative IgE blood test (<0.35 kU/L) or a
negative skin test for a specific food provides a high negative predictive value. On the other
hand, the positive predictive value for a skin or blood test may be only 50%.
Although the most commonly implicated foods often are eliminated from the diet, such an
approach does not improve symptoms in most (60% to 70%) children because they do not have
IgE-mediated AD. The unnecessary elimination of multiple foods can have an adverse effect on
nutrition, and food avoidance should be guided by the dietary history, eczema severity, and skin
or blood testing.
Frequently, children experience perioral rashes after drinking fruit juice. Such rashes
typically are nonpruritic, limited to the area of contact, and resolve within a few hours. The
mechanism of such rashes is unknown, but children generally outgrow such reactions by age 4
years. In cases involving more widespread cutaneous symptoms, such as described in the
vignette, elimination of fruit or acidic juices is unnecessary.
Parents often request testing for environmental allergies. House dust mites have been
implicated in some cases of AD, although they are less likely a cause for moderate-to-severe
atopic dermatitis than food allergies. Climate changes such as cold, dry air or hot, humid
weather can worsen AD, but specific seasonal allergens such as oak tree or ragweed are not
associated with eczema in infants.
A skin biopsy can provide insight into the pathophysiology of chronic rashes or lesions.
Generally, skin biopsies neither are advised nor provide insight into the causes of typical AD
manifestations in infants, but atypical presentations or lack of expected improvement with
appropriate therapy should prompt consideration of a dermatology referral.
References:
Burks W. Skin manifestations of food allergy. Pediatrics. 2003;111:1617-1624. Available at:
http://pediatrics.aappublications.org/cgi/content/full/111/6/S2/1617
Greer FR, Sicherer SH, Burks AW, Committee on Nutrition and Section on Allergy and
Immunology. Effects of early nutritional interventions on the development of atopic disease in
infants and children: the role of maternal dietary restriction, breastfeeding, timing of introduction
of complementary foods and hydrolyzed formulas. Pediatrics. 2008;121:183-191. Available at:
http://pediatrics.aappublications.org/cgi/content/full/121/1/183
Sampson HA, Leung DYM. Adverse reactions to foods. In: Kleigman RM, Behrman RE, Jenson
HB, Stanton BF, eds. Nelson Textbook of Pediatrics. 18th ed. Philadelphia, Pa: Saunders
Elsevier; 2007:986-989
page 103
The mother of a 2-year-old boy calls you because she found her son holding an open bottle of
liquid dishwasher detergent. He is crying, drooling profusely, and has vomited three times. In
answer to your questions, she reports that he is not sleepy and did not seem to get it in his eyes
or on his skin. On examination in your office, you note an ulcer on his lower lip and several
ulcers on his tongue.
Of the following, the most appropriate next step in the evaluation and management of this boy
is to
page 104
Preferred Response: D
Dishwasher detergent is a highly caustic alkali that can cause significant oropharyngeal, upper
airway, and esophageal injury if ingested. Alkaline agents cause deep liquefaction necrosis of
the affected tissues, with ulceration and perforation likely complications. Common household
products such as oven and drain cleaners, dishwasher detergents, and cleaning products that
contain ammonia pose a risk of caustic injury. Substances that have a pH greater than 11 are
likely to cause injury, even in small amounts.
Children who have ingested caustic or corrosive agents typically present with drooling,
dysphagia, or emesis, as reported for the boy in the vignette, although the presence and
severity of symptoms is not predictive of esophageal injury. The presence or absence of oral
lesions also is not predictive of esophageal injury.
The initial evaluation and treatment of patients who have ingested caustic substances
includes stabilization of vital functions, pain management, and other supportive measures.
Decontamination with activated charcoal is not indicated because it will not adsorb acids or
alkalis and is likely to impair endoscopic evaluation. Use of ipecac syrup or gastric lavage is
contraindicated because of the potential to cause further injury to the esophagus. In the
symptomatic patient, endoscopy should be undertaken within 6 to 24 hours of ingestion to
determine the presence of esophageal burns and the degree of severity.
Evaluation of the asymptomatic patient is controversial. Because the available data suggest
that esophageal burns may be found in as many as 45% of those who do not have oral lesions,
endoscopy is recommended in clear cases of ingestion. If the history is questionable and the
patient remains asymptomatic, with normal swallowing, after several hours of observation, some
authors have suggested that it is safe to forego endoscopy.
References:
Cordero B, Savage RR, Cheng TL. In brief: corrosive ingestions. Pediatr Rev. 2006;27:154-155.
Available at: http://pedsinreview.aappublications.org/cgi/content/full/27/4/154
Ferry GD. Caustic esophageal injury in children. UpToDate. 2008. Available for subscription at:
http://www.utdol.com/utd/content/topic.do?topicKey=pedigast/11441&view=print
Gaudreault P, Parent M, McGuigan MA, Chicoine L, Lovejoy FH Jr. Predictability of esophageal
injury from signs and symptoms: a study of caustic ingestion in 378 children. Pediatrics.
1983;71:767-770. Available at: http://pediatrics.aappublications.org/cgi/content/full/71/5/767
page 105
A 12-year-old boy has had cholestasis since infancy from Alagille syndrome. He has been lost
to medical follow-up for the last several years. He now presents to your office with pain in his
right upper thigh after a fall. His thigh is intensely tender, and ultrasonography demonstrates a
large hematoma in his quadriceps. The parents state that he has tended to bruise easily in the
past few months.
Of the following, the condition MOST likely to account for this patients symptoms is
page 106
Preferred Response: D
page 107
A term infant is delivered by emergency cesarean section following the acute onset of maternal
vaginal bleeding and profound fetal bradycardia on electronic fetal heart rate monitoring. The
Apgar scores are 1, 2, and 3 at 1, 5, and 10 minutes, respectively. Resuscitation includes
intubation and assisted ventilation, chest compressions, and intravenous epinephrine. The infant
is admitted to the neonatal intensive care unit and has seizures at 6 hours of age.
Of the following, a TRUE statement about infants who have seizures following perinatal asphyxia
is that most
A. develop epilepsy
B. develop microcephaly
C. do not have severe long-term neurodevelopmental delay
D. experience hearing loss
E. require multiple anticonvulsant medications
page 108
Preferred Response: C
The infant described in the vignette has concerning perinatal events, low Apgar scores through
10 minutes of postnatal age, the need for vigorous resuscitation, and subsequent seizure
activity 6 hours after birth. Seizures occurring in the first 24 hours after birth in newborns who
have perinatal asphyxia are considered evidence of hypoxic-ischemic encephalopathy.
Accompanied by profound acidemia, these seizures also may reflect significant cerebral cellular
injury or cell death.
The causes of neonatal seizure broadly include hypoxia-ischemia, which may be global (as
in perinatal asphyxia) or focal (as in arterial or venous thrombosis); hemorrhagic injury; brain
malformation; transient metabolic disorders (such as hypoglycemia or hypocalcemia); infectious
(viral, bacterial, or fungal); inborn errors of metabolism; or unknown (Item C50A). As
demonstrated in Item C50B, most term infants who have neonatal seizures do not manifest longterm neurodevelopmental sequelae. Further, even though motor abnormalities may be found on
examination in a slight majority (53%) of newborns having a seizure for any cause, few have
cerebral palsy.
However, when seizures occur in newborns due to asphyxia (generalized hypoxic-ischemic
injury), the prevalence of abnormal neurodevelopmental outcomes is more concerning. Although
severe impairment in neurodevelopmental outcomes may occur in fewer than 50% of these
infants, the presence of mild-to-moderate neurodevelopmental impairment in cognitive and motor
function as well as some (~33%) children having long-term epilepsy brings the level of abnormal
outcomes to approximately 50% (Item C50B). Early predictions of outcome for such children
may be determined by evaluating the worst early electroencephalography (EEG) finding, the
follow-up 1-week EEG, and the findings evident on cranial magnetic resonance imaging.
Following perinatal asphyxia and resultant seizures, most newborns can be treated with a
single anticonvulsant drug. Longstanding epilepsy and microcephaly occur in fewer than 50% of
patients. Hearing loss, though more common in asphyxiated infants who have neurologic injury
than in uncomplicated term infants, still occurs in fewer than 10% of infants.
References:
Hill A. Neonatal seizures. Pediatr Rev. 2000;21:117-121. Available at:
http://pedsinreview.aappublications.org/cgi/content/full/21/4/117
Ronen GM, Buckley D, Penney S, Streiner DL. Long-term prognosis in children with neonatal
seizures: a population-based study. Neurology. 2007;69:1816-1822. Abstract available at:
http://www.ncbi.nlm.nih.gov/pubmed/17984448
Tekgul H, Gauvreau K, Soul J, et al. The current etiologic profile and neurodevelopmental
outcome of seizures in term newborn infants. Pediatrics. 2006;117:1270-1280. Available at:
http://pediatrics.aappublications.org/cgi/content/full/117/4/1270
page 109
A 16-year-old girl who is new to your practice complains of a nearly constant headache for the
past year. She describes the pain as a band around her head that often is throbbing and is
worse during the middle of the day. She denies nausea or vomiting but reports occasional
fatigue. There is no family history of headaches. She has missed more than 20 days of school
this year because of the headache, and she is struggling to maintain a C average. She admits to
hating school and does not participate in extracurricular activities because she "doesnt like
anything." Findings on her physical examination, including complete neurologic and funduscopic
evaluation, are normal.
Of the following, the BEST next step in the management of this girls headaches is to
page 110
Preferred Response: E
Chronic headache is a common complaint in children and adolescents. A careful history and
complete neurologic examination are indicated in the evaluation of headache. The most
important initial consideration is to identify characteristics of the headache that suggest serious
diagnoses, such as brain tumors or other diseases causing increased intracranial pressure.
Such characteristics include worsening pain at night or immediately upon awakening,
association with vomiting, and worsening pain with coughing or straining. Papilledema or focal
neurologic findings may be found in patients who have increased intracranial pressure or the
examination results may be normal.
Migraine headaches are periodic, may be accompanied by an aura, and typically are
relieved by sleep. A family history of migraines usually can be elicited. The neurologic
examination typically yields normal results, although complicated migraines can be accompanied
by focal neurologic deficits such as hemiparesis, cranial nerve palsies, and visual disturbances.
Pain from stress-related or tension headache generally is diffuse and may be described as
"bandlike" or throbbing, as reported for the girl in the vignette. Pain usually occurs on most days,
and school absence is frequent. The neurologic examination yields normal results.
It is also very important for the clinician to obtain further information regarding any emotional,
social, or academic difficulties the patient may be experiencing. Such data can help to determine
both an underlying cause for the headache and the effect the headache is having on the child's
quality of life. Family stressors and depression are known causes of headache, and questions
regarding sleep patterns, anhedonia, school performance, and relationships with family and
friends can help to screen for these conditions. Studies have shown that children and
adolescents who have frequent or severe headaches have greater impairment in academic and
social functioning than those who do not have headaches. A thorough psychosocial evaluation
can aid in evaluating patients for depression and assessing the effect of headaches on their
daily functioning. Nonpharmacologic therapies, such as rest, relaxation techniques, and removal
of stress from the environment, can be effective once the stress is identified.
Advising the girl in the vignette to keep a diary for 2 months likely would prolong her
difficulties and not address the underlying cause of her headaches. Lumbar puncture and
computed tomography scan of the brain may play some role in the evaluation of an acute
headache, especially if meningitis or a mass lesion of the brain is suspected, but these
diagnoses are very unlikely in a patient who has had a headache for a year and normal results
on neurologic examination. Oral sumatriptan is used in the treatment of migraine, but this girl's
history points to depression or another psychological cause for her headache, and prescribing
medications without a psychosocial evaluation probably would not be effective.
References:
Forsyth R, Farrell K. Headache in childhood. Pediatr Rev. 1999;20:39-45. Available at:
http://pedsinreview.aappublications.org/cgi/content/full/20/2/39
Silver N. Headache (chronic, tension type). BMJ Clinical Evidence. 2007. Available for
subscription at: http://clinicalevidence.bmj.com/ceweb/conditions/nud/1205/1205_guidelines.jsp
Strine TW, Okoro CA, McGuire LC, Balluz LS. The associations among childhood headaches,
emotional and behavioral difficulties, and health care use. Pediatrics. 2006;117: 1728-1735.
Available at: http://pediatrics.aappublications.org/cgi/content/full/117/5/1728
page 111
When a 14-year-old girl had frequent complaints of shoulder pain made worse by pitching
softball a few months ago, you diagnosed overuse injury. Nonsteroidal anti-inflammatory drugs
and rest have provided some relief. She presents today with complaints of recurrent upper arm
pain that is unrelated to exercise and sometimes awakens her from sleep. Physical examination
reveals a slightly larger circumference of the left proximal humerus compared with the right.
There is minimal tenderness on palpation over the area, although the girl reports a constant
ache. She has full range of motion of the arm at the shoulder and elbow. You obtain a shoulder
radiograph (Item Q52).
Of the following, the MOST likely diagnosis is
A. acromioclavicular separation
B. acute osteomyelitis
C. chronic osteomyelitis
D. osteosarcoma
E. supracondylar fracture of the humerus
page 112
Preferred Response: D
The girl described in the vignette has radiographic findings and clinical history that are most
suggestive of osteogenic sarcoma of the humerus. The most common presenting symptom of
osteosarcoma is pain, particularly with activity, as described for the girl. The affected patient
may have a history of swelling, depending on the size of the lesion and its location. Patients or
their parents may complain of a "sprain," "arthritis," or so-called "growing pains." Symptoms may
be present for weeks, months, or occasionally longer before osteosarcoma is diagnosed. The
child often has a history of trauma, as in this patient, because trauma occurs frequently in
teenagers. Pathologic fractures are not particularly common with this tumor compared with
leukemia and other malignancies. Systemic symptoms, such as fever and night sweats, are
rare.
Osteosarcoma is the third most common cancer in adolescence, after lymphomas and brain
tumors. Peak age at diagnosis is 10 to 25 years. The incidence is 400 cases per year in the
United States (4.8 cases per 1 million persons younger than 20 years) and is slightly higher in
African Americans than in whites. An increased incidence during adolescence corresponds with
the growth spurt.
Osteosarcoma occurs in the long bones near metaphyseal growth plates (Item C52A). Most
are high-grade intramedullary osteosarcomas, with only about 5% being low-grade lesions. The
most common sites are the femur (42%, with 75% of femoral tumors occurring distally), tibia
(19%, with 80% in the proximal tibia), and humerus (10%, with 90% in the proximal humerus).
The overall 5-year survival rate for patients whose condition was diagnosed between 1974 and
1994 was 63% (59% for males, 70% for females). The current 5-year survival rate is estimated
to be 65%. The mainstay of therapy is excision of the lesion. Chemotherapy is required to treat
micrometastatic disease, which is present but not detectable in most patients at diagnosis.
Metastatic spread to the lungs only rarely results in respiratory symptoms; such symptoms
usually indicate extensive lung involvement. Metastasis to other sites is extremely rare.
The exact cause of osteosarcoma is unknown, but it is believed to be a tumor of
osteoprogenitor cells, which are multipotential, hormone-responsive stromal cells in the
periosteum and marrow that are capable of differentiating into many lineages, depending on their
environment. Among the known risk factors are rapid bone growth, exposure to radiation, and
potentially a genetic predisposition. Children who have a prior personal or family history of
retinoblastoma and those who have received radiation therapy for a previous malignancy are at
higher risk of developing osteosarcoma.
Initial evaluation of an adolescent or older child presenting with bone pain and swelling,
especially with a palpable mass, should include:
o Plain radiographs (two views) of the suspected lesions, although no single feature on
radiographs is diagnostic. Osteosarcomatous lesions can be purely osteolytic (about 30% of
patients), purely osteoblastic (about 45% of patients), or a mixture of both. Elevation of the
periosteum may appear as the characteristic Codman triangle. Extension of tumor through the
periosteum may result in a so-called "sunburst appearance" (about 60% of patients).
o Both magnetic resonance imaging of the primary lesion and computed tomography scan of
the chest are necessary to confirm the diagnosis and for staging purposes. Such scans
frequently are performed at the tertiary center using their protocols.
The differential diagnosis of bone pain and swelling includes stress fracture, hematoma,
bone cysts, and other bony tumors such as Ewing sarcoma.
Although acute osteomyelitis is common in children, it is relatively less common in
teenagers; approximately 50% of cases occur in preschool-age children. Infection frequently is
characterized by overlying erythema, warmth, and more acute systemic signs such as fever
and malaise. The white blood cell count is elevated in 50% of patients, and the erythrocyte
sedimentation rate or C-reactive protein value is increased. Plain radiographs may be read as
normal early in the disease course.
Chronic osteomyelitis also may cause pain. However, results of blood tests often are
normal. The radiograph for the patient in the vignette, however, shows a classic sunburst
pattern, which is not consistent with chronic osteomyelitis.
Acromioclavicular (AC) separation typically results in sudden pain and limited range of
page 113
motion. The diagnosis can be made by an anteroposterior radiograph, which can demonstrate
excessive separation of the AC joint (Item C52B). Supracondylar fracture of the humerus
usually is caused by falling onto the extremity (often outstretched) and may be associated with
acute pain, swelling, and deformity near the elbow (Item C52C).
References:
Arndt CAS. Neoplasms of bone. In: Kliegman RM, Behrman RE, Jenson HB, Stanton BF, eds.
Nelson's Textbook of Pediatrics. 18th ed. Philadelphia, Pa: Saunders Elsevier; 2007:2146-2150
Cripe TP. Osteosarcoma. eMedicine Specialties, Pediatrics, Oncology. 2006. Available at:
http://www.emedicine.com/ped/topic1684.htm
Gorlick R, Anderson P, Andrulis I, et al. Biology of childhood osteogenic sarcoma and potential
targets for therapeutic development. Clin Cancer Res. 2003;9:5442-5453. Available at:
http://clincancerres.aacrjournals.org/cgi/content/full/9/15/5442
Gurney JG, Swensen AR, Bulterys M. Malignant bone tumors. In: Ries LAG, Smith MA, Gurney
JG, et al, eds. Cancer Incidence and Survival Among Children and Adolescents: United States
SEER Program 1975-1995. Bethesda, Md: National Cancer Institute, SEER Program; 1999:88110. Available at: http://seer.cancer.gov/publications/childhood/bone.pdf
Kalyoussef S, Tolan RW Jr. Osteomyelitis. eMedicine Specialties, Pediatrics, Infectious
Diseases. 2006. Available at: http://www.emedicine.com/PED/topic1677.htm
Seade LE, Bryan, WJ, Bartz RL, Josey R. Acromioclavicular joint injury. eMedicine Specialties,
Sports Medicine, Shoulder. 2006. Available at: http://www.emedicine.com/sports/TOPIC3.HTM
page 114
A 7-month-old female has undergone the second stage of surgical palliation (Glenn operation)
for hypoplastic left heart syndrome. She was discharged from the hospital 1 week ago, and her
mother brings her to the office because of irritability that began this morning. On physical
examination, the infant is awake and irritable, with a heart rate of 150 beats/min and a respiratory
rate of 50 breaths/min. She has cyanosis of the face and mucosal surfaces and swelling of the
arms and head.
Of the following, the BEST explanation for this patients clinical presentation is
A. polycythemia
B. postpericardiotomy syndrome
C. protein-losing enteropathy
D. superior vena cava syndrome
E. thoracic duct injury
page 115
Preferred Response: D
The child described in the vignette has had surgery involving her superior vena cava, which has
been sewn by an end-to-side anastomosis to her right pulmonary artery (Glenn operation). She
is at risk for stenosis at the surgical site, thrombosis within the superior vena cava, and altered
hemodynamics if the pulmonary vascular resistance (and, thus, the pulmonary artery pressure)
increases, which raises the pressure against which the venous drainage must occur. Her
symptoms and physical findings are consistent with superior vena cava syndrome (Item C53),
and she should undergo an aggressive evaluation and rapid institution of treatment.
Obstruction of the systemic venous return may result from one of three primary causes:
extrinsic compression of either the superior or inferior vena cava, intrinsic obstruction of
systemic return, or abnormal hemodynamics with significantly elevated right atrial pressures.
The systemic veins are thin-walled vascular structures that typically drain at low pressure into
the superior and inferior vena cavae. Normally, these large veins drain into the right atrium at low
pressure. Flow in any system moves from high to low pressure, and the cardiovascular system
is no exception. The right atrium in the healthy individual typically has a pressure of less than 10
mm Hg, often less than 5 mm Hg. Mechanisms that facilitate venous drainage to the right atrium
include gravity for the vessels of the upper body and muscular contraction of the lower
extremities, which serves to "push blood up" the valved veins of the caudal portion of the body.
As long as the pressure in the right atrium ("downstream") remains lower than the pressures in
the veins, forward flow ensues. Any process that increases the pressure in the right atrium
raises the pressure needed to ensure forward drainage of the systemic veins. Similarly, any
obstruction of the superior or inferior vena cava raises the pressure "upstream" and may limit
normal venous drainage.
Extrinsic compression can result from a mediastinal mass or tumor that physically
compresses the vena cava, thereby raising the pressure needed to pass blood through the
narrowing and into the right atrium. Intrinsic obstructions can result from surgical anastomotic
sites, baffle stenosis, thrombosis (eg, from an indwelling catheter), or cardiac tumors that
physically obstruct blood return through the vena cava.
When blood return from the superior vena cava into the right atrium is obstructed
significantly, patients may demonstrate signs of superficial venous distention, venous
congestion, and facial and upper body edema, as described for the infant in the vignette. As the
venous pressure increases proximal to the obstruction, the venous drainage of the brain may
become engorged, leading to discomfort, irritability, and seizure and brain injury.
Polycythemia (elevated hemoglobin concentrations) might lead to sluggish blood flow
through the small capillaries, but would not cause the findings described in this child.
Postpericardiotomy syndrome, which can occur in children who have had cardiac surgery,
generally presents with fever and systemic symptoms. Protein-losing enteropathy is a serious
complication that can occur in patients who have increased pressure in the venous drainage of
the gut, including those who undergo single ventricle palliation. Such patients typically present
with diarrhea and edema of the entire body, not localized to the upper compartments, as in this
patient. Thoracic duct injury can occur in any patient undergoing cardiac surgery and often
leads to a chylothorax.
References:
Moore P. Obstructive lesions. In: Rudolph CD, Rudolph AM, eds. Rudolph's Pediatrics. 21st ed.
New York, NY: McGraw-Hill Medical Publishing Division; 2003:1800-1813
Silberbach M, Hannon D. Presentation of congenital heart disease in the neonate and young
infant. Pediatr Rev. 2007;28:123-131. Available at:
http://pedsinreview.aappublications.org/cgi/content/full/28/4/123
page 116
A. arteriovenous malformation
B. brain damage from a seizure
C. ependymoma
D. glioblastoma multiforme
E. herpes encephalitis
page 117
Preferred Response: A
The most common cause of hemorrhagic stroke in children is vascular malformation, of which
there are two types: arteriovenous malformations (AVMs) and cavernous malformations. These
can present in childhood with hemorrhage that leads to headache and seizures, as described for
the boy in the vignette. Severe, "worst-ever" headache; seizure; and mental status change are
indications for emergency evaluation and imaging of the central nervous system. The computed
tomography scan obtained for this boy shows a contrast-enhancing tortuous vascular mass
without surrounding edema or midline shift (Item C54A).
Seizures are not believed to cause brain damage in children, except in rare cases. A large
hemorrhage can cause seizures, but seizures cannot cause a large hemorrhage.
Ependymomas are one of the more common brain tumors in children. They emerge from the
ependymal lining of ventricles, do not hemorrhage, and typically occur in the posterior fossa of
young children (Item C54B). Therefore, they present more indolently, with headache and cranial
nerve findings. Glioblastoma multiforme, the highest grade astrocytoma, is uncommon in young
children. Although glioblastoma multiforme can hemorrhage, it does not have a tortuous vascular
appearance on imaging and has surrounding edema. Herpes encephalitis causes hemorrhagic
necrosis (Item C54C) and can present with seizures, but usually the patient is febrile and the
characteristic imaging appearance does not involve a tortuous vascular-appearing mass.
References:
Huang J, Gailloud PH, Tamargo RJ. Vascular malformations. In: Singer HS, Kossoff EH,
Hartman AL, Crawford TO, eds. Treatment of Pediatric Neurologic Disorders. Boca Raton, Fla:
Taylor & Francis; 2005:409-414
Klimo P Jr, Rao G, Brockmeyer D. Pediatric arteriovenous malformations: a 15-year experience
with an emphasis on residual and recurrent lesions. Childs Nerv Syst. 2007;23:31-37. Abstract
available at: http://www.ncbi.nlm.nih.gov/pubmed/17053936
Thai Q, Moriarty JL, Tamargo RJ. Central nervous system vascular malformations in pediatric
patients. In: Maria BL, ed. Current Management in Child Neurology. 3rd ed. Hamilton, Ontario,
Canada: BC Decker Inc; 2005:595-605
page 118
While examining a newborn, you note a persistent curve in the spine regardless of the babys
position. You order spine radiographs, which reveal multiple vertebral malformations and
segmentation defects (Item Q55).
Of the following, the MOST appropriate studies to guide further management are
page 119
Preferred Response: C
Congenital scoliosis, as described for the infant in the vignette, is caused by errors in vertebral
development during embryogenesis, resulting in abnormal segmentation or formation (Item C55).
It is associated with other malformations in more than 50% of affected individuals, most
commonly involving the heart, kidneys, and spinal cord. Most cases of congenital scoliosis are
not hereditary, although numerous syndromes include spinal malformations.
The child who has congenital scoliosis should receive serial posteroanterior and lateral
spine radiographs beginning, ideally, at birth. After 1 year of age, spine radiographs should be
taken with the child standing and the pelvis maintained in a level position. Computed tomography
scan with thin slices and three-dimensional reconstruction may be helpful, especially if
malformations are multiple or complex. Magnetic resonance imaging (MRI) with cartilage
sequences is recommended to determine growth potential. Significant progression of scoliosis
usually warrants surgery. Bracing typically is not helpful as a primary treatment modality but
may be recommended after surgery.
Every child who has congenital scoliosis and vertebral anomalies should be evaluated
clinically for associated malformations. Up to 20% of affected individuals have abnormalities of
the urinary tract, and up to 25% have cardiac anomalies. Therefore, children should receive
screening renal ultrasonography and echocardiography. Up to 40% of affected children may
have abnormalities of the spinal cord such as syrinx and tethered cord, warranting spinal MRI.
Chromosome analysis, ophthalmology evaluation, and head ultrasonography typically are not
helpful in directing management but must be considered on a case-by-case basis.
References:
Arlet V, Odent T, Aebi M. Congenital scoliosis. Eur Spine J. 2003;12:456-463. Abstract available
at: http://www.ncbi.nlm.nih.gov/pubmed/14618384
Ferguson RL. Medical and congenital comorbidities associated with spinal deformities in the
immature spine. J Bone Joint Surg Am. 2007;89:34-41
Hedequist D, Emans J. Congenital scoliosis. J Am Acad Orthop Surg. 2004;12:266-275.
Abstract available at: http://www.ncbi.nlm.nih.gov/pubmed/15473678
page 120
A community group asks you to speak at a forum on teenage pregnancy. The number of
pregnancies among young adolescents at the local middle school has increased this year, and
several community members want more information about adolescent pregnancy and its longterm effects.
Of the following, the MOST appropriate statement to include in your talk about pregnant and
parenting adolescents in the United States is that
page 121
Preferred Response: D
Although the proportion of teenagers reporting sexual activity has dropped and adolescent
pregnancy rates have declined in recent years, the United States still has the highest adolescent
birth rate among comparable developed countries. Most adolescent pregnancies occur among
older adolescents (ie, 18 and 19 year olds), and young women in this age group have a lower
risk of medical complications involving the mother or child than those younger than 17 years of
age. The adverse consequences of adolescent pregnancy are myriad, including school
interruption, persistent poverty, limited vocational opportunities, separation from the father of the
baby, divorce, and repeat pregnancy. Poverty is an important factor correlated with adolescent
pregnancy, with as many as 83% of adolescents who gave birth and 61% of adolescents who
had abortions coming from poor or low-income homes compared with 38% of nonpregnant
adolescent females. Adolescent fathers have been shown to have similar outcomes to
adolescent mothers, with a higher likelihood of having poor academic performance, higher
school drop-out rates, and decreased income potential compared with their peers who are not
fathers.
In an attempt to decrease adolescent child-bearing rates as well as unsafe sexual practices,
every pediatrician should integrate sexuality education into clinical practice with children from
early childhood through adolescence. Pediatricians also should consider participating in
development and implementation of sexuality education curricula for schools or public efforts.
References:
American Academy of Pediatrics Committee on Psychosocial Aspects of Child and Family
Health and Committee on Adolescence. Sexuality education for children and adolescents.
Pediatrics. 2001;108:498-502. Available at:
http://pediatrics.aappublications.org/cgi/content/full/108/2/498
Klein JD, and the AAP Committee on Adolescence. Adolescent pregnancy: current trends and
issues. Pediatrics. 2005;116:281-286. Available at:
http://pediatrics.aappublications.org/cgi/content/full/116/1/281
page 122
You are treating a 14-year-old boy in the pediatric intensive care unit who suffered a traumatic
brain injury in a motor vehicle crash earlier today and underwent surgery to drain a right-sided
epidural hematoma. He is currently receiving mechanical ventilation and is sedated. The nurse
calls you to the bedside because the intraventricular catheter is clotted and no intracranial
pressure waveform is seen on the monitor. On physical examination, you note that his right pupil
is dilated and unresponsive to light, which differs from findings on your examination immediately
after surgery.
Of the following, the MOST appropriate immediate next step is
A. administration of fentanyl
B. administration of mannitol
C. cerebral angiography
D. replacement of the intraventricular catheter
E. ophthalmology consultation
page 123
Preferred Response: B
Increased intracranial pressure can cause herniation of the cerebral contents, which is a
neurologic emergency. Transtentorial herniation results from downward or upward displacement
of the brain through the tentorium at the level of the incisura. Temporal lobe (uncal) herniations
are a subcategory of transtentorial herniations that are characterized by dilation of a unilateral
pupil due to compression of the oculomotor nerve, as described for the boy in the vignette (Item
C57). Uncal herniations usually occur with rapid expansion of the contents of the temporal lobe
fossa, such as seen with epidural hematomas, focal injury, or infection.
Findings for the boy in the vignette suggest rebleeding of the epidural hematoma, which
requires prompt action to reduce the associated swelling and reverse the herniation. Osmotic
agents such as mannitol or hypertonic saline should be administered while calling the
neurosurgeon and planning for emergent radiologic evaluation.
Pain or anxiety may produce bilateral, not unilateral, pupillary dilation. Therefore, treatment
with fentanyl would not address this patient's neurologic emergency. Ischemic stroke can
produce pupil dilation but is less likely than uncal herniation in this scenario. Cerebral
angiography to evaluate blood vessel characteristics usually is performed after stroke is
diagnosed by computed tomography scan or magnetic radiographic imaging. Replacement of
the clotted intraventricular catheter is indicated for overall management, but acute measures to
reverse the uncal herniation are of much higher priority. Ophthalmology consultation for
anisocoria is not indicated at this time because the most likely cause of the patient's signs and
symptoms is acute herniation.
References:
Avner JR. Altered states of consciousness. Pediatr Rev. 2006:27:331-338. Available at:
http://pedsinreview.aappublications.org/cgi/content/full/27/9/331
Frankel LR. Neurological emergencies and stabilization. In: Kliegman RM, Behrman RE, Jenson
HB, Stanton BF, eds. Nelson Textbook of Pediatrics. 18th ed. Philadelphia, Pa: Saunders
Elsevier; 2007:405-410
page 124
The mother of a 10-year-old boy, whom you have been following since he was 3 years old,
complains that he is always hungry and is gaining weight. The mother, who is overweight,
reports that the boy refuses to exercise, and she cannot control his diet. She just read an article
in a magazine about weight gain from Cushing syndrome and wonders if he could have this
condition.
Of the following, the growth chart shown in Item 58 that suggests Cushing syndrome is
A. Growth chart A
B. Growth chart B
C. Growth chart C
D. Growth chart D
E. Growth chart E
page 125
Preferred Response: C
Weight gain from exogenous obesity can be confused with Cushing syndrome, but
glucocorticoid excess, as seen in Cushing syndrome, almost always is associated with
attenuation of normal growth, as documented with Growth Chart C (Item C58A). The other
growth charts are more typical for exogenous obesity, with height either enhanced or
unchanged in the presence of weight gain. Other signs and symptoms of Cushing syndrome
include hypertension, violaceous skin striae (Item C58B), "buffalo hump" and muscle weakness
because of loss of muscle mass, centripetal obesity, cushingoid facies, easy bruisability,
hirsutism, failure of pubertal progression or amenorrhea in women, loss of libido in men,
headache, depression, and dysphoria. Comparison of school photographs from past years can
be a useful exercise, although the clinical diagnosis of pituitary Cushing syndrome (Cushing
disease) can be difficult.
Documentation of several elevated 24-hour urine free cortisol measurements as well as
elevated overnight dexamethasone-suppressed serum cortisol, evening salivary cortisol, or
midnight serum cortisol values aid in diagnosis. However, children who are very obese,
stressed, or depressed may have inappropriate elevations in serum or urine cortisol values, and
children who have mild Cushing disease may have normal values on one or more occasions.
References:
Batista DL, Riar J, Keil M, Stratakis CA. Diagnostic tests for children who are referred for the
investigation of Cushing syndrome. Pediatrics. 2007;120:e575-e586. Available at:
http://pediatrics.aappublications.org/cgi/content/full/120/3/e575
Greening JE, Storr HL, McKensie SA, et al. Linear growth and body mass index in pediatric
patients with Cushing's disease or simple obesity. J Endocrinol Invest. 2006;29:885-887.
Abstract available at: http://www.ncbi.nlm.nih.gov/pubmed/17185896
Klish WJ. Clinical evaluation of the obese child and adolescent. UpToDate Online 15.3. 2008.
Available for subscription at:
http://www.uptodateonline.com/utd/content/topic.do?topicKey=pedigast/11089
Magiakou MA, Mastorakas G, Oldfield EH, et al. Cushing's syndrome in children and
adolescents. Presentation, diagnosis, and therapy. N Engl J Med. 1994;331:629-636. Abstract
available at: http://www.ncbi.nlm.nih.gov/pubmed/8052272
Nieman L, Ilias I. Evaluation and treatment of Cushing's syndrome. Am J Med. 2005;118:13401346. Abstract available at: http://www.ncbi.nlm.nih.gov/pubmed/16378774
page 126
The parents of an 8-year-old boy are concerned because he recently has begun to struggle in
school. In the past, he always had been an attentive and motivated student. His current teacher
reports that at times when he is speaking in class, he stops speaking abruptly, stares with
glassy eyes, then resumes speaking. At home, his parents note that he "spaces out" when
eating dinner. His parents ask your input and the best approach to treat his issues.
Of the following, the MOST appropriate initial evaluation is
page 127
Preferred Response: C
The child described in the vignette appears to be experiencing absence seizures. These
present as multiple daily episodes of abrupt arrest of consciousness and voluntary movement
that last a few seconds (Item C59), followed by immediate return to normal consciousness.
Decline in school function is common at the onset of this disorder. Electroencephalography
(EEG) confirms the diagnosis, showing a pattern of spike and wave electrical complexes that
have a frequency of 3 Hz (3/second). Computed tomography scan and magnetic resonance
imaging are not indicated in the evaluation of children who have uncomplicated absence
seizures.
An audiologic evaluation would be indicated if the child was suspected of not hearing
properly. Decisions regarding an educational evaluation should be deferred until the diagnosis of
epilepsy is clarified. Occasionally, inattention associated with attention-deficit/hyperactivity
disorder (ADHD) may mimic absence epilepsy. However, children who have ADHD do not have
a sudden cessation of motor activity, blank facial expression, or flickering of the eyelids
characteristic of absence seizures. Computed tomography scan, magnetic resonance imaging,
and EEG generally are not indicated for the diagnosis of either ADHD or a learning disability.
Children who have learning disabilities are identified by results from standardized
psychoeducational testing. ADHD is diagnosed if the child meets American Psychiatric
Association Diagnostic and Statistical Manual of Mental Disorders, fourth edition, text revision
(DSM-IV-TR) criteria and has symptoms of inattention and hyperactivity/impulsivity that have
lasted for more than 6 months and were observed prior to age 7 years. The behaviors cause
"clinically significant impairment in social, academic or occupational functioning" and present in
more that one setting. Symptoms do not include the type of "staring spells" described for the boy
in the vignette.
References:
American Academy of Pediatrics. Committee on Quality Improvement, Subcommittee on
Attention-Deficit/Hyperactivity Disorder. Clinical practice guideline: diagnosis and evaluation of
the child with attention-deficit/hyperactivity disorder. Pediatrics. 2000;105:1158-1170. Available
at: http://pediatrics.aappublications.org/cgi/content/full/105/5/1158
American Psychiatric Association. Diagnostic criteria for ADHD. In: Diagnostic and Statistical
Manual of Mental Disorders. 4th ed. Text revision. Arlington, Va: American Psychiatric
Association; 2000:85-94
American Psychiatric Association. Diagnostic criteria for learning disability. In: Diagnostic and
Statistical Manual of Mental Disorders. 4th ed. Text revision. Arlington, Va: American Psychiatric
Association; 2000:49-56
Posner E. Absence seizures in children. BMJ Clinical Evidence. 2007. Available for subscription
at: http://clinicalevidence.bmj.com/ceweb/conditions/chd/0317/0317_background.jsp
Pritchard D. Attention deficit hyperactivity disorder in children. BMJ Clinical Evidence. 2006.
Available for subscription at:
http://clinicalevidence.bmj.com/ceweb/conditions/chd/0312/0312_background.jsp
page 128
You are seeing a young girl for a health supervision visit. Her older brother recently underwent a
bone marrow transplant, and you inquire about his health. The mother is tearful as she tells you
it has been difficult, explaining that he has had fever for about 10 days, his "counts are still
down," and they are planning to start amphotericin B just in case he has a fungal infection. She is
concerned because she was told about potential adverse effects of the medication and how they
need to watch the "electrolytes in his blood" very closely. She doesnt know what "electrolytes"
are, but asks what parameter in his blood might be affected.
Of the following, the MOST clinically important parameter to monitor during the initiation of
amphotericin B therapy is
A. bicarbonate
B. creatinine
C. glucose
D. potassium
E. sodium
page 129
Preferred Response: D
The major metabolic abnormality that occurs early in therapy with amphotericin B administration
is hypokalemia, which can be difficult to correct, especially if there is concurrent
hypomagnesemia. Amphotericin B augments potassium and magnesium excretion by the
kidney. Therefore, potassium concentrations cannot be corrected in most patients until the
hypomagnesemia also is corrected. The hypokalemia can occur whether amphotericin B
deoxycholate or the lipid formulations of amphotericin B are used. Problems with sodium,
glucose, and bicarbonate usually do not occur with amphotericin B therapy. Elevation of the
blood urea nitrogen or creatinine values associated with amphotericin B therapy occurs slowly
and is due to the cumulative renal effects of the medication. Once the medication is stopped,
renal function returns to normal in most patients.
Other common adverse reactions associated with amphotericin B therapy include fever,
chills, rigors, malaise, nausea, vomiting, and headaches. Younger children and infants generally
do not have these reactions. Older children and adolescents may require premedication with
ibuprofen or acetaminophen to lessen such reactions. In addition, lengthening of infusion times to
between 4 and 6 hours can blunt such symptoms. In severe cases, patients may require
antihistamines, antiemetics, meperidine, or the addition of hydrocortisone to the infusion before
administration of amphotericin B.
References:
American Academy of Pediatrics. Antifungal drugs for systemic fungal infections. In: Pickering
LK, Baker CJ, Long SS, McMillan JA, eds. Red Book: 2006 Report of the Committee on
Infectious Diseases. 27th ed. Elk Grove Village, Ill: American Academy of Pediatrics; 2006:774776
Zaoutis TE, Benjamin DK, Steinbach WJ. Antifungal treatment in pediatric patients. Drug Resist
Update. 2005;8:235-245. Abstract available at: http://www.ncbi.nlm.nih.gov/pubmed/16054422
page 130
You are evaluating an 8-year-old boy who has acute lymphoblastic leukemia and is in septic
shock caused by Klebsiella pneumoniae. The antibiotic susceptibilities for the organism reveal
that it is resistant to ampicillin, cefazolin, ceftriaxone, and gentamicin.
Of the following, the MOST appropriate antibiotic to use in this patient is
A. cefuroxime
B. clindamycin
C. meropenem
D. penicillin G
E. piperacillin
page 131
Preferred Response: C
Imipenem and meropenem are carbapenems, a type of beta-lactam antibiotic that has a very
broad antibacterial spectrum of activity. These agents are beta-lactamase-stable and bind with
high affinity to the penicillin-binding proteins of gram-positive and gram-negative bacteria.
Imipenem is slightly more active against gram-positive bacteria than meropenem; meropenem is
slightly more active against gram-negative aerobic organisms. Both have excellent activity
against aerobic hemolytic streptococci, Streptococcus pneumoniae (including resistant strains),
methicillin-susceptible S aureus and S epidermidis, Listeria sp, and Bacillus sp. Penicillinsusceptible strains of Enterococcus faecalis are susceptible to imipenem (bacteriostatic), but
resistant to meropenem. As a class, carbapenems also are highly active against Neisseria
gonorrhoeae and N meningitidis, Haemophilus influenzae and other Haemophilus sp, Morganella
sp, Proteus sp, most Enterobacteriaceae, Citrobacter sp, Enterobacter sp, Providencia sp,
Pseudomonas sp, and most anaerobic species, including anaerobic gram-positive cocci. They
also are active against Nocardia sp and Actinomyces sp.
Carbapenems are useful for the treatment of a wide variety of infections. Imipenem and
meropenem are most appropriate for treatment of infections caused by cephalosporin-resistant
Enterobacteriaceae, especially Serratia marcescens, Providencia sp, C freundii, and
Enterobacter sp; as empiric therapy for serious infections in patients previously treated with
multiple antibiotics; as single agents in the treatment of febrile neutropenia; and as treatment for
polymicrobial and nosocomial infections. Thus, meropenem is appropriate therapy for the boy
described in the vignette, who is infected with a multiple drug-resistant strain of Klebsiella
pneumoniae (a gram-negative organism) that is resistant to the cephalosporins (including the
third-generation cephalosporins).
An infrequent adverse effect of imipenem (<1%) is seizures. This effect has been reported
in patients receiving very high doses who have underlying central nervous system pathology
and in patients who have decreased renal function in whom the dose has not been adjusted.
Seizures are not a major problem with meropenem, even in patients being treated for meningitis.
Cefuroxime is a second-generation cephalosporin to which the organism infecting the patient
in the vignette is resistant. Clindamycin only has activity against gram-positive organisms.
Because the K pneumoniae strain is resistant to ampicillin, it also is resistant to penicillin G. The
antibiotic resistance pattern of the identified pathogen indicates that one of its major resistance
mechanisms is the production of extended-spectrum beta-lactamases, which are enzymes that
hydrolyze many different penicillins and cephalosporins. Because of the production of these
enzymes, piperacillin is not an effective treatment option.
References:
Balfour JA, Bryson HM, Brogden RN. Imipenem/cilastatin: an update of its antibacterial activity,
pharmacokinetics and therapeutic efficacy in the treatment of serious infections. Drugs.
1996;51:99-136. Abstract available at: http://www.ncbi.nlm.nih.gov/pubmed/8741235
Nicolau DP. Carbapenems: a potent class of antibiotics. Expert Opin Pharmacother. 2008;9:2337. Abstract available at: http://www.ncbi.nlm.nih.gov/pubmed/18076336
Norrby SR. Carbapenems. Med Clin North Am. 1995;79:745-759. Abstract available at:
http://www.ncbi.nlm.nih.gov/pubmed/7791421
Wiseman LR, Wagstaff AJ, Brogden RN, Bryson HM. Meropenem: a review of its antibacterial
activity, pharmacokinetic properties and clinical efficacy. Drugs. 1995;50:73-101. Abstract
available at: http://www.ncbi.nlm.nih.gov/pubmed/7588092
Zhanel GG, Wiebe R, Diley L, et al. Comparative review of the carbapenems. Drugs.
2007;67:1027-1052. Abstract available at: http://www.ncbi.nlm.nih.gov/pubmed/17488146
page 132
A 10-year-old boy has marked fluid intake, frequent urination, and decreased visual acuity. On
physical examination, the boy is short (<5th percentile), neurologic evaluation findings are
normal, and no edema is present. His electrolyte values are normal. Other laboratory results
include:
Blood urea nitrogen, 36.0 mg/dL (12.9 mmol/L)
Creatinine, 2.0 mg/dL (176.8 mcmol/L)
Hemoglobin, 6.5 g/dL (65.0 g/L)
Urine specific gravity, 1.005
Urine pH, 6
Urine protein, 1+
Ophthalmologic evaluation reveals features of retinitis pigmentosa.
Of the following, the MOST likely diagnosis is
A. Alport syndrome
B. diabetic nephropathy
C. juvenile nephronophthisis
D. Lowe syndrome
E. nephropathic cystinosis
page 133
Preferred Response: C
The symptoms of polyuria and polydipsia; clinical finding of short stature; and laboratory findings
of azotemia, anemia, and proteinuria described for the patient in the vignette point to the
diagnosis of juvenile nephronophthisis (NPH). This autosomal recessive inherited disorder is
characterized by chronic tubulointerstitial disease. NPH begins with tubular dysfunction, which
progresses to azotemia and ultimately, end-stage renal disease (ESRD). The clinical
presentation includes symptoms consistent with a urinary concentrating defect (polyuria and
polydipsia) at 2 to 3 years of age that progresses to more overt symptoms in the next few
years. Tubular wasting of sodium can lead to hyponatremia and salt craving. Signs of acidosis,
anemia, and azotemia appear next. The median age of onset of ESRD is 13 years. NPH is
estimated to account for 5% to 10% of all cases of ESRD in children. The diagnosis is based
primarily on clinical symptoms; renal ultrasonography usually is not helpful because the finding of
mild increased echogenicity of the kidneys is nonspecific. Medullary cysts, which characterize
the disease, may be missed on ultrasonography but usually are seen on computed tomography
scan. Renal biopsy, which is not always performed, demonstrates tubular damage, with
interstitial fibrosis and normal glomeruli. Cysts may be seen in advanced disease.
NPH may be associated with other disorders, such as the Senior-Loken syndrome, which is
seen in 15% of cases. This syndrome involves the eye, with tapetoretinal degeneration, retinitis
pigmentosa (Item C62A), and the development of blindness. Other findings include nystagmus,
coloboma (Item C62B), and cataracts (Item C62C). NPH also is associated with Joubert
syndrome (NPH with aplasia of the cerebellar vermis causing ataxia and retinal
coloboma/retinitis pigmentosa), hepatic fibrosis, and skeletal defects (cone-shaped epiphyses).
Four genes have been identified in association with NPH. Most cases of juvenile NPH are
related to the NPHP1 gene, which is located on chromosome 2q12.3 and encodes for
nephrocystin 1. NPHP2 and NPHP3 encode for proteins for the infantile and adolescent forms of
NPH, respectively.
Alport syndrome is an X-linked disorder associated with a defect of the alpha 5 chain of type
IV collagen (Col4A5). It manifests with hematuria, high-frequency sensorineural hearing loss,
and anterior lenticonus (and occasionally cataracts). Diabetic nephropathy can result in retinal
and renal disease, but the patient in the vignette lacks a history of diabetes and has no
glycosuria. Lowe syndrome is an X-linked disorder characterized by Fanconi syndrome
(glycosuria, metabolic acidosis, aminoaciduria, and hypophosphatemia), mental retardation, and
congenital cataracts, but it does not result in azotemia, anemia, or retinal abnormalities.
Nephropathic cystinosis is a lysosomal storage disorder that has an autosomal recessive
inheritance pattern and involves the Fanconi syndrome, failure to thrive, and cystine crystal
deposition within the cornea (Item C62D), resulting in severe photophobia.
References:
Hildebrandt F. Nephronophthisis-medullary cystic kidney disease. In: Avner ED, Harmon WE,
Niaudet P, eds. Pediatric Nephrology. 5th ed. Philadelphia, Pa: Lippincott Williams & Wilkins;
2004:665-673
Niaudet P. Inherited nephropathies. In: Kher KK, Schnaper HW, Makker SP, eds. Clinical
Pediatric Nephrology. 2nd ed. London, England: Informa Healthcare; 2007:195-212.
Saunier S, Calado J, Benessy F, et al. Characterization of the NPHP1 locus: mutational
mechanism involved in deletions in familial juvenile nephronophthisis. Am J Hum Genet.
2000;66:778-789. Available at:
http://www.pubmedcentral.nih.gov/articlerender.fcgi?tool=pubmed&pubmedid=10712196
page 134
A mother brings in her 11-month-old son after he broke out in "hives" today during breakfast.
The infant had stayed home from child care with a low-grade fever, and the mother had let him
eat eggs for the first time. Immediately after breakfast, the mother noted a diffuse erythematous,
pruritic rash covering the boys trunk and extremities. She is concerned that her son may have
an egg allergy.
Of the following, the BEST statement regarding immunoglobulin E-mediated egg food allergy is
that
page 135
Preferred Response: C
Immunoglobulin (Ig) E-mediated egg allergy is one of the more common childhood food allergies,
affecting approximately 1% to 2% of children. As described in the vignette, cutaneous features
are common, including atopic dermatitis, urticaria, and pruritus. Once the diagnosis of egg
allergy is determined, patients generally are advised to avoid all egg food products with the hope
that most children will outgrow their egg allergy within 3 to 5 years (Item C63).
The primary allergenic egg protein is ovomucoid, a protein predominantly in the egg white.
Approximately 50% of children may be able to tolerate small amounts of egg protein that has
been heated extensively (eg, baked goods). Prolonged heating at high temperatures can
denature proteins from a conformational form to a linear form. Some children who are allergic to
eggs do not recognize the linear protein form as an allergen and, therefore, do not experience a
reaction. Of note, the brief cooking used to make scrambled eggs will not denature heat-stable
proteins.
The relationship between egg allergy and vaccination is a common question. The measlesmumps-rubella vaccine is safe for children who have egg allergy and should be administered
without special precautions. The trivalent influenza and live attenuated influenza vaccines
contain small amounts of egg protein and are contraindicated for patients who have egg allergy.
However, studies have supported a two-dose protocol for the administration of the influenza
vaccine in egg-allergic patients. The two-dose protocol involves administering one tenth of the
vaccine, observing the recipient for a period of time, and administering the rest of the vaccine,
followed by a similar observation period.
In westernized countries, milk generally is regarded as the most common food allergen in
infants, with an incidence of 2.5%, compared with an incidence of 1.5% for egg allergy.
References:
Boyano-Martnez, T, Garca-Ara C, Daz-Pena JM, and Martn-Esteban M. Prediction of
tolerance on the basis of quantification of egg white-specific IgE antibodies in children with egg
allergy. J Allergy Clin Immunol. 2002;110:304-309. Abstract available at:
http://www.ncbi.nlm.nih.gov/pubmed/12170273
Piquer-Gibert M, Plaza-Martn A, Martorell-Aragons A, et al. Recommendations for
administering the triple viral vaccine and antiinfluenza vaccine in patients with egg allergy.
Allergol Immunopathol (Madr). 2007;35:209-212. Available at: http://db.doyma.es/cgibin/wdbcgi.exe/doyma/mrevista.pubmed_full?inctrl=05ZI0102&rev=105&vol=35&num=5&pag=20
9
Sampson HA, Leung DYM. Adverse reactions to foods. In: Kleigman RM, Behrman RE, Jenson
HB, Stanton BF, eds. Nelson Textbook of Pediatrics. 18th ed. Philadelphia, Pa: Saunders
Elsevier; 2007:986-989
page 136
page 137
Preferred Response: D
Pediatric fracture patterns differ from those in adults for a variety of biomechanical, physiologic,
and anatomic reasons. Most importantly, children are at risk of orthopedic injuries involving the
growth plates (physes) that may lead to adverse sequelae if not diagnosed acutely and
managed appropriately. Physeal injuries occur most commonly in school-age children and
account for 15% to 30% of all pediatric fractures.
The physis, or growth plate, of the pediatric bone is located between the metaphysis and the
epiphysis. This region of the pediatric skeleton is composed predominantly of cartilage cells that
multiply rapidly, calcify along the metaphyses, and lead to longitudinal bone growth. Although the
growth activity at the physes is responsible for rapid fracture healing, damage to this area can
impair healing and future growth. This region of the bone is particularly vulnerable to injury
because the weaker physeal cartilage is less resistant to traumatic forces than the mineralized
bone and ligaments.
Epiphyseal fractures typically are categorized using the five-tier Salter-Harris classification,
a scale based on the degree of physeal, bone, and joint involvement (Item C64A and Item
C64B). This categorization largely determines fracture management and prognosis. The
radiograph of the boy described in the vignette indicates that he has suffered a type IV tibial
fracture (Item C64C). His fracture is unique in that it occurs through three anatomic planes:
coronally through the distal tibial metaphysis, transversely through the physis, and sagittally
through the epiphysis (ie, a triplane fracture). These fractures occur because the physis has not
fused completely. An associated fibular fracture is a common finding.
Osteochondritis desiccans most commonly affects the medial femoral condyle of the knee.
Caused by repetitive stress, it is characterized by bone fragment development and separation at
the posterolateral aspect of the distal femur (Item C64D).
Unicameral, or simple, bone cysts usually are found in the metaphyses of long bones of
children between 4 and 12 years (Item C64E). They may lead to pathologic fractures but most
often are asymptomatic and stop expanding after skeletal maturity is reached.
Unless the fracture is open, osteomyelitis is not a likely complication. Distal tibial avascular
necrosis has been reported as an extremely rare complication of severe fractures in adults
whose physes already are closed.
References:
Dinolfo EA, Adam HM. In brief: fractures. Pediatr Rev. 2004;25:218-219. Available at:
http://pedsinreview.aappublications.org/cgi/content/full/25/6/218
Gholve PA, Hosalkar HS, Wells L. Common fractures. In: Kleigman RM, Behrman RE, Jenson
HB, Stanton BF, eds. Nelson Textbook of Pediatrics. 18th ed. Philadelphia, Pa: Saunders
Elsevier; 2007:2834-2841
Perron AD, Miller MD, Brady WJ. Orthopedic pitfalls in the ED: pediatric growth plate injuries. Am
J Emerg Med. 2002;20:50-54. Abstract available at:
http://www.ncbi.nlm.nih.gov/pubmed/11781914
page 138
A 4-month-old male infant presents for his initial examination. The family recently immigrated to
the United States from southeast Asia. They describe progressive abdominal distention (Item
Q65A) in the infant over the past 2 months. Physical examination demonstrates a firm liver edge
2 cm below the right costal margin and a spleen tip palpable 3 cm below the costal margin.
Abdominal ultrasonography demonstrates a fluid-filled abdomen (Item Q65B).
Of the following complications from his underlying disorder, this child is MOST at risk for
A. acute intussusception
B. emphysematous chronic lung disease
C. gastrointestinal bleeding
D. pneumococcal meningitis
E. renal failure
page 139
Preferred Response: C
The firm liver, enlarged spleen, and fluid in the abdomen seen on ultrasonography described for
the infant in the vignette is consistent with a number of clinical scenarios, including infection and
malignancy. However, the findings strongly suggest chronic liver disease, with secondary portal
hypertension resulting in ascites and splenomegaly. Accordingly, he is at risk for esophageal
variceal bleeding. Acute intussusception, emphysematous chronic lung disease, pneumococcal
meningitis, and renal failure are not associated directly with portal hypertension.
Portal hypertension may occur from either hepatic venous thrombosis (Budd-Chiari
syndrome), portal venous thrombosis ("cavernous transformation of the portal vein"), or hepatic
cirrhosis. Hepatic cirrhosis is the most common cause in both adults and children. In adults,
alcoholic cirrhosis is the most common cause. In children, cirrhosis may occur from a number of
congenital, hereditary, and infectious conditions. These include biliary atresia, Alagille syndrome,
alpha-1-antitrypsin deficiency, cystic fibrosis, hepatitis C, congenital hepatic fibrosis, and
congestive heart failure.
The portal vein drains the intestinal blood and lymph that subsequently enter the liver, where
blood and nutrients are filtered through the hepatic sinusoids. The cirrhotic liver has increased
resistance to the flow of portal blood, resulting in increased portal venous pressure. This
increase in pressure is transmitted to the splenic vein (resulting in splenomegaly) and the
esophageal and hemorrhoidal venous systems (resulting in esophageal varices and
hemorrhoids). The changes in oncotic pressure also may result in hypoalbuminemia and
accumulation of ascitic fluid in the abdominal cavity, as seen in this infant.
The earliest clinical sign that suggests portal hypertension usually is an enlarged spleen. As
liver disease progresses, ascites, prominent abdominal veins, and esophageal or gastric
varices may develop. Many patients who have portal hypertension exhibit laboratory signs of
chronic liver disease, including transaminase elevation and jaundice. However, patients who
have some conditions associated with portal hypertension, including congenital hepatic fibrosis
and alpha-1-antitrypsin deficiency, may not have jaundice. In such patients, portal hypertension
should be suspected if the liver or spleen is chronically enlarged.
References:
Molleston JP. Variceal bleeding in children. J Pediatr Gastroenterol Nutr. 2003;37:538-545
Shepherd RW, Ramm GA. Liver function and dysfunction: fibrogenesis and cirrhosis. In: Walker
WA, Goulet O, Kleinman RE, Sherman PM, Shneider BL, Sanderson IR. Pediatric
Gastrointestinal Disease. 4th ed. Hamilton, Ontario, Canada: BC Decker; 2004:80-88
page 140
A 2.1-kg, 34-week gestation infant is delivered to a mother who has chorioamnionitis and had a
positive group B streptococcal urinary tract infection at 30 weeks of gestation. Four hours after
birth, the infant requires admission to the intensive care nursery because of respiratory distress.
Physical examination reveals a temperature of 96.8F (36.0C), heart rate of 160 beats/min,
respiratory rate of 80 breaths/min, blood pressure of 60/30 mm Hg, mean arterial pressure of 40
mm Hg, and pulse oximetry of 82% on room air. The infant audibly grunts, has flaring of the ala
nasi and intercostal and subcostal chest wall retractions, and is poorly perfused, with a capillary
refill time of 4 seconds and mild acrocyanosis. There is no heart murmur.
Of the following, the MOST likely radiographic findings expected for this infant are
A. air bronchograms, diffusely hazy lung fields, and low lung volume
B. cardiomegaly, hazy lung fields, and pulmonary vascular engorgement
C. fluid density in the horizontal fissure, hazy lung fields with central vascular prominence, and
normal lung volume
D. gas-filled loops of bowel in the left hemithorax and opacification of the right lung field
E. patchy areas of diffuse atelectasis, focal areas of air-trapping, and increased lung volumes
page 141
Preferred Response: A
Infants delivered at any gestational age are at risk for respiratory distress from many causes.
Respiratory distress syndrome (RDS) is due to surfactant deficiency and characteristically
affects newborns delivered at less than 32 weeks' gestation. In the term or late preterm infant,
such as the infant described in the vignette, additional or alternative causes of respiratory
distress should be considered, including infection, aspiration, cardiac disease, or congenital
anomalies (eg, tracheoesophageal fistula, diaphragmatic hernia). Infants born to mothers who
have a history of prolonged rupture of membranes, chorioamnionitis, or known maternal group B
streptococcal carriage have a high risk for infection, including congenital pneumonia.
Respiratory distress may be seen anytime after birth but characteristically is present in the first
24 hours. Tachypnea, tachycardia, and increased work of breathing with a supplemental oxygen
requirement are the most common clinical signs. Temperature instability, irritability, lethargy, and
poor feeding also may be present. The chest radiograph for a newborn who has congenital
pneumonia may resemble that of RDS with hyaline membrane disease (air bronchograms,
diffusely hazy lung fields, and low lung volumes) (Item C66A). Rarely, a lobar infiltrate may be
seen.
The most likely infectious agents in congenital pneumonia include those that ascend from
the genital tract before or during labor: group B streptococci, Escherichia coli, and Klebsiella sp.
No specific clinical sign or laboratory finding in the sick newborn confirms the presence or
absence of pneumonia. Therefore, all newborns who exhibit respiratory distress should have
blood cultures obtained and be started on broad-spectrum antibiotic therapy (eg, ampicillin and
gentamicin) until a diagnosis of bacterial infection can be excluded. Obtaining a Gram stain of
tracheal aspirates from newborns in whom pneumonia is suspected and who have required
tracheal intubation and assisted ventilation may be helpful.
The chest radiograph depicting cardiomegaly, hazy lung fields, and pulmonary vascular
engorgement is characteristic of congenital heart disease with pulmonary overcirculation (eg,
critical aortic stenosis, total anomalous pulmonary venous return with obstruction, truncus
arteriosus, transposition of the great arteries) (Item C66B). Fluid densities in the horizontal
fissure, hazy lung fields with central vascular prominence, and normal lung volumes are
characteristic of retained fetal lung fluid, referred to clinically as transient tachypnea of the
newborn (Item C66C). Gas-filled loops of bowel in either hemithorax are suspicious for a
diaphragmatic hernia, more commonly seen on the left, with a mediastinal shift to the right and
compression atelectasis and opacification of the right lung field (Item C66D). Patchy atelectasis
with focal areas of air-trapping and increased lung volumes are characteristic of aspiration
syndromes, most notably meconium (Item C66E).
References:
Aly H. Respiratory disorders in the newborn: identification and diagnosis. Pediatr Rev.
2004;25:201-208. Available at: http://pedsinreview.aappublications.org/cgi/content/full/25/6/201
Herting E, Gefeller O, Land M, van Sonderen L, Harms K, Robertson B, and Members of the
Collaborative European Multicenter Study Group. Surfactant treatment of neonates with
respiratory failure and group B streptococcal infection. Pediatrics. 2000;106:957-964. Available
at: http://pediatrics.aappublications.org/cgi/content/full/106/5/957
Sivit CJ. Diagnostic imaging. In: Martin RJ, Fanaroff AA, Walsh MC, eds. Fanaroff and Martin's
Neonatal-Perinatal Medicine. 8th ed. Philadelphia, Pa: Mosby Elsevier; 2006:713-732
Thilo EH, Rosenberg AA. The newborn infant. In: Hay WW Jr, Levin M, Sondheimer JM,
Deterding RR, eds. Current Pediatric Diagnosis & Treatment. 18th ed. New York, NY: The
McGraw-Hill Companies, Inc; 2007:chap 1
page 142
A 16-month-old boy is brought to your clinic because his mother says he is "walking funny"
today. She states that he has been walking for 4 months and is very active, but she is unaware
of any trauma or falls. She denies fever or other symptoms. He appears well and has normal
vital signs. Physical examination reveals mild tenderness to palpation over the medial aspect of
the lower leg just above the ankle. There is no overlying bruising, erythema, or edema, and you
can elicit full range of motion in the hips, knees, and ankles.
Of the following, the MOST likely diagnosis is
page 143
Preferred Response: C
The differential diagnosis of the child who has an acute limp includes infectious causes such as
osteomyelitis and septic arthritis, postinfectious entities such as transient synovitis and
rheumatic fever, and injuries related to accidental or nonaccidental trauma. The presence of
fever or other signs and symptoms, including erythema and limitation of joint motion, should lead
the clinician to investigate infectious or postinfectious causes. However, in the absence of
constitutional symptoms and typical external signs of infection, such as described for the boy in
the vignette, an occult fracture should be considered, even if no history of trauma is elicited.
Careful questioning regarding any falls or injuries, observation of the parent-child interaction,
and a complete physical examination that includes the skin are important because many children
who have fractures from inflicted trauma come to medical attention because of vague limb pain
or limp. However, the presence of a fracture in a toddler does not necessarily indicate inflicted
injury (eg, child abuse) because children who are just learning to walk and run can fall and
sustain injury without the parent's awareness of the event.
Anteroposterior and lateral radiographs of the lower extremities are indicated if a fracture is
suspected, and the type of fracture seen can provide some clues as to the mechanism of injury.
One of the most common fractures in children is a nondisplaced spiral fracture of the tibia,
otherwise as known as a "toddler's fracture" (Item C67A). The mechanism of injury is a twisting
movement of the lower leg that occurs as the toddler learns to move around his or her
environment. Examination often reveals only a limp or tenderness over the affected area. Initial
radiographs can appear normal, with a periosteal reaction appearing in follow-up films. Torus
(buckle) and greenstick (Item C67B) fractures also can be seen with relatively minor trauma
because children's bones are more porous and more likely to "buckle" or bend with trauma than
adult bones. Fractures of the growth plate are common with trauma and are particularly
concerning because of the risk of growth retardation at the site of the fracture. The Salter-Harris
system is used to classify types of growth plate fractures and identify patients who are at high
risk of growth failure. Finally, some types of fractures should alert the clinician immediately to the
possibility of child abuse, including multiple fractures in different stages of healing, metaphyseal
"chip" fractures (Item C67C), femoral fracture in children younger than 1 year of age (Item
C67D), and scapular fracture.
The minimal external signs and no constitutional symptoms such as fever reported for the
child in the vignette make the diagnosis of osteomyelitis unlikely. Transient synovitis occurs after
a viral illness and generally affects the knees and hips. Some limitation of motion at the joint is
expected, but external signs such as erythema typically are absent. Aneurysmal bone cysts are
metaphyseal lesions (Item C67E) that usually occur in adolescence and cause pain with or
without swelling. The ligaments and tendons in children generally are stronger than the growing
bones underneath them, in contrast to adults, whose bones are stronger. The amount of injuring
force required to cause a sprain in an adult is more likely to cause a fracture in a child.
References:
Dinolfo EA. Fractures. Pediatr Rev. 2004;25:218-219. Available at:
http://pedsinreview.aappublications.org/cgi/content/full/25/6/218
Eiff MP, Hatch RL. Boning up on common pediatric fractures. Contemp Pediatr. 2003;20:30-59
Kellogg ND and the Committee on Child Abuse and Neglect. Evaluation of suspected child
physical abuse. Pediatrics. 2007;119:1232-1241. Available at:
http://pediatrics.aappublications.org/cgi/content/full/119/6/1232
page 144
An 11-year-old girl presents 2 weeks after an office visit for a presumed viral illness
characterized by fever, malaise, and flushing of the cheeks. Today, her mother notes that she
no longer has a fever, but she complains of pain in her knees and elbows. On physical
examination, the left knee is slightly swollen and warm but not erythematous. The girl reports
pain on movement of both elbows, but there are no physical findings on examination of the
elbows or other joints. The remainder of the physical examination findings are normal, except for
an oral temperature of 100.6F (38.1C). Results of laboratory studies include a white blood cell
count of 8.9x103/mcL (8.9x109/L) with 40% polymorphonuclear leukocytes, 45% lymphocytes,
and 15% monocytes; hemoglobin of 11.0 g/dL (110.0 g/L); platelet count of 472.0x103/mcL
(472.0x109/L); and erythrocyte sedimentation rate of 20 mm/hr.
Of the following, the MOST likely pathogen to cause this childs joint complaints is
A. Borrelia burgdorferi
B. Coxsackievirus
C. group A beta-hemolytic streptococci
D. influenza A virus
E. parvovirus B19
page 145
Preferred Response: E
The girl described in the vignette has developed swelling of the knee and arthralgias following a
recent febrile illness, which is typical of a postinfectious arthritis. The flushing of her cheeks
suggests that her febrile illness was due to human parvovirus B19 infection, commonly known
as erythema infectiosum (EI) (Item C68). Parvovirus B19 infection is a common cause of
postinfectious arthritis. Multiple viruses can cause postinfectious arthritis, including influenza,
hepatitis B, rubella, and Epstein-Barr.
Arthralgias may occur in 10% of children who have clinical or laboratory evidence of EI.
Older children, particularly girls and young women, frequently experience involvement of the
knees, although involvement of both large and small joints has been reported. Parvovirus B19
has been detected in synovial fluid and serum samples of such patients. Some clinical features
of parvovirus B19 infection are similar to those of autoimmune connective tissue diseases, and
some children who have EI may develop positive antinuclear antibody serum test results or
rheumatoid factor-positive serology.
Lyme disease results from infection with the spirochete Borrelia burgdorferi, which is
transmitted by deer tick bites. In North America, Lyme disease is most prevalent in the
northeastern, midwestern, and southern and western coastal areas of the United States as well
as in Ontario, Canada. School-age children are affected most commonly, with boys and girls
affected equally. Arthritis is the second most frequent presentation of Lyme disease, following
the cutaneous signs of erythema migrans. Arthralgias usually develop in the early phase, but the
onset of arthritis may occur months to years after the original infection. Initially, the arthritis is
episodic, but it may evolve to a recurrent and prolonged condition. Two thirds of affected
children present with monoarthritis of the knee, but oligoarticular involvement of the large joints
and, rarely, a polyarthritis of the small joints also can occur.
The diagnosis of Lyme arthritis is based on history and physical examination findings as well
as laboratory tests to document infection with B burgdorferi. The immunoglobulin G titers to B
burgdorferi can remain positive for years and, therefore, cannot be used to monitor treatment
response or failure. In contrast to adults, the prognosis for Lyme arthritis in children generally is
good, and symptoms resolve over time without permanent damage to joints.
Group A beta-hemolytic streptococcal infection can be associated with arthritis that may
occur during the acute illness or after the acute illness has resolved (poststreptococcal reactive
arthritis). Other bacterial causes of postinfectious arthritis include Neisseria gonorrhoeae,
Staphylococcus aureus, and other streptococcal species. Coxsackieviruses are not often
associated with arthritis. Enterovirus, hepatitis B, rubella, and mumps infections may cause
transient arthritis. Many other infections result in malaise and myalgias associated with a
prodrome, but they do not cause true arthritis.
References:
American Academy of Pediatrics. Enterovirus (nonpoliovirus) infections (group A and B
Coxsackieviruses, echoviruses, numbered enteroviruses. In: Pickering LK, Baker CJ, Long SS,
McMillan JA, Eds. Red Book: 2006 Report of the Committee on Infectious Diseases. 27th ed. Elk
Grove Village, Ill: American Academy of Pediatrics; 2006:284-285
American Academy of Pediatrics. Group A streptococcal infections. In: Pickering LK, Baker CJ,
Long SS, McMillan JA, Eds. Red Book: 2006 Report of the Committee on Infectious Diseases.
27th ed. Elk Grove Village, Ill: American Academy of Pediatrics; 2006:610-620
American Academy of Pediatrics. Influenza. In: Pickering LK, Baker CJ, Long SS, McMillan JA,
Eds. Red Book: 2006 Report of the Committee on Infectious Diseases. 27th ed. Elk Grove
Village, Ill: American Academy of Pediatrics; 2006:401-411
American Academy of Pediatrics. Parvovirus B19/erythema infectiosum, fifth disease). In:
Pickering LK, Baker CJ, Long SS, McMillan JA, Eds. Red Book: 2006 Report of the Committee
on Infectious Diseases. 27th ed. Elk Grove Village, Ill: American Academy of Pediatrics;
page 146
2006:484-487
Khouqeer R, Cohen M. Viral arthritis. eMedicine Specialties, Rheumatology, Infectious Arthritis.
2006. Available at: http://www.emedicine.com/med/topic3414.htm
Koch WC. Parvovirus B19. In: Kliegman RM, Behrman RE, Jenson HB, Stanton BF, eds.
Nelson's Textbook of Pediatrics. 18th ed. Philadelphia, Pa: Saunders Elsevier; 2007:1357-1359
Lehman HW, Knll, A, Kster RM, Modrow S. Frequent infection with a viral pathogen,
parvovirus B19, in rheumatic diseases of childhood. Arthritis Rheum. 2003;48:1631-1638.
Available at: http://www3.interscience.wiley.com/cgi-bin/fulltext/104536478/HTMLSTART
Siegel DM. In brief: antinuclear antibody (ANA) testing. Pediatr Rev. 2003;24:320-321. Available
at: http://pedsinreview.aappublications.org/cgi/content/full/24/9/320
Tse SML, Laxer RM. Approach to acute limb pain in childhood. Pediatr Rev. 2006;27:170-180.
Available at: http://pedsinreview.aappublications.org/cgi/content/full/27/5/170
page 147
You are prescribing atenolol for a 15-year-old boy in whom you diagnosed hypertrophic
cardiomyopathy. There is a family history of asthma. He is concerned about the potential
adverse effects of medicines.
Of the following, a TRUE statement about treatment with this drug is that
page 148
Preferred Response: D
Beta-blocking drugs have an antagonistic effect on the beta-adrenergic receptors. This diverse
and widely used group of medications can have multiple and various effects on the heart and
other organ systems. In the heart, they typically exhibit some degree of negative chronotropic
(slowing of the heart rate), negative dromotropic (slowing of the conduction through the
atrioventricular node), and negative inotropic (decrease in the ventricular force of contraction)
effects. As a result, this class of medications is used in the treatment of many pediatric
conditions, including hypertension, arrhythmias, hypertrophic cardiomyopathy, dilated
cardiomyopathy, Marfan syndrome, and migraine prophylaxis.
The adverse effects of beta-blocker drugs have been well-documented and may be divided
into three broad categories based on end-organ effects (Item C69).
The patient described in the vignette has been prescribed a beta blocker that has relative
cardiac selectivity. Some children and adolescents experience a decrease in school
performance while being treated with beta blockers, most likely related to the central nervous
penetration, and it may be difficult to differentiate this effect from the known fatigue or depression
that also can occur.
Although bronchospasm or asthma is a contraindication to the use of some beta blockers
(eg, propranolol), a family history alone is not a contraindication for their use. Some beta
blockers can cause a decreased reaction to hypoglycemia, leading to a relative contraindication
of their use in those who have diabetes, but their use does not increase the risk of developing
diabetes mellitus. Similarly, beta blocker use typically does not lead to gynecomastia. Although
reflex tachycardia may be noted when chronic beta blocker use is discontinued, these drugs are
not associated with tachyphylaxis.
References:
Feld LG, Corey H. Hypertension in childhood. Pediatr Rev. 2007;28:283-298. Available at:
http://pedsinreview.aappublications.org/cgi/content/full/28/8/283
Opie LH, Sonnenblick EH, Frishman WH, Thadani U. Beta-blocking agents. In: Opie LH, ed.
Drugs for the Heart. 4th ed. Philadelphia, Pa: W.B. Saunders Co; 1995:1-30
page 149
A 14-year-old girl is brought to the emergency department because she has back pain and a
sudden inability to walk. Neurologic examination shows normal upper limb strength. However,
her legs are flaccid, relatively symmetrically weak, areflexic, and numb to pinprick. Vibratory and
position sense in the legs persists. A sensory deficit exists below the sixth thoracic dermatome.
Rectal examination shows low rectal tone. The remainder of her physical examination findings,
including vital signs, are normal.
Of the following, the MOST appropriate next diagnostic study is emergent
page 150
Preferred Response: E
The child described in the vignette has findings consistent with an acute spinal cord lesion,
which is a neurologic emergency. In some cases, emergency neurosurgery is needed for
lesions causing acute spinal cord compression, and the initial diagnostic emphasis should be
directed toward identifying such lesions. The first step is to localize the problem to the proper
level of the nervous system: brain, brainstem/cerebellum, spinal cord, nerve, junction, or
muscle. Such localization not only allows for urgent appropriate diagnosis and treatment but can
reduce discomfort, risks, and costs of inappropriate diagnostic testing. Urgent phone or inperson consultation with a neurologist or neurosurgeon can be useful before ordering testing.
The back pain, acute bilateral flaccid weakness, and sensory loss below the level of the
lesion described for the girl in the vignette localize this problem to the spinal cord. In this case,
the rapid onset of symptoms and the preservation of vibratory and proprioceptive sensation
point to the anterior cord, bilaterally, which is consistent with an anterior spinal artery stroke.
Spinal cord strokes in children are uncommon but can occur after aortic surgery or as a
consequence of thrombotic disorders, infection, inflammatory diseases, or trauma.
Based on the history and examination findings, magnetic resonance imaging (MRI) of the
spine is the test of choice. This should aid in determining the specific location of the lesion and
whether emergency neurosurgery to decompress the spinal cord is needed.
Brain MRI is not the initial modality of choice because of the urgency of the presentation and
the localization of the lesion suggested by clinical findings. If a spinal cord lesion is radiologically
consistent with the diagnosis of transverse myelitis, a subsequent brain MRI with gadolinium
should be obtained to look for evidence of demyelinating lesions in the brain, which is supportive
of a diagnosis of multiple sclerosis.
A subsequent lumbar puncture also may help identify the cause of the patient's symptoms,
but it may not be necessary and should not delay obtaining the spine MRI. Lumbar puncture is
the appropriate test for suspected Guillain-Barr syndrome, an acute inflammatory
demyelinating polyneuropathy, which can present with weakness beginning in the proximal legs
but not with the sensory findings seen in this child. Lumbar puncture is also helpful for identifying
inflammatory causes of spinal cord disease. Somatosensory evoked potentials are
neurophysiologic tests that document the integrity of sensory pathways from peripheral nerves
through the spinal cord to the brain. They are not part of standard emergency management in
this clinical setting. Nerve conduction studies and electromyography are not needed to assess
the nerves because the problem localizes to spinal cord.
References:
Hakimi KN, Massagli TL. Anterior spinal artery syndrome in two children with genetic thrombotic
disorders. J Spinal Cord Med. 2005;28:69-73. Abstract available at:
http://www.ncbi.nlm.nih.gov/pubmed/15832907
Haslam RHA. Spinal cord disorders. In: Kliegman RM, Behrman RE, Jenson HB, Stanton BF,
eds. Nelson Textbook of Pediatrics. 18th ed. Philadelphia, Pa: Saunders Elsevier; 2007:25262530
Menkes JH, Ellenbogen RC. Traumatic brain and spinal cord injuries in children. In: Maria BL,
ed. Current Management in Child Neurology. 3rd ed. Hamilton, Ontario, Canada: BC Decker Inc;
2005:515-527
Nance JR, Golomb MR. Ischemic spinal cord infarction in children without vertebral fracture.
Pediatr Neurol. 2007;36:209-216. Abstract available at:
http://www.ncbi.nlm.nih.gov/pubmed/17437902
page 151
An infant in the newborn nursery is normally grown and normally formed, except for a
preauricular pit (preauricular sinus) bilaterally (Item Q71). He has passed his newborn hearing
screening. When you meet the babys mother, you learn that she has progressive, bilateral
sensorineural hearing loss for which she uses hearing aids.
Of the following, the MOST helpful test to aid in diagnosis and management of this babys
condition is
A. chromosome analysis
B. head ultrasonography
C. ophthalmology consultation
D. radiographic skeletal survey
E. renal ultrasonography
page 152
Preferred Response: E
For many years, the association between ear anomalies and other malformations, especially
involving the kidneys, have been noted in the medical literature. Based on numerous studies, the
documented frequency of defects in other organ systems is 5% to 40% in children who have ear
anomalies.
Largely due to the association between crumpled, deformed ears and renal agenesis (as
seen in Potter sequence), it has become customary in many centers to evaluate the kidneys in
the newborn, even if only a preauricular pit/sinus (PPS) is noted (Item C71). However, recent
studies have shown that PPS alone is not associated with an increased risk for renal anomalies,
and renal ultrasonography is not warranted in these cases.
It is critical to evaluate newborns and children who have auricular anomalies carefully for
dysmorphic features and evidence of organ malformations. It is important to know if the mother
has diabetes because diabetic embryopathy is associated with ear anomalies. In addition, a
family history should be obtained, with special attention to any individuals who have ear
anomalies, deafness, and renal malformations. Should such a history be elicited, as for the
mother in the vignette, renal ultrasonography is warranted, and further evaluations may be in
order. Chromosome analysis, head ultrasonography, ophthalmology consultation, and
radiographic skeletal survey are not necessary initial tests for a child who has auricular
anomalies.
References:
Adam M, Hudgins L. The importance of minor anomalies in the evaluation of the newborn.
NeoReviews. 2003;4:e99-e104. Available for subscription at:
http://neoreviews.aappublications.org/cgi/content/full/4/4/e99
Arora RS, Pryce R. Is ultrasonography required to rule out renal malformations in babies with
isolated preauricular tags? Arch Dis Child. 2004;89:492-493
Huang XY, Tay GS, Wansaicheong GK-L, Low WK. Preauricular sinus: clinical course and
associations. Arch Otolaryngol Head Neck Surg. 2007;133:65-68. Available at:
http://archotol.ama-assn.org/cgi/content/full/133/1/65
Wang RY, Earl DL, Ruder RO, Graham JM Jr. Syndromic ear anomalies and renal ultrasounds.
Pediatrics. 2001;108:e32. Available at:
http://pediatrics.aappublications.org/cgi/content/full/108/2/e32
page 153
A 16-year-old girl comes to your office with complaints of a thick white vaginal discharge. She is
sexually active with one partner with whom she always uses condoms. She has no complaints
of fever or abdominal pain, but she reports external "burning" of the vaginal area when she
urinates. On physical examination, she is afebrile. Pelvic examination reveals fiery red labia
majora and minora and an adherent white discharge on the vaginal walls, with a moderate
amount of white discharge in the vaginal vault. The speculum examination is uncomfortable for
her, but there is no cervical motion, uterine, or adnexal tenderness, and the cervix shows no
friability or discharge.
Of the following, the MOST likely pathogen responsible for this patients symptoms is
A. Candida albicans
B. Chlamydia trachomatis
C. group A Streptococcus
D. Neisseria gonorrhoeae
E. Trichomonas vaginalis
page 154
Preferred Response: A
Candidal vulvovaginitis is a common problem for young women and usually is caused by
Candida albicans, although other candidal species also may be involved. Typical symptoms of
vulvovaginal candidiasis include a thick, white, creamy vaginal discharge; pruritus; vaginal
discomfort; dyspareunia; and external dysuria. The diagnosis is suggested clinically by the
previously noted symptoms and the presence of vulvar swelling, erythema, and fissures or
erosions, as described for the girl in the vignette. The diagnosis may be confirmed by a wet
preparation or Gram stain showing pseudohyphae or yeasts (Item C72A) or by culture.
Chlamydia trachomatis and Neisseria gonorrhoeae, both of which may present with
abnormal vaginal discharge, produce cervical, not vaginal infections. C trachomatis and N
gonorrhoeae infections can be asymptomatic, but they often present with a yellowish purulent or
mucopurulent endocervical discharge, friability of the endocervix, and cervical motion
tenderness; vulvar inflammation does not occur. Group A Streptococcus (S pyogenes) is a
respiratory pathogen that can cause vaginitis in prepubertal girls, but rarely causes vaginal
discharge in the adolescent. Infection with Trichomonas vaginalis usually causes a diffuse,
malodorous, yellow-green vaginal discharge (Item C72B) with vulvar irritation, although some
affected women can have minimal or no symptoms.
References:
American Academy of Pediatrics. Chlamydia trachomatis. In: Pickering LK, Baker CJ, Long SS,
McMillan JA, eds. Red Book: 2006 Report on the Committee on Infectious Diseases. 27th ed.
Elk Grove Village, Ill: American Academy of Pediatrics; 2006:252-257
American Academy of Pediatrics. Gonococcal infections. In: Pickering LK, Baker CJ, Long SS,
McMillan JA, eds. Red Book: 2006 Report on the Committee on Infectious Diseases. 27th ed.
Elk Grove Village, Ill: American Academy of Pediatrics; 2006:301-309
American Academy of Pediatrics. Group A streptococcal infections. In: Pickering LK, Baker CJ,
Long SS, McMillan JA, eds. Red Book: 2006 Report on the Committee on Infectious Diseases.
27th ed. Elk Grove Village, Ill: American Academy of Pediatrics; 2006:610-620
Emans SJ. Office evaluation of the child and adolescent. In: Emans SJH, Laufer MR, Goldstein
DP, eds. Pediatric and Adolescent Gynecology. 5th ed. Philadelphia, Pa: Lippincott, Williams &
Wilkins; 2005:1-50
Workowski KA, Berman SM, Centers for Disease Control and Prevention. Sexually transmitted
diseases treatment guidelines, 2006. MMWR Recomm Rep. 2006;55(RR11):1-94. Available at:
http://www.cdc.gov/mmwr/preview/mmwrhtml/rr5511a1.htm
page 155
An 18-month-old boy fell into a swimming pool 12 hours ago. He had no heart rate when he was
pulled from the pool, and cardiopulmonary resuscitation (CPR) was initiated at the scene. The
CPR was continued for 30 minutes until spontaneous circulation was restored in the emergency
department. He is now in the pediatric intensive care unit, receiving mechanical ventilation with
maximal intensive care support. Over the past several hours, his blood pressure has increased,
he has developed persistent bradycardia, and he exhibits no movement in response to
stimulation. He has not received any neuromuscular blockers or sedation. In addition, his pupils
are dilated bilaterally and do not respond to light. Bedside electroencephalography demonstrates
generalized burst suppression with loss of reactivity to external stimuli.
In discussion with his parents, you inform them that these recent changes are MOST likely a
result of
A. agitation
B. increasing intracranial pressure
C. myocardial failure
D. ongoing seizure activity
E. physiologic response to the ventilator
page 156
Preferred Response: B
Hypoxic-ischemic injury, which occurs after periods of impaired perfusion, such as with
cardiopulmonary arrest, produces a broad spectrum of organ failure. Unfortunately, the brain is
one of the organs most vulnerable to decreased oxygen and nutrient delivery, and hypoxicischemic central nervous system (CNS) injury is common following asphyxia. Irreversible CNS
injury may occur after as little as 3 to 5 minutes of interrupted blood flow or oxygen delivery.
Both ischemia and hypoxia trigger numerous pathophysiologic processes that result in cellular
injury, cell death, and subsequent development of cerebral edema that compromises blood flow
to adjacent areas of the brain. These areas are initially either potentially recoverable or uninjured
but at risk due to impaired perfusion.
The boy described in the vignette is exhibiting evidence of increased intracranial pressure
due to significant global hypoxic-ischemic injury. His hypertension and bradycardia represent
the Cushing reflex (the raising of systemic arterial pressure to increase cerebral perfusion with
associated stimulation of the carotid bodies and subsequent bradycardia). Cushing triad
(systemic arterial hypertension, bradycardia, and depressed or irregular respirations) is a late
sign of increased intracranial pressure that often develops just prior to cerebral herniation. The
lack of spontaneous movements and dilated, unresponsive pupils described for the boy are
consistent with severe CNS injury and a poor outcome.
Agitation or pain due to underlying injuries, ventilator asynchrony, or anxiety can produce
hypertension, but this typically is associated with tachycardia and increased motor movements.
Myocardial hypoxic-ischemic injury normally is manifested by decreased ventricular function
and hypotension. Although patients who have hypoxic-ischemic injuries are at risk for the
development of seizures, the electroencephalographic findings in this patient are consistent with
a severe hypoxic-ischemic encephalopathy and portend a poor prognosis. Abnormal motor
movements or vital signs in combination with neurologic examination results that are not
consistent with a known cause should prompt the consideration of seizure and subsequent
evaluation and treatment.
References:
Doherty DR, Hutchison JS. Hypoxic ischemic encephalopathy after cardiorespiratory arrest. In:
Wheeler DS, Wong HR, Shanley T, eds. Pediatric Critical Care Medicine: Basic Science and
Clinical Evidence. New York, NY: Springer-Verlag; 2007:935-946
Kallas HJ. Drowning and submersion injury. In: Kliegman RM, Behrman RE, Jenson HB, Stanton
BF, eds. Nelson Textbook of Pediatrics. 18th ed. Philadelphia, Pa: Saunders Elsevier; 2007:438449
Meyer RJ, Theodorou AA, Berg RA. Childhood drowning. Pediatr Rev. 2006;27:163-169.
Available at: http://pedsinreview.aappublications.org/cgi/content/full/27/5/163
page 157
On the initial health supervision visit of a 7-year-old boy who is new to your practice, you note
that his height is 43 inches, which is at the 50th percentile for a 5-year-old, and that his weight is
appropriate for his age. His parents say that he has been wearing the same size clothes for at
least the past year. The boy also has dry skin. You suspect he has hypothyroidism and decide
to measure thyroid-stimulating hormone concentrations.
Of the following, the MOST appropriate additional study needed to evaluate this child for
hypothyroidism is
page 158
Preferred Response: B
The initial evaluation of the boy described in the vignette should be directed at determining
whether he has hypothyroidism and defining its severity. The assay of free thyroxine (FT4)
generally provides the best estimate of circulating active concentrations of thyroxine. Total
thyroxine (TT4) concentrations are dependent on circulating thyroxine and on circulating binding
proteins, including thyroxine-binding globulin (TBG) and thyroxine-binding prealbumin (TBPA). In
general, TT4 is an accurate measure of thyroid hormone sufficiency, but about 1 in 8,000
individuals has familial sex-linked deficiency or excess of TBG and, therefore, has a TT4 value
that cannot be interpreted properly. Although hypothyroidism in children usually is caused by
primary thyroid gland failure, a substantial proportion of affected individuals could have a
hypothalamic or pituitary disease, in which case the FT4 value is low and the thyroid-stimulating
hormone value is normal or low.
Tri-iodothyronine (T3) is the active form of intracellular thyroid hormone. Only about 15% of
T3 is produced by the thyroid gland; the rest is produced by the enzymatic deiodination of T4 in
most cells of the body, including the hepatocyte. Because the process of enzymatic deiodination
is nutritionally controlled, individuals who are particularly well-fed or obese or small children who
eat frequently have elevated T3 values compared with adult normal values. In contrast, sick
people or people who are not eating well have very low values. Therefore, T3 cannot be used to
diagnose hypothyroidism.
Children who have hypothyroidism often have delayed bone age, which may be used as an
indicator of how long the hypothyroidism has been present. Although a bone age radiograph
might be part of the extended evaluation of hypothyroidism, the diagnosis first must be
established by measuring FT4.
Most children who have acquired primary hypothyroidism have chronic lymphocytic
thyroiditis, with measurable serum antibodies directed against the thyroperoxidase enzyme, the
thyroid microsomes that contain these antibodies, or thyroglobulin. In addition, they often have
palpable thyroid glands. If they do not have positive antithyroid antibodies, thyroid
ultrasonography can aid in assessing the shape and position of the thyroid gland. Some affected
children could have congenitally abnormal thyroid glands with maldescent but enough thyroid
hormone production not to become hypothyroid until early childhood. Assessment using
technetium or radioactive iodine imaging also might be appropriate.
References:
Hunter I, Greene SA, MacDonald TM, Morris AD. Prevalence and aetiology of hypothyroidism in
the young. Arch Dis Child. 2000;83:207-210. Abstract available at:
http://www.ncbi.nlm.nih.gov/pubmed/10952634
LaFranchi S. Acquired hypothyroidism in childhood and adolescence. UpToDate Online 15.3.
2008. Available for subscription at:
http://www.uptodateonline.com/utd/content/topic.do?topicKey=pediendo/4633
Ferry RJ Jr, Bauer AJ. Hypothyroidism. eMedicine Specialties, Pediatrics, Endocrinology. 2006.
Available at: http://www.emedicine.com/ped/TOPIC1141.HTM
page 159
An 8-year-old boy has difficulty in academics and a short attention span. His father states that
he had the same problems when he was a child. Physical examination reveals macrocephaly,
multiple caf au lait macules (Item Q75A) and axillary freckles (Item Q75B). Upon questioning,
the father explains that he has similar skin findings.
Of the following, the MOST likely diagnosis is
A. fragile X syndrome
B. hypomelanosis of Ito
C. neurofibromatosis type 1
D. tuberous sclerosis
E. velocardiofacial syndrome
page 160
Preferred Response: C
The frequency of learning difficulties is increased in individuals who have a positive family history
for learning disorders or genetic syndromes such as neurofibromatosis type 1 (NF-1), fragile X
syndrome, or velocardiofacial syndrome (VCFS). It is important to obtain a family history when
evaluating children who have suspected learning disorders to ascertain if there is a possible
genetic cause. A thorough family history also may identify speech or language problems in up to
30% of first-degree relatives of children who have language delays, a risk factor for learning
difficulties.
The multiple caf au lait macules and axillary freckling described for the boy in the vignette
meet diagnostic criteria for NF-1. The offspring of an affected individual has a 50% risk of
inheriting the altered NF1 gene, which can be detected by molecular genetic testing, although
this rarely is necessary for diagnosis. Learning disabilities occur with increased frequency in
those who have a family history of NF-1, ranging from 40% to 60%.
Fragile X syndrome, the most common genetic cause of intellectual disabilities, occurs in 1
in 1,200 males and 1 in 2,500 females. A mutation in the FMR1 gene (Xq27.3) leads to
expansion of the CGG trinucleotide repeat. Women who have alleles in the range of 29 to 200
CGG repeats are at risk for giving birth to affected children. Males who have fragile X syndrome
have large heads, long faces, large ears, and macro-orchidism but no skin findings. Males who
have the full mutation have mean intelligence quotient (IQ) scores of 40; affected females have
mean IQ scores in the low-average to borderline range.
Velocardiofacial syndrome affects 1 in 400 individuals, and 10% of cases are inherited in an
autosomal dominant pattern. The condition is caused by a microdeletion on chromosome
22q11.2 and may be associated with cardiac and facial anomalies, but not cutaneous
abnormalities. Learning difficulties are reported in 82% to 100% of affected children.
Hypomelanosis of Ito is a congenital skin disorder that is characterized by linear or whorled
hypopigmentation that follows the lines of Blaschko, the paths of embryonic neural crest cell
migration (Item C75A). There is no evidence for genetic transmission. Associated intellectual
disability has been reported in as many as 70% of cases, seizures in 40%, and microcephaly in
25%. However, these high percentages may be the result of selection bias, and the actual
frequency of associated abnormalities may be lower.
Tuberous sclerosis complex (TSC) is inherited in an autosomal dominant pattern, but as
many as two thirds of cases represent new mutations. TSC may present during infancy with
infantile spasms and a hypsarrhythmic electroencephalographic pattern. Autism is seen 1% of
affected individuals. The most common skin finding is hypopigmented macules that have been
likened to an ash leaf; they are seen in more than 90% of affected individuals (Item C75B).
References:
Haslam RHA. Neurocutaneous syndromes. In: Kliegman RM, Behrman RE, Jenson HB, Stanton
BF, eds. Nelson Textbook of Pediatrics. 18th ed. Philadelphia, Pa: Saunders Elsevier; 2007:24832488
Kates WR, Antshel KM, Femont W, Roizen, NJ, Shprintzen RJ. Velocardiofacial syndrome. In:
Accardo PJ, ed. Capute & Accardo's Neurodevelopmental Disabilities in Infancy and Childhood.
Volume II: The Spectrum of Neurodevelomental Disabilities. 3rd ed. Baltimore, Md: Paul H.
Brookes Publishing Co; 2008:363-373
Lyon GR, Shaywitz SE, Shaywitz BA. Dyslexia. In: Kliegman RM, Behrman RE, Jenson HB,
Stanton BF, eds. Nelson Textbook of Pediatrics. 18th ed. Philadelphia, Pa: Saunders Elsevier;
2007:150-151
Morelli JG. Hyperpigmented lesions. In: Kliegman RM, Behrman RE, Jenson HB, Stanton BF,
eds. Nelson Textbook of Pediatrics. 18th ed. Philadelphia, Pa: Saunders Elsevier; 2007:26822685
page 161
page 162
A 6-year-old girl presents with a history of swelling on her jaw of 1 months duration. The mother
has been to a "couple of emergency rooms," but nobody can tell her what is wrong with the girl.
The childs father died about 3 years ago from pneumonia, and the mother reports that she has
"no energy," but she has not sought medical care. The mother states that her daughter has
been fairly healthy except for frequent ear infections. On physical examination, the girl is afebrile;
her weight is 16 kg (3rd percentile); her height is 105 cm (3rd percentile); and she has scarred
tympanic membranes, bilateral parotid swelling, mild clubbing, and some fine crackles on lung
examination.
Of the following, the MOST likely diagnosis is
A. bacterial parotitis
B. common variable immunodeficiency
C. human immunodeficiency virus infection
D. lymphoma
E. mumps
page 163
Preferred Response: C
The chronic parotid swelling, clubbing of the digits, history of chronic otitis media, and paternal
and maternal illness described for the girl in the vignette are most characteristic of human
immunodeficiency virus (HIV) infection. Among HIV-infected children, digital clubbing and
wheezing are due to a chronic lung disease known as lymphocytic interstitial pneumonitis.
Patients who have bacterial parotitis usually are febrile and have purulent material coming
from Stensen's duct (Item C76A). Although patients who have HIV infection may have
lymphoma, the presentation usually is not bilateral and is much less common than chronic
parotitis. Sinopulmonary infections can occur with common variable immunodeficiency (CVID),
but HIV is more common and also is more likely to affect both parents, unlike CVID.
Mumps can present with parotid swelling (Item C76B) but is an uncommon infection in
developed countries, and the duration of the swelling is not long. The parotid swelling associated
with mumps usually peaks in 3 days and resolves over the next 7 days. If mumps is being
considered, the diagnosis can be confirmed by isolation of the virus in cell culture from throat
washings, saliva, urine, or cerebrospinal fluid. In addition, a positive mumps immunoglobulin M
titer, mumps polymerase chain reaction test, or a significant increase between acute and
convalescent serum mumps titers can help establish the diagnosis.
References:
American Academy of Pediatrics. Mumps. In: Pickering LK, Baker CJ, Long SS, McMillan JA,
eds. Red Book: 2006 Report of the Committee on Infectious Diseases. 27th ed. Elk Grove
Village, Ill: American Academy of Pediatrics; 2006:464-468
Burchett SK, Pizzo PA. HIV infection in infants, children, and adolescents. Pediatr Rev.
2003;24:186-194. Available at: http://pedsinreview.aappublications.org/cgi/content/full/24/6/186
page 164
You are evaluating a 2-year-girl who recently was adopted from Russia for a 4-day history of
temperature to 102.5F (39.2C), rash, coryza, malaise, conjunctivitis, and cough that have
worsened over the last 24 hours. She had nasal congestion and rhinorrhea for 5 days prior to
developing the fever, rash, and cough. The girl has been in the United States for 7 days. She
was adopted from a rural orphanage, where she was exposed to farm animals, but information
regarding her past medical history and immunizations is unavailable. Physical examination
shows a tired-appearing, irritable toddler who is clinging to her adopted mother. She has a
temperature of 103.0F (39.5C), bilateral conjunctival injection, profuse clear rhinorrhea, an
erythematous buccal mucosa with scattered whitish specks (Item Q77A) on the left side, and an
erythematous posterior pharynx with no tonsillar exudates. There is a confluent erythematous
maculopapular rash on her face, trunk, and abdomen (Item Q77B), with scattered patches on
her legs.
Of the following, the test MOST likely to confirm the diagnosis for this child is
A. blood culture
B. C-reactive protein measurement
C. serology
D. throat culture
E. urine culture
page 165
Preferred Response: C
The patient described in the vignette has history and physical examination findings that are
classic for measles. Measles is caused by the measles (rubeola) virus, which is an RNA virus in
the Paramyxoviridae family. The simplest and preferred method for establishing the diagnosis is
by demonstrating a positive serologic test for measles immunoglobulin M (IgM) antibody on a
single serum specimen obtained during the first encounter with a person suspected of having
the disease. Measles IgM antibody is detectable for at least 1 month after the onset of the rash.
The measles vaccine alone does not give an IgM response. Other methods of diagnosis include
detection of measles virus antigen in respiratory epithelial cells or tissue by immunofluorescent
methods and detection of viral genome by polymerase chain reaction. Isolation of the measles
virus from blood, throat, and urine cultures is possible but is very difficult and is not readily
available. C-reactive protein is a nonspecific measurement of inflammation that is not specific for
the diagnosis of measles.
References:
American Academy of Pediatrics. Measles. In: Pickering LK, Baker CJ, Long SS, McMillan JA,
eds. Red Book: 2006 Report of the Committee on Infectious Diseases. 27th ed. Elk Grove
Village, Ill: American Academy of Pediatrics; 2006:441-452
Centers for Disease Control and Prevention. Measles, mumps, and rubella - vaccine use and
strategies for elimination of measles, rubella, and congenital rubella syndrome and control of
mumps: recommendations of the Advisory Committee on Immunization Practices (ACIP).
MMWR Recomm Rep. 1998;47(RR-8):1-57. Available at:
http://www.cdc.gov/mmwr/preview/mmwrhtml/00053391.htm
Maldonado YA. Rubeola virus (measles and subacute sclerosing panencephalitis). In: Long SS,
Pickering LK, Prober CG, eds. Principles and Practice of Pediatric Infectious Diseases. 2nd ed.
Philadelphia, Pa: Churchill Livingstone; 2003:1148-1155
page 166
You employ voiding cystourethrography (VCUG) to evaluate a 4-year-old girl who had a febrile
urinary tract infection 1 month ago. The study reveals a smooth-walled bladder, absence of
vesicoureteral reflux, and a mildly narrowed urethra.
Of the following, the MOST appropriate management in this situation is
page 167
Preferred Response: B
The patient described in the vignette has had a febrile urinary tract infection (UTI), but voiding
cystourethrography (VCUG) failed to demonstrate vesicoureteral reflux. It did, however,
demonstrate a narrow urethra with an otherwise normal bladder. The question that arises is
whether the "narrow urethra" is associated with potential voiding dysfunction and development of
the UTI.
The recognition of a distal urethral ring in some females who had UTIs in the 1960s led to
the hypothesis that a narrowed urethra or "urethral stenosis" was responsible for UTIs.
However, it is now recognized that the previously termed "narrow urethra" in females is a normal
variant. Indeed, dilatations led to more invasive surgeries of the internal urethra to prevent
"restenosis." A recent survey of Fellows in the Section of Urology of the American Academy of
Pediatrics found that 61% of respondents never use dilatation under any circumstance, and only
2% perform this procedure regularly. Therefore, no treatment is needed for the child in the
vignette.
Children who have recurrent UTIs may have vesicoureteral reflux or dysfunctional voiding
with incomplete bladder emptying. More recently, greater emphasis has been placed on the
effects of pelvic floor muscle dysfunction in the child who has dysfunctional voiding and
symptoms of enuresis, UTI, and constipation. It is theorized that the pelvic floor muscles
responsible for continence in such children are hyperactive, resulting in abnormal bladder
emptying and high postvoid residual volumes. Biofeedback is being used to teach children how
to improve relaxation of pelvic floor muscles and reduce urinary retention and UTI risk.
There is no indication for prophylactic antibiotics for a child who has a single UTI and no
vesicoureteral reflux. Only children who have significant urinary retention, such as seen with
neurogenic bladder (myelomeningocele) or detrusor sphincter dyssynergia (Hinman syndrome),
warrant intermittent catheterization. Follow-up VCUG is not needed for the patient in the vignette.
Cystoscopy is not indicated in a child who has a smooth wall bladder and no urinary symptoms.
References:
Brock WA, Kaplan GW. Abnormalities of the lower urinary tract. In: Edelmann CM Jr, Bernstein
J, Meadow SR, Spitzer A, Travis LB, eds. Pediatric Kidney Disease. 2nd ed. Boston, Ma: Little,
Brown, and Company; 1992:2037-2076
McKenna PH, Herndon CD, Connery S, Ferrer FA. Pelvic floor muscle retraining for pediatric
voiding dysfunction using interactive computer games. J Urol. 1999;162:1056-1063. Abstract
available at: http://www.ncbi.nlm.nih.gov/pubmed/10458431
Metwalli AR, Cheng EY, Kropp BP, Pope JC 4th. The practice of urethral dilation for voiding
dysfunction among fellows of the Section on Urology of the American Academy of Pediatrics. J
Urol. 2002;168:1764-1767. Abstract available at: http://www.ncbi.nlm.nih.gov/pubmed/12352355
page 168
An 18-year-old girl is admitted to the hospital for intravenous therapy for a complicated urinary
tract infection that failed to respond to outpatient therapy with a sulfa-based antibiotic. Her urine
culture shows more than 100,000 colony-forming units/mL of Pseudomonas aeruginosa that is
sensitive to aztreonam and imipenem. As you take her medical history, she mentions she is
"highly allergic" to penicillin.
Of the following, a TRUE statement regarding penicillin drug reactions is that
A. a nonpruritic maculopapular rash that occurs in patients who receive amoxicillin during
mononucleosis is a contraindication for future penicillin therapy
C. desensitization can be used to administer penicillin safely to patients who have experienced
Stevens-Johnson reactions to penicillin
D. skin testing to major and minor determinants of penicillin can exclude IgE-mediated and nonIgE-mediated reactions
E. a patient who can only recall a childhood history of penicillin allergy but does not remember the
details is very likely to react to future penicillin courses
page 169
Preferred Response: B
Penicillin is the most common cause of drug-induced anaphylaxis. Because the likelihood of an
immunoglobulin (Ig) E-mediated penicillin allergy is less than 20% based on history alone,
patients often are referred to an allergist-immunologist for evaluation and skin testing. The
negative predictive value of penicillin skin testing for IgE-mediated reactions is 97% to 99%.
However, standardized skin testing materials such as benzylpenicilloyl polylysine or minor
determinants either no longer are available or are not approved by the United States Food and
Drug Administration, respectively. In addition, non-IgE-mediated penicillin reactions, such as
Stevens-Johnson syndrome (Item C79A), result in negative IgE skin tests, even though drug
administration causes adverse effects.
Because appropriate penicillin skin testing material is not available in most allergy clinics,
administration of a non-beta-lactam antibiotic is advised for the patient who has a history of
suspected IgE-mediated penicillin allergy. If a penicillin, cephalosporin, or carbapenem drug is
required, consultation with an allergist-immunologist should be considered. A detailed discussion
regarding avoidance, graded challenge, or drug desensitization usually ensues from the
consultation. Carbapenems have a high cross-reactivity with penicillin and should be avoided,
but monobactams such as aztreonam do not cross-react and may be administered safely.
Desensitization is a process that allows the gradual administration of a drug to a patient who
has an IgE-mediated drug allergy. Thus, desensitization cannot be successful for a patient who
experienced a non-IgE-mediated reaction such as Stevens-Johnson syndrome. Also, once the
desensitization is complete and the patient finishes the appropriate antibiotic therapy, he or she
still is considered "allergic" to that drug and must undergo repeat desensitizations to receive
subsequent courses of that antibiotic.
One common non-IgE-mediated reaction to penicillin occurs when an antibiotic such as
amoxicillin is administered to a patient who has an Epstein-Barr virus infection (eg, infectious
mononucleosis). Typical symptoms include a nonpruritic, maculopapular rash that occurs a few
days into therapy (Item C79B). Clinically, it may be difficult to distinguish this reaction from a true
drug allergy, but this rash is not due to an allergic reaction, and future drug use need not be
restricted.
References:
Boguniewicz M, Leung DYM. Adverse reactions to drugs. In: Kleigman RM, Behrman RE,
Jenson HB, Stanton BF, eds. Nelson Textbook of Pediatrics. 18th ed. Philadelphia, Pa:
Saunders Elsevier; 2007:990-994
Wolf R, Orion E, Marcos B, Matz H. Life-threatening acute adverse cutaneous drug reactions.
Clin Dermatol. 2005;23:171-181. Abstract available at:
http://www.ncbi.nlm.nih.gov/pubmed/15802211
page 170
A 5-year-old boy is brought to the emergency department because of a nose bleed that has
lasted 1} hours. His mother reports that he has had nose bleeds in the past that usually
stopped when she pinched his nose, but this time he continued to bleed. She says that he does
pick his nose and that he has had cold symptoms for the past 3 days. There is no family history
of bleeding disorders, and he had no excessive bleeding after circumcision. On physical
examination, the awake, alert, and anxious patient is holding a bloody washcloth to his nose. His
heart rate is 140 beats/min, respiratory rate is 24 breaths/min, blood pressure is 100/60 mm Hg,
and oxygen saturation is 98%. There is active bleeding from his right naris, but an active anterior
bleeding site is not visible. Bleeding is controlled with phenylephrine instillation and packing.
Of the following, the MOST appropriate further evaluation is
A. chest radiograph
B. computed tomography scan of the sinuses
C. magnetic resonance arteriography of the sinuses
D. nasopharyngoscopy
E. no further evaluation
page 171
Preferred Response: D
Nosebleeds are common in children, most often associated with trauma (eg, nose-picking),
mucosal friability due to upper respiratory tract infections, and mucosal drying related to
environmental conditions. Most epistaxis episodes are self-limited and require only bleeding
control instructions and comfort care. Recurrent nosebleeds are seen in fewer than 10% of
patients and only rarely are related to an underlying anatomic or hematologic abnormality.
Evidence suggests that antibacterial creams instilled in the affected naris may decrease the
incidence of recurrent idiopathic epistaxis as the child "outgrows" the predisposition.
Patients who have nosebleeds that recur frequently or are difficult to control or localize,
have a family history suggestive of a bleeding disorder, or have signs and symptoms suggestive
of a blood dyscrasia (eg, petechiae) may require further evaluation for an underlying cause.
Evaluation should be directed at likely causes, including vascular anomalies, foreign bodies,
benign and malignant masses, and primary or secondary bleeding diatheses such as
thrombocytopenia, von Willebrand disease, platelet dysfunction, hemophilias, leukemia, liver
disease, or those related to medications.
The history and physical examination should guide further evaluation. The inability to localize
the bleeding to the anterior vestibule for the boy described in the vignette suggests bleeding from
the posterior nasopharynx. Ninety percent of epistaxis events result from injury to the
Kiesselbach vascular plexus in the anterior nasal septum, and the bleeding site should be visible
on nasal inspection. Bleeding from the anterior or posterior ethmoidal or sphenopalatine arteries
in the posterior nasopharynx is more difficult to see and control. Therefore, nasopharyngoscopy
is indicated to identify the site and to aid in directing treatment. If a mass or vascular anomaly is
identified, computed tomography scan or magnetic resonance angiography/magnetic resonance
venography should be undertaken. A chest radiograph is not likely to provide information that
would guide evaluation or treatment of epistaxis.
When a bleeding disorder is suspected, the initial hematologic evaluation should include
complete blood and platelet counts, with evaluation of the blood smear, prothrombin time, and
partial thromboplastin time and typing and cross-matching if transfusion is anticipated. If
abnormalities are identified, consultation with a hematologist can help to guide further evaluation,
which may include a closure test to evaluate for platelet dysfunction and von Willebrand factor or
ristocetin cofactor assays to assess for von Willebrand disease. Of note, in one study of
patients who had recurrent epistaxis and were referred to a hematologist, one third ultimately
were diagnosed with bleeding disorders, and 50% of these patients had von Willebrand disease.
Family history was the only predictor that identified patients who had primary bleeding diatheses.
References:
Haddad J Jr. Acquired disorders of the nose. In: Kleigman RM, Behrman RE, Jenson HB,
Stanton BF, eds. Nelson Textbook of Pediatrics. 18th ed. Philadelphia, Pa: Saunders Elsevier;
2007:1744-1745
McGarry G. Nosebleeds in children. BMJ Clinical Evidence. 2006. Available for subscription at:
http://clinicalevidence.bmj.com/ceweb/conditions/chd/0311/0311.jsp
Messner AH. Epidemiology and etiology of epistaxis in children. UpToDate Online 15.3. 2008.
Available for subscription at: http://www.utdol.com/utd/content/topic.do?topicKey=ped_lryn/5986
Messner AH. Evaluation of epistaxis in children. UpToDate Online 15.3. 2008. Available for
subscription at:
http://www.utdol.com/utd/content/topic.do?topicKey=ped_lryn/6248&selectedTitle=4~150&sourc
e=search_result
Sandoval C, Dong S, Visintainer P, Ozkaynak MF, Jayabose S. Clinical and laboratory features
of 178 children with recurrent epistaxis. J Pediatr Hematol Oncol. 2002;24:47-49. Abstract
available at: http://www.ncbi.nlm.nih.gov/pubmed/11902740
page 172
A 4-month-old infant who has gastroschisis underwent surgical repair on the first day after birth,
but continues to require support with parenteral nutrition and lipids. He now has developed poor
feeding, irritability, and progressive diarrhea. Radiography demonstrates metaphyseal fraying,
but calcium, phosphorus, and 25-hydroxyvitamin D concentrations are normal. When you
review his prior laboratory studies, you note he has had neutropenia for the past 4 weeks.
Of the following, this childs symptoms are MOST consistent with
A. copper deficiency
B. magnesium deficiency
C. vitamin A deficiency
D. vitamin B6 deficiency
E. zinc deficiency
page 173
Preferred Response: A
The poor feeding, irritability, metaphyseal irregularity, normal vitamin D value, and neutropenia
described for the infant in the vignette are consistent with copper deficiency, a rare disorder that
develops when children receive parenteral nutrition without trace element supplementation.
Copper is a trace element that has many important biochemical functions. It is an important
component of respiratory chain enzymes (eg, cytochrome C) and lysyl oxidase (an enzyme
critical to collagen production and bone formation). Menkes disease, a rare syndrome involving
a defect in copper transport and profoundly low serum copper concentrations, is characterized
by hypotonia, developmental delay, seizures, and "steely hair." Copper deficiency also may be
seen in preterm infants who do not receive appropriate trace element supplementation. The
clinical presentation of copper deficiency in the preterm infant is more subtle, but can include
pallor, poor feeding, hypochromic anemia, neutropenia, and skeletal changes (including
metaphyseal fraying and osteoporosis). The patient's symptoms are more consistent with
copper deficiency than with magnesium, vitamin B6, zinc, or vitamin A deficiency. Specifically,
magnesium deficiency can cause hypotonia and apnea, vitamin B6 deficiency can cause
hypotonia and seizures, zinc deficiency can cause diarrhea and skin rashes, and vitamin A
deficiency can lead to corneal lesions and impaired vision.
To prevent copper deficiency, most parenteral nutrition is supplemented with 200 mcg/L of
copper. Infants who have cholestasis may have impaired copper excretion into the bile,
necessitating a decrease in the concentration of copper in the parenteral nutrition to prevent
copper overload.
References:
Collier S, Gura KM, Richardson D, Duggan C. Parenteral nutrition. In: Hendricks KM, Duggan C.
Manual of Pediatric Nutrition. 4th ed. Hamilton Ontario, Canada: BC Decker; 2005:317-375
Giles E, Doyle LW. Copper in extremely low-birthweight or very preterm infants. NeoReviews.
2007;8:e159-e164. Available for subscription at:
http://neoreviews.aappublications.org/cgi/content/full/8/4/e159
page 174
You are counseling a 23-year-old woman who has diabetes mellitus and has been your patient
for the past 18 years. She recently found out that she is pregnant and asks you about potential
complications for her unborn child.
Of the following, the MOST likely complications to expect for this womans child are
page 175
Preferred Response: D
page 176
A 17-year-old girl complains of an itchy rash all over her back and trunk for 2 weeks. Topical
hydrocortisone has not relieved the rash or itching. She denies fever or other symptoms, and
her vital signs are normal. Examination of the skin reveals multiple 5- to 8-mm salmon-colored
thin scaling plaques over her trunk (Item Q83). There is one similar lesion on her abdomen that
measures 2x3 cm. There are no other lesions, and the remaining findings of her physical
examination are normal.
Of the following, the BEST approach to managing this girls rash is to
page 177
Preferred Response: E
The rash described for the girl in the vignette is most consistent with the diagnosis of pityriasis
rosea, a self-limited condition believed to have a viral etiology, although this remains unproven. A
nonspecific prodrome of fever and malaise is seen occasionally. Typical features include a
salmon-colored patch surrounded by a darker rim and a fine scale, known as a "herald patch"
(Item C83A). Several days after the appearance of the herald patch, multiple similar but smaller
lesions develop over the trunk and back along lines of skin cleavage, giving the impression of a
"Christmas tree" pattern (Item C83B). The palms and soles typically are spared. Some pruritus
occurs in approximately 25% of patients. The rash is generally present for 2 to 12 weeks and
resolves spontaneously. Exposure to sunlight may hasten resolution of the lesions.
Numerous therapies have been prescribed for the treatment of pityriasis rosea, including
oral antibiotics, oral antihistamines, and oral steroids, but most have not proven to be effective.
Therefore, reassurance is all that is necessary. Topical antipruritic agents may relieve itching in
some cases. Coal tar preparations are useful in the treatment of psoriasis and other chronic
dermatoses. Topical selenium sulfide shampoo may be helpful for tinea versicolor and
seborrheic dermatitis, but these conditions are not suggested by the appearance of the girl's
rash.
One of the most important diagnoses to consider in the differential diagnosis of pityriasis
rosea is secondary syphilis. The skin lesions can look similar to those of pityriasis rosea and
may follow the same lines of distribution, but the palms and soles generally are affected (Item
C83C). Patients also may complain of vague constitutional symptoms, such as headaches,
fatigue, and lymph node swelling. It is critical to obtain an accurate sexual history in any
adolescent who presents with this type of rash. Intramuscular penicillin is the drug of choice for
secondary syphilis.
References:
Chuh AAT, Dofitas BL, Comisel GG, et al. Interventions for pityriasis rosea. Cochrane Database
Syst Rev. 2007;2:CD005068. Available at:
http://www.mrw.interscience.wiley.com/cochrane/clsysrev/articles/CD005068/frame.html
Morelli JG. Diseases of the epidermis. In: Kliegman RM, Behrman RE, Jenson HB, Stanton BF,
eds. Nelson Textbook of Pediatrics. 18th ed. Philadelphia, Pa: Saunders Elsevier; 2007:27022707
Wolfrey JD, Billica WH, Gulbranson SH, et al. Pediatric exanthems. Clin Fam Pract. 2003;5:557588
page 178
A 4-year-old boy who has had mild eczema in the past that was treated successfully with
emollients presents with the worst exacerbation he ever has had. He has multiple lichenified
lesions, especially in the antecubital fossae (Item Q84) and popliteal fossa, which is usual for
him, but he also has nummular lesions on the trunk. His mother reports no changes in
detergents or personal hygiene products. The boy has been going to a summer day camp at the
local community center for the first time.
Of the following, the factor MOST likely involved in his eczema exacerbation is
page 179
Preferred Response: D
A number of factors may worsen atopic dermatitis by causing itching and scratching. Exposure
to heat (as may occur during warm weather months or because of overdressing during cold
weather months), sunlight, chemicals (including swimming pool chemicals such as chlorine), and
sweat retention cause exacerbations. Cold weather and low humidity may cause skin to become
dry, leading to increased pruritus. Wool or synthetic materials, fragrances, harsh soaps or
detergents, and some fabric softeners also may cause itching.
For the child described in the vignette, overheating during outdoor play is the most likely
activity that is exacerbating his eczema. Activities in an air-conditioned gymnasium would be
beneficial. Although the use of glues and craft materials may cause irritation, they would be
anticipated to produce a rash on the hands at sites of exposure. The snacks the child is
receiving are unlikely to worsen atopic dermatitis. Food allergy as an exacerbating factor is
observed most often in infants, not older children. The allergens most commonly implicated are
milk, eggs, soy, wheat, and peanuts.
References:
Ashcroft DM, Chen L-C, Garside R, Stein K, Williams HC. Topical pimecrolimus for eczema.
Cochrane Database Syst Rev. 2007;4:CD005500. Available at:
http://www.mrw.interscience.wiley.com/cochrane/clsysrev/articles/CD005500/frame.html
Bath-Hextall F, Williams H. Eczema (atopic). BMJ Clinical Evidence. 2006. Available for
subscription at: http://clinicalevidence.bmj.com/ceweb/conditions/skd/1716/1716_I15.jsp
Beattie PE, Lewis-Jones MS. A comparative study of impairment of quality of life in children with
skin disease and children with other chronic childhood diseases. Br J Dermatol. 2006;155:145151. Abstract available at: http://www.ncbi.nlm.nih.gov/pubmed/16792766
Byremo G, Rd G, Carlsen KH. Effect of climatic change in children with atopic eczema. Allergy.
2006;61:1403-1410. Abstract available at: http://www.ncbi.nlm.nih.gov/pubmed/17073869
Ersser SJ, Latter S, Sibley A, Satherley PA, Welbourne S. Psychological and educational
interventions for atopic eczema in children. Cochrane Database Syst Rev. 2007;3:CD004054.
Available at:
http://www.mrw.interscience.wiley.com/cochrane/clsysrev/articles/CD004054/frame.html
Kramer MS, Kakuma R. Maternal dietary antigen avoidance during pregnancy or lactation, or
both, for preventing or treating atopic disease in the child. Cochrane Database Syst Rev.
2006;3:CD000133. Available at:
http://www.mrw.interscience.wiley.com/cochrane/clsysrev/articles/CD000133/frame.html
Osborn DA, Sinn J. Formulas containing hydrolysed protein for prevention of allergy and food
intolerance in infants. Cochrane Database Syst Rev. 2003;4:CD003664. Available at:
http://www.mrw.interscience.wiley.com/cochrane/clsysrev/articles/CD003664/frame.html
Osborn DA, Sinn JK. Probiotics in infants for prevention of allergic disease and food
hypersensitivity. Cochrane Database Syst Rev. 2007;4:CD006475. Available at:
http://www.mrw.interscience.wiley.com/cochrane/clsysrev/articles/CD006475/frame.html
Osborn DA, Sinn J. Soy formula for prevention of allergy and food intolerance in infants.
Cochrane Database Syst Rev. 2006;4:CD003741. Available at:
http://www.mrw.interscience.wiley.com/cochrane/clsysrev/articles/CD003741/frame.html
page 180
You are called by the mother of 3-year-old girl because the child appears confused and is pale
and sweating. The mother thinks the child may have taken some of her grandmothers
imipramine. You advise her to contact emergency medical services for immediate transport to
the emergency department, where you plan to meet them.
Of the following, the MOST appropriate action to take in the emergency department is
page 181
Preferred Response: B
The child described in the vignette has symptoms suggestive of tricyclic antidepressant (TCA)
ingestion. This class of antidepressants is used less frequently today for treatment of
depression because of the availability of selective serotonin reuptake inhibitors (SSRIs). TCAs
are not the drugs of choice for treatment of depression in children and adolescents but are used
occasionally in the treatment of other disorders (eg, enuresis, narcolepsy). SSRIs are much
safer to use than TCAs because they have fewer adverse effects and are unlikely to result in
death when overdose occurs.
When TCAs are ingested in toxic amounts, they primarily affect the central nervous and
cardiovascular systems. Central nervous system signs and symptoms of TCA toxicity include
irritability, euphoria, seizures, and unresponsiveness. Autonomic nervous system symptoms
such as mydriasis, dry skin, dry mouth, urinary retention, and tachycardia also may be evident.
Among the direct effects on the cardiac system are a delay in signal conduction through the
bundle of His, depression of myocardial contractile function, and prolongation of the QRS and
the QT intervals. These latter cardiac effects may potentiate arrhythmia formation.
Continuous cardiac monitoring for the possible occurrence of arrhythmia is essential in
patients who are suspected of toxic TCA ingestion. Electrocardiography is the best tool for
assessing the function of the conduction system. The voltage intervals should be measured,
with particular attention to the QRS duration and the QT interval. A QRS duration of greater than
100 msec is associated with the development of seizures; a QRS duration of more than 160
msec is associated with ventricular dysrhythmias that may be particularly difficult to treat.
Echocardiography is an excellent tool to evaluate cardiac structure and function but is not
necessarily indicated as part of the emergency department evaluation of a child in whom toxic
TCA ingestion is suspected and does not obviate the need for continuous cardiac monitoring.
Similarly, chest radiography does not play a role in the acute management of such a patient.
Electroencephalography has no place in the initial evaluation and management of children in
whom a TCA toxic ingestion is suspected. Seizures, when they occur, usually do so early in the
course and often resolve by the time anticonvulsants are administered. Measuring serum drug
concentrations of TCAs is not helpful for the prognosis or management of TCA ingestion. Serum
electrolyte concentrations may be measured, but they have no predictive value in the
management of TCA toxicity.
References:
Boehnert MT, Lovejoy FH Jr. Value of the QRS duration versus the serum drug level in
predicting seizures and ventricular arrhythmias after an acute overdose of tricyclic
antidepressants. N Engl J Med. 1985;313:474-479. Abstract available at:
http://www.ncbi.nlm.nih.gov/pubmed/4022081
Hatcher-Kay C, King CA. Depression and suicide. Pediatr Rev. 2003;24:363-371. Available at:
http://pedsinreview.aappublications.org/cgi/content/full/24/11/363
Liebelt EL, Francis PD, Woolf AD. ECG lead aVR versus QRS interval in predicting seizures and
arrhythmias in acute tricyclic antidepressant toxicity. Ann Emerg Med. 1995;26:195-201.
Abstract available at: http://www.ncbi.nlm.nih.gov/pubmed/7618783
McGuigan ME. Poisoning potpourri. Pediatr Rev. 2001;22:295-302. Available at:
http://pedsinreview.aappublications.org/cgi/content/full/22/9/295
Prez-Fontn J, Lister G. The acutely ill infant and child. In: Rudolph CD, Rudolph AM, eds.
Rudolph's Pediatrics. 21st ed. New York, NY: McGraw-Hill Medical Publishing Division; 2003:364365
page 182
A 4-year-old boy who has neuroblastoma presents with back pain and an inability to urinate. He
is alert, with normal general examination findings and normal mental status. Strength and tone in
the arms are normal, but tone is low in the legs, and patellar reflexes are diminished.
Of the following, the MOST appropriate next step for diagnosis is
A. lumbar puncture
B. magnetic resonance imaging with contrast of the spine
C. postvoid bladder residual measurement
D. radiograph of the spine
E. voiding cystourethrography
page 183
Preferred Response: B
Back pain in a young child is unusual and requires prompt evaluation. When there are additional
signs that can localize to the spinal cord, such as inability to urinate, low tone, and diminished
reflexes, as described for the boy in the vignette, immediate evaluation in the emergency
department is needed. Lesions within or near the spinal cord that are causing symptoms can
progress to irreversible damage that may be avoided with emergency neurosurgery.
The child described in the vignette has a history of neuroblastoma, an aggressive tumor that
is prone to metastasis, and emerging flaccid paralysis. Therefore, magnetic resonance imaging
(MRI) of the spine with gadolinium contrast is indicated to assess the nature of the problem and
determine the location of the lesion more exactly (Item C86). Hypervascularity or vascular wall
breakdown in a metastasis may result in contrast enhancement, improving visualization of the
lesion.
Lumbar puncture may be needed subsequently to evaluate for other causes or for tumor
cytology, but this procedure should be deferred until after imaging the spine and, if necessary,
emergent neurosurgical consultation. Similarly, spine imaging takes precedence over urologic
dynamic studies such as postvoid bladder residual measurement or voiding
cystourethrography. A plain radiograph may provide some information, but if MRI is available, it
is preferred because it can assist in emergency surgical planning.
Management of spinal cord compression due to a tumor includes immediate neurosurgical
consultation. Intravenous dexamethasone typically is used, along with a proton pump inhibitor or
histamine2 blocker. Aggressive pain management may be needed.
References:
Haslam RHA. Spinal cord disorders. In: Kliegman RM, Behrman RE, Jenson HB, Stanton BF,
eds. Nelson Textbook of Pediatrics. 18th ed. Philadelphia, Pa: Saunders Elsevier; 2007:25262530
Kim S, Chung DH. Pediatric solid malignancies: neuroblastoma and Wilms' tumor. Surg Clin
North Am. 2006;86:469-487
page 184
The mother of a boy in your practice is contemplating another pregnancy and asks for your
advice. The woman is tall and thin and works as a model part-time. She had previously reported
to you a history of bulimia. She is extremely concerned about any "extra" weight she may gain
during the pregnancy, and she confides that she sometimes smokes cigarettes to avoid eating.
Additionally, she occasionally has taken her sons methylphenidate to suppress her appetite.
When asked about alcohol use, she describes herself as a "social drinker."
Of the following, the MOST accurate statement to make in counseling this woman is that
page 185
Preferred Response: B
The extent of the impact that environmental exposures have on pregnancy outcome is still
largely unknown. However, some large, well-designed epidemiologic studies offer insight into
complications that can arise from fetal exposure to maternal eating disorders, tobacco smoking,
and substances of abuse. Undoubtedly, genetic and other factors modify outcomes.
The impact of cigarette smoking on fetal development and pregnancy has been, and
continues to be, an active area of study. Intrauterine growth restriction is the most consistent
negative effect of maternal smoking on the fetus, with an average 200-g reduction in the
birthweights of term infants born to mothers who smoke during pregnancy. The more a woman
smokes, the greater the reduction in fetal weight.
Cigarette smoking during pregnancy also has been associated with increased health risks
to exposed children as they grow. Among the reported complications are abnormal pulmonary
function, increased cancer risk, and lower sperm counts. Numerous studies have found an
increased risk for sudden infant death in prenatally exposed infants.
Studies of the effects of eating disorders on pregnancy outcome are limited. However, both
prospective, controlled studies and literature reviews indicate that pregnant women who have
past or active eating disorders, including bulimia, are at increased risk for delivering babies of
significantly lower-than-average birthweight and head circumference (including microcephaly).
Prenatal exposure to alcohol may result in varied outcomes collectively termed fetal alcohol
spectrum disorder. To date, no safe quantity of alcohol has been established for consumption
during pregnancy. Therefore, the strong recommendation is that women abstain from drinking
alcoholic beverages from at least the time of conception and throughout pregnancy.
The effects of methylphenidate on pregnancy outcome have been studied in experimental
animals and a limited number of humans. To date, there does not appear to be an increase in
congenital anomalies associated with prenatal exposure to methylphenidate.
The question of the potential benefits of vitamin and micronutrient supplementation for
women who smoke cigarettes and drink alcohol during pregnancy and their babies is under
investigation. At present, it is not clear whether supplementation can mitigate poor outcomes,
and no specific recommendations exist. However, it is clear that good nutrition and vitamin
supplementation during pregnancy reduce the risk for pregnancy complications.
References:
Cigarette smoking, methamphetamine. Reprotox. Available for subscription at:
http://www.reprotox.org
Cigarette smoking, methamphetamine. Teris. Available for subscription at
http://depts.washington.edu/terisweb/teris/
Cogswell ME, Weisberg P, Spong C. Cigarette smoking, alcohol use and adverse pregnancy
outcomes: implications for micronutrient supplementation. J Nutr. 2003;133:1722S-1731S.
Available at: http://jn.nutrition.org/cgi/content/full/133/5/1722S
Kouba S, Hllstrm T, Londholm C, Lindn Hirschbe A. Pregnancy and neonatal outcomes in
women with eating disorders. Obstet Gynecol. 2005;105:255-260. Available at:
http://www.greenjournal.org/cgi/content/full/105/2/255
Kunz LH, King JC. Impact of maternal nutrition and metabolism on health of the offspring. Semin
Fetal Neonatal Med. 2007;12;71-77. Abstract available at:
http://www.ncbi.nlm.nih.gov/pubmed/17200031
Micali N, Simonoff E, Treasure J. Risk of major adverse outcomes in women with eating
disorders. Br J Psychiatry. 2007;190:255-259. Abstract available at:
http://www.ncbi.nlm.nih.gov/pubmed/17329747
page 186
An 18-year-old young man comes to your office with complaints of burning pain with urination
over the past 24 hours. He has seen a small amount of yellowish discharge from his penis
during this time. He also complains of some lower back pain over the past 48 hours. He denies
fever or rashes, but his eyes are a little irritated. He is sexually active and uses condoms "most
of the time." On physical examination, he is afebrile, his palpebral and bulbar conjunctivae are
mildly injected (Item Q88), and his back is tender at the lower lumbar area, but there is no
costovertebral angle tenderness. Genital examination reveals no scrotal tenderness and scant
yellow discharge at the urethral orifice.
Of the following, the MOST likely cause of this patients symptoms is
A. Chlamydia trachomatis
B. Gardnerella vaginalis
C. Neisseria gonorrhoeae
D. Treponema pallidum
E. Trichomonas vaginalis
page 187
Preferred Response: A
Urethritis can have infectious and noninfectious causes. Symptoms include mucopurulent or
purulent discharge, dysuria, and urethral pruritus. Several organisms, including Neisseria
gonorrhoeae and Chlamydia trachomatis, cause urethritis. Ureaplasma urealyticum,
Mycoplasma genitalium, Gardnerella vaginalis, herpes simplex virus, adenovirus, and
Trichomonas vaginalis are implicated in nonchlamydial, nongonococcal urethritis (NGU), but
they are more difficult to detect than N gonorrhoeae and C trachomatis.
The constellation of conjunctivitis, urethritis, and arthritis reported for the young man in the
vignette represents the classic symptoms of a form of reactive arthritis once called Reiter
syndrome. The term reactive arthritis refers to rheumatic disorders that appear after an
infection, but in which the responsible pathogen is not detected in the affected joint. C
trachomatis is the only genital pathogen commonly accepted to be a cause of reactive arthritis.
N gonorrhoeae can cause septic arthritis or disseminated gonococcal infection (ie, a rash and
tenosynovitis) but does not produce reactive arthritis. Although T vaginalis and G vaginalis may
cause urethritis, they do not produce the other symptoms exhibited by the boy described in the
vignette. Syphilis, caused by infection with Treponema pallidum, may affect bones congenitally
(osteochondritis) or in late stages of the disease (with gummas, granulomatous lesions that
involve bones as well as soft tissue or viscera) but does not produce urethral discharge or
conjunctivitis.
Reactive arthritis caused by C trachomatis is treated with a single 1-g oral dose of
azithromycin or with 100 mg doxycycline orally twice a day for 7 days, after testing for both N
gonorrhoeae and C trachomatis is completed. This is also the recommended regimen for all
NGUs. First-line treatment of uncomplicated gonococcal urethritis is accomplished with
ceftriaxone 125 mg intramuscularly or cefixime 400 mg orally, both in a single dose.
References:
Centers for Disease Control and Prevention. Update to CDC's sexually transmitted diseases
treatment guidelines, 2006: fluoroquinolones no longer recommended for treatment of
gonococcal infections. MMWR Morbid Mortal Wkly Rep. 2007;56:332-336. Available at:
http://www.cdc.gov/mmwr/preview/mmwrhtml/mm5614a3.htm
Fortenberry JD, Neinstein LS. Syphilis. In: Neinstein LS, ed. Adolescent Health Care: A Practical
Guide. 5th ed. Philadelphia, Pa: Lippincott Williams & Wilkins; 2008:825-833
Workowski KA, Berman SM, Centers for Disease Control and Prevention. Sexually transmitted
diseases treatment guidelines, 2006. MMWR Recomm Rep. 2006;55(RR11):1-94. Available at:
http://www.cdc.gov/mmwr/preview/mmwrhtml/rr5511a1.htm
Yu DT. Reactive arthritis (formerly Reiter syndrome): definition, diagnosis, and management.
UpToDate Online 15.3. 2008. Available for subscription at:
http://www.utdol.com/utd/content/topic.do?topicKey=spondylo/7349
page 188
You are evaluating an 18-month-old girl for vomiting. She has a history of febrile seizures and
recurrent ear infections. She is receiving no medications. Over the past several weeks, her
parents have noticed that she has been "increasingly clumsy." She has vomited each of the last
three mornings but has had no diarrhea or fever. Physical examination findings are normal
except for an ataxic gait and hyperreflexia.
Of the following, the MOST appropriate next step is
A. administration of an antiemetic
B. computed tomography scan of the head
C. electroencephalography
D. lumbar puncture
E. reassurance and re-evaluation in 3 to 5 days
page 189
Preferred Response: B
Initial symptoms of increased intracranial pressure often consist of headaches and confusion
that may be accompanied by lethargy. The child described in the vignette exhibits signs of a
progressive increase in pressure, such as vomiting (especially in the morning) and changes in
motor tone. Physical examination findings include a full or bulging fontanelle and widened
sutures. These signs, as well as pupillary changes and papilledema, should prompt rapid
evaluation to prevent permanent neurologic injury and progression of the increased pressure
with potential neurologic catastrophe.
Intracranial pressure is maintained by the balance of the contents of the cranial vault, which
includes brain, blood, and cerebrospinal fluid. Increases in intracranial pressure can occur with a
wide variety of disease processes, such as brain tumors, hydrocephalus, infections, head
trauma, and hypoxic-ischemic injury. Intracranial pressure increases when the compensatory
mechanisms of the cranial vault are exceeded and produce numerous symptoms, depending on
the age of the patient and the underlying pathology.
Computed tomography scan or magnetic resonance imaging of the head is the first priority
in evaluating suspected increased intracranial pressure. Meningitis is unlikely in this patient due
to the chronicity of symptoms and absence of fever. Accordingly, lumbar puncture is not
indicated at this time. Electroencephalography might be indicated if atypical migraines or
seizures were suspected, but the initial priority is to evaluate the patient for potential lifethreatening disease processes. The child has no evidence of viral infection, and reassurance or
administration of antiemetics is not appropriate.
References:
Frankel LR. Neurological emergencies and stabilization. In: Kliegman RM, Behrman RE, Jenson
HB, Stanton BF, eds. Nelson Textbook of Pediatrics. 18th ed. Philadelphia, Pa: Saunders
Elsevier; 2007:405-412
Larsen GY, Goldstein B. Consultation with the specialist: increased intracranial pressure. Pediatr
Rev. 1999;20:234-239. Available at:
http://pedsinreview.aappublications.org/cgi/content/full/20/7/234
page 190
A 16-year-old girl comes to your office complaining that her menstrual periods have been
irregular and scanty. Her last period was 3 months ago and lasted for only 2 days. Among the
findings on physical examination are fine, moist skin; firm, palpable thyroid gland (Item Q90); and
finger tremor. Results of laboratory studies include a thyroid-stimulating hormone value of less
than 0.05 mIU/L (normal, 0.5 to 5.0 mIU/L) and free thyroxine value of 1.9 ng/dL (24.5 pmol/L)
(normal, 0.6 to 1.3 ng/dL [7.7 to 16.8 pmol/L]).
Of the following, the additional physical examination finding that BEST supports the diagnosis of
hyperthyroidism is
A. abdominal obesity
B. atrophy of lingual papillae
C. hepatomegaly
D. hirsutism
E. muscle weakness
page 191
Preferred Response: E
The clinical findings of hyperthyroidism in children usually are obvious, but the diagnosis can be
subtle in mild disease. Classic clinical findings include weight loss, increased appetite,
decreased strength and sports performance, hyperactivity, tremors, sweating, nocturnal
sleeplessness (sometimes with daytime somnolence), irritability, decreased school
performance, pruritus, and nocturia. Menarcheal girls may have scant, infrequent menses, as
described for the girl in the vignette. On physical examination, most children have a palpable,
firm thyroid gland (Item C90A) that has an audible bruit. Some children may have fine, moist
skin; a visible tremor; slight skin darkening; fine scalp hair, with some hair loss at the temples;
muscle weakness; and some loss of muscle mass that can be identified by examining the thenar
and hypothenar eminences. A "thyrotoxic stare" accompanies hyperthyroidism. Exophthalmos
(Item C90B) may be found in thyrotoxicosis due to Graves disease. Hirsutism, hepatomegaly,
abdominal obesity, and changes in the lingual papillae are not findings of hyperthyroidism.
Other laboratory studies that are of use in caring for the girl in the vignette include a
measurement of triiodothyronine, which often is substantially elevated in hyperthyroidism, and
measures of thyroid-stimulating immunoglobulins, which are elevated in Graves disease.
Radioactive iodine or technetium uptake imaging can distinguish between subacute thyroiditis
(low uptake) and Graves disease (high uptake). This distinction is important because treatments
for Graves disease, including use of antithyroid drugs, radioactive iodine, and surgery, are not
appropriate for transient subacute thyroiditis.
References:
Fenton CL, Gold JG. Hyperthyroidism. eMedicine Specialties, Pediatrics, Endocrinology. 2006.
Available at: http://www.emedicine.com/ped/topic1099.htm
Ferry RJ Jr, Levitsky LL. Graves disease. eMedicine Specialties, Pediatrics, Endocrinology.
2006. Available at: http://www.emedicine.com/ped/topic899.htm
LaFranchi S. Clinical manifestations and diagnosis of hyperthyroidism in children and
adolescents. UpToDate Online 15.3. 2008. Available for subscription at:
http://www.uptodateonline.com/utd/content/topic.do?topicKey=pediendo/5570
page 192
An infant in the newborn nursery does not appear to respond to visual or auditory input. On
physical examination, he shows evidence of intrauterine growth restriction (IUGR), absent red
reflexes, and numerous bluish papules (Item Q91). The mother, who immigrated to the United
States during her third trimester, did not receive prenatal care. She denies use of alcohol, drugs,
or tobacco products during pregnancy. She reports that she had a low-grade fever and rash
during the second month of the pregnancy.
Of the following, the MOST likely infectious cause of the findings in this infant is
A. cytomegalovirus
B. human immunodeficiency virus
C. rubella virus
D. Toxoplasma gondii
E. varicella-zoster virus
page 193
Preferred Response: C
Congenital infections may lead to developmental sequelae in infancy and childhood, including
visual impairment, hearing loss, and intellectual disabilities (Item C91A). The infant described in
the vignette has clinical findings most consistent with congenital rubella syndrome (CRS). These
findings include intrauterine growth restriction (IUGR), absent red reflexes (due to cataracts),
and bluish papules (Item C91B). Other clinical findings associated with CRS include nerve
deafness, microphthalmia, cardiac defects, meningoencephalitis, hepatomegaly, and
microcephaly.
Maternal cytomegalovirus (CMV) infection is common, but 90% of infants who have CMV
infection are asymptomatic at birth. Maternal symptoms include a flulike illness that may involve
fever, but a rash is not seen, as reported by the mother in the vignette. Among the clinical
findings of congenital CMV infection are hepatomegaly, splenomegaly, jaundice, petechiae,
chorioretinitis, IUGR, purpura, and microcephaly. CMV infection is the leading nongenetic reason
for sensorineural hearing loss and the most common congenital infection to cause intellectual
disability.
Congenital varicella infection may result in zigzag scarring of the skin, limb deformities, or
cataracts. Occasionally, severely affected infants may have central nervous system
involvement with necrotizing cerebral lesions or microcephaly.
Congenital toxoplasmosis presents with IUGR, anemia, jaundice, hepatosplenomegaly,
intracranial calcifications, hydrocephalus, microcephaly, and chorioretinitis, but not skin lesions.
A newborn infected perinatally with human immunodeficiency virus exhibits no symptoms or
signs. Later, the infant may develop subtle clinical findings, such as lymphadenopathy and
hepatosplenomegaly, or nonspecific symptoms, such as failure to thrive, chronic or recurrent
diarrhea, interstitial pneumonia, or oral thrush. Central nervous system involvement is variable,
ranging from mild learning disabilities to severe mental retardation.
References:
Adler SP, Marshall B. Cytomegalovirus infections. Pediatr Rev. 2007;28:92-100. Available at:
http://pedsinreview.aappublications.org/cgi/content/full/28/3/92
Mason W. Rubella. In: Kliegman RM, Behrman RE, Jenson HB, Stanton, BF, eds. Nelson
Textbook of Pediatrics. 18th ed. Philadelphia, Pa: Saunders Elsevier; 2007:1337-1341
McLeod R, Remington JS. Toxoplasmosis (Toxoplasma gondii). In: Kliegman RM, Behrman RE,
Jenson HB, Stanton BF, eds. Nelson Textbook of Pediatrics. 18th ed. Philadelphia, Pa:
Saunders Elsevier; 2007:1486-1495
Myers MG, Seward J, La Russa P. Varicella-zoster virus. In: Kliegman RM, Behrman RE,
Jenson HB, Stanton BF, eds. Nelson Textbook of Pediatrics. 18th ed. Philadelphia, Pa:
Saunders Elsevier; 2007:1366-1372
Stagno S. Cytomegalovirus. In: Kliegman RM, Behrman RE, Jenson HB, Stanton BF, eds.
Nelson Textbook of Pediatrics. 18th ed. Philadelphia, Pa: Saunders Elsevier; 2007:1377-1379
Yogev R, Gould Chadwick E. Acquired immunodeficiency syndrome (human immunodeficiency
virus). In: Kliegman RM, Behrman RE, Jenson HB, Stanton BF, eds. Nelson Textbook of
Pediatrics. 18th ed. Philadelphia, Pa: Saunders Elsevier; 2007:1427-1442
page 194
A 6-month-old boy presents to the emergency department with a 2-day history of fever and a 1day history of left cheek swelling. You discover that his parents do not believe in providing their
children with immunizations. Despite this, the boy has never been ill. He has two older siblings,
and nobody is sick at home. The mother denies any recent bug bites or trauma to the area on
his cheek. Physical examination reveals a mildly toxic-appearing child who has a temperature of
103.0F (39.4C), heart rate of 145 beats/min, respiratory rate of 26 breaths/min, and blood
pressure of 80/45 mm Hg. His anterior fontanelle is slightly bulging, his tympanic membranes are
erythematous, his left cheek is indurated and appears erythematous to slightly violaceous (Item
Q92), and he is irritable.
Of the following, the MOST likely organism to cause this childs illness is
page 195
Preferred Response: A
Children who have not received the Haemophilus influenzae type b (Hib) vaccine are at risk for
illnesses commonly caused by this organism, including buccal and periorbital cellulitis (as
described for the boy in the vignette) (Item C92A), pyogenic arthritis, epiglottitis (Item C92B),
and bacterial meningitis. Neisseria meningitidis usually does not cause a facial cellulitis, and
Staphylococcus aureus and Streptococcus pyogenes are less likely pathogens in the absence
of a history of a break in the skin. S pneumoniae can be the cause of a nontraumatic facial
cellulitis in Hib-vaccinated children, but in an unvaccinated child, Hib would be the most likely
pathogen.
Hib disease can be verified by recovery of the organism from a sterile site (eg, blood,
cerebrospinal fluid, joint fluid) or by urine antigen testing. Once the organism is isolated,
antimicrobial susceptibility testing is important because approximately 30% to 40% of Hib
isolates produce beta-lactamase, making these organisms resistant to ampicillin.
References:
American Academy of Pediatrics. Haemophilus influenzae infections. In: Pickering LK, Baker CJ,
Long SS, McMillan JA, eds. Red Book: 2006 Report of the Committee on Infectious Diseases.
27th ed. Elk Grove Village, Ill: American Academy of Pediatrics; 2006:310-318
Tzanakaki G, Mastrantonio P. Aetiology of bacterial meningitis and resistance to antibiotics of
causative pathogens in Europe and in the Mediterranean region. Int J Antimicrob Agents.
2007;29:621-629. Abstract available at: http://www.ncbi.nlm.nih.gov/pubmed/17368858
page 196
You are seeing a 5-year-old boy who has developed diplopia, dysphagia, dry mouth, diarrhea,
weakness in his arms, and shortness of breath over the past 18 hours. According to his
records, he received his diphtheria, tetanus, acellular pertussis (DTaP), poliovirus inactivated
(IPV), measle-mumps-rubella (MMR), and varicella booster immunizations about 1 month ago.
He attended a class picnic 3 weeks ago that was held in a state park. He has no history of
unusual exposures or ill contacts, and except for falling off his bike 5 days ago and scraping his
arm, he has had no other trauma. Physical examination reveals an awake and alert boy who
complains of "seeing double" and of pain with swallowing. His pupils are 3 mm bilaterally and
sluggish, and his mucous membranes are dry. He takes shallow breaths, but his lungs are
clear, and his abdomen is mildly distended. His left arm has a 4x4-cm abrasion that is mildly
swollen, erythematous, and tender, with some serosanguineous drainage. His left arm has 2/5
strength and decreased tone. He has 1+ reflexes in the upper and lower extremities.
Of the following, the MOST likely cause of this patients condition is
A. botulism
B. cerebral vascular accident
C. Guillain-Barr syndrome
D. tetanus
E. tick paralysis
page 197
Preferred Response: A
Clostridium botulinum spores are found worldwide in soil and marine sediments. Botulism results
from the absorption of botulinum toxins, most commonly produced by C botulinum (a large, grampositive, anaerobic bacillus that has a subterminal spore), into the circulation from a wound or
mucosal surface. Of the seven antigenic toxin types that the organism produces, human
botulism is caused by neurotoxins A, B, E, and occasionally F. The clinical forms of botulism
include: foodborne, infant, wound, and that of undetermined cause, which is rare. Almost all
cases of infant botulism and wound botulism are caused by types A and B toxin. The onset of
symptoms may be abrupt (within hours) or evolve gradually over several days. Foodborne
botulism occurs with the ingestion of food that is contaminated with spores of C botulinum and
has been stored improperly under anaerobic conditions, allowing for toxin production. In wound
botulism, the spores are introduced into the wound at the time of trauma, where they germinate
and produce toxin. Infant botulism results from ingestion of C botulinum spores that germinate,
multiply, and produce toxin in the intestine. Most cases of infant disease occur in breastfed
infants at the time of introduction of nonhuman milk substances, and the source of the spores
usually is not identified.
The classic presentation of botulism is acute, bilateral cranial nerve palsies associated with
symmetric descending weakness. Fever is absent, sensory deficits do not occur, and the
patient remains responsive, with a normal level of consciousness. Foodborne disease usually
develops between 12 and 36 hours after toxin ingestion. The initial symptoms include nausea,
dry mouth, and diarrhea. Disease progresses to cranial nerve dysfunction, most commonly
starting with the eyes (diplopia, blurry vision), and descending to include dysphagia, upper
extremity weakness, respiratory dysfunction, and lower extremity weakness. The symptoms of
wound botulism are similar to that of food-borne disease, although the incubation period is 4 to
14 days from the time of injury until the onset of symptoms. Also, in many cases, the wound may
not appear to be healing, as described for the boy in the vignette. Infant botulism develops 3 to
30 days from the time of exposure to the spore-containing material. Infants present with
constipation, which is followed by feeding difficulties, hypotonia, increased drooling, a weak cry,
diminished gag reflex, truncal weakness, cranial nerve palsies, and generalized weakness with
ventilatory failure. Progression occurs for 1 to 2 weeks, followed by stabilization for another 2 to
3 weeks before recovery starts.
An important element in the treatment of all forms of botulism is meticulous supportive care.
In addition, patients who have suspected foodborne and wound botulism should be treated with
equine trivalent antitoxin (types A, B, and E) available from the Centers for Disease Control and
Prevention through state health departments. Immediate administration of antitoxin is critical for
successful therapy because it arrests the progression of paralysis but does not reverse it.
Patients who have wound botulism also should undergo wound debridement, even if the wound
appears to be healing. The role of antibiotic therapy is unknown, but penicillin G or metronidazole
frequently is recommended. Human-derived botulinum antitoxin (botulism immune globulin
intravenous) (BIGIV) is used in the treatment of infant botulism caused by type A or type B C
botulinum toxin. BIGIV therapy should be initiated as early in the illness as possible. Antibiotic
therapy is not recommended for infant botulism.
Patients who have cerebral vascular accidents usually do not present with bilateral
symmetric weakness or the other symptoms demonstrated by the patient in the vignette.
Patients who have tetanus have persistent, painful tonic spasms of the muscles of the neck,
jaw, and trunk and are very rigid. The paralysis of tick paralysis is ascending, beginning in the
lower extremities and ascending symmetrically to involve the trunk, upper extremities, and head
within a few hours. Patients who have Guillain-Barr syndrome typically present with numbness
and paresthesias of the hands and feet, followed by progressive weakness involving all four
extremities. Motor impairment begins in the lower extremities and progresses in an ascending
pattern to involve the upper extremities, trunk, and cranial nerves.
References:
American Academy of Pediatrics. Botulism and infant botulism (Clostridium botulinum). In:
page 198
Pickering LK, Baker CJ, Long SS, McMillan JA, eds. Red Book: 2006 Report of the Committee
on Infectious Diseases. 27th ed. Elk Grove Village, Ill: American Academy of Pediatrics;
2006:257-260
American Academy of Pediatrics. Tetanus (lockjaw). In: Pickering LK, Baker CJ, Long SS,
McMillan JA, eds. Red Book: 2006 Report of the Committee on Infectious Diseases. 27th ed. Elk
Grove Village, Ill: American Academy of Pediatrics; 2006:648-653
Bleck TP. Clostridium botulinum (botulism). In: Mandell GL, Bennett JE, Dolin R, eds. Mandell,
Douglas, and Bennett's Principles and Practice of Infectious Diseases. 6th ed. Philadelphia, Pa:
Elsevier Churchill Livingstone; 2005:2822-2828
Mathieu ME, Wilson BB. Ticks (including tick paralysis). In: Mandell GL, Bennett JE, Dolin R,
eds. Mandell, Douglas, and Bennett's Principles and Practice of Infectious Diseases. 6th ed.
Philadelphia, Pa: Churchill Livingstone; 2005:3312-3315
Parke JT. Peripheral neuropathies. In: McMillan JA, Feigin RD, DeAngelis CD, Jones MD Jr,
eds. Oski's Pediatrics Principles and Practice. 4th ed. Philadelphia, Pa: Lippincott Williams &
Wilkins; 2006:2310-2316
Schlagger B, Kornberg AJ, Prensky AL. Cerebrovascular disease in childhood. In: McMillan JA,
Feigin RD, DeAngelis CD, Jones MD Jr, eds. Oski's Pediatrics Principles and Practice. 4th ed.
Philadelphia, Pa: Lippincott Williams & Wilkins; 2006:2270-2279
page 199
An 8-month-old girl who has a history of cardiomyopathy following viral myocarditis presents
with poor weight gain. She is receiving a 20-kcal/oz milk-based formula and has no history of
vomiting or diarrhea. Her only medication is furosemide. Physical examination findings include a
heart rate of 130 beats/min, respiratory rate of 60 breaths/min, and blood pressure of 88/44 mm
Hg.
Of the following, the MOST appropriate initial strategy to increase weight gain for this girl is to
page 200
Preferred Response: A
Young infants who have cardiac dysfunction, such as the girl described in the vignette, often
have difficulty ingesting sufficient calories for growth because they frequently have increased
caloric expenditure with feeding. Increased caloric intake may place the infants at risk of fluid
overload, necessitating the use of diuretics. Because an increased volume of feedings with a 20kcal/oz formula may place the infant at risk of fluid overload, use of a more concentrated formula
is required. Accordingly, the infant described in the vignette should be changed to a 24-kcal/oz
formula. Increasing the caloric density of feedings can meet the goals of increased calories and
"relative" fluid restriction.
Discontinuation of furosemide likely would lead to weight gain from fluid retention, not a true
weight gain. A gastrostomy tube provides a conduit for feeding and probably plays a role in a
child incapable of taking in sufficient calories (eg, chronic renal failure or severe developmental
delay), but the child in the vignette deserves a trial of high-calorie feedings before subjecting her
to this surgical procedure. Finally, parenteral nutrition is not a suitable option because this child
has a functional gastrointestinal tract and can tolerate enteral nutrition.
References:
Kelleher DK, Laussen P, Teixeira-Pinto A, Duggan C. Growth and correlates of nutritional status
among infants with hypoplastic left heart syndrome (HLHS) after stage 1 Norwood procedure.
Nutrition. 2006; 22:237-244. Abstract available at: http://www.ncbi.nlm.nih.gov/pubmed/16500550
Pillo-Blocka F, Adatia I, Sharieff W, McCrindle BW, Zlotkin S. Rapid advancement to more
concentrated formula in infants after surgery for congenital heart disease reduces duration of
hospital stay: a randomized clinical trial. J Pediatr. 2004;145:761-766. Abstract available at:
http://www.ncbi.nlm.nih.gov/pubmed/15580197
Yahav J, Avigad S, Frand M, et al. Assessment of intestinal and cardiorespiratory function in
children with congenital heart disease on high-caloric formulas. J Pediatr Gastroenterol Nutr.
1985;4:778-785. Abstract available at: http://www.ncbi.nlm.nih.gov/pubmed/4045636
page 201
An 18-month-old girl has been having an intermittent nonproductive cough for the past 6 months.
Her parents state that the cough awakens the toddler at night a few times a month and occurs
when playing vigorously. During a recent upper respiratory tract illness, her cough worsened
and occurred daily for 3 weeks. On physical examination, there is no nasal discharge, and the
toddler appears healthy.
Of the following, the MOST likely diagnosis is
A. asthma
B. atypical pneumonia
C. gastroesophageal reflux
D. sinusitis
E. upper airway cough syndrome
page 202
Preferred Response: A
The chronic cough that is exacerbated during the night, with activity, and during an upper
respiratory tract infection described for the child in the vignette most likely represents asthma.
Chronic cough typically is defined as one that persists for more than 8 weeks. When the
patient's chest radiograph appears normal, three causes account for 95% of chronic coughs:
asthma, gastroesophageal reflux (GER), and upper airway cough syndrome (UACS)
(previously termed postnasal drip syndrome) (Item C95).
Asthma usually develops in early childhood, with 80% of patients reporting symptoms prior
to age 6 years. Symptoms may include cough, wheezing, shortness of breath, and chest
tightness. The most common trigger for infants and toddlers is a viral upper respiratory tract
infection (URI). Fortunately, URI-induced wheezing resolves in most infants by age 6 years (socalled "transient wheezers"). Those who continue to have asthma symptoms after age 6 are at
greater risk for persistent asthma.
UACS encompasses allergic rhinitis, nonallergic rhinitis, and sinusitis. Allergic rhinitis
typically occurs in children older than 3 years of age and is associated with other ocular and
nasal symptoms, such as pruritus, sneezing, and rhinorrhea. Sinusitis also is characterized by
rhinorrhea and postnasal symptoms.
Atypical pneumonia caused by Mycoplasma pneumoniae and Chlamydophila pneumoniae
(previously termed Chlamydia pneumoniae ) may present at any age, although it is unusual prior
to age 3 years. Characteristic constitutional symptoms include fever, malaise, and headache.
Cough can represent the sole manifestation of GER, but GER usually becomes
symptomatic during the first few postnatal months, improving by 12 months of age. GER may
worsen at night during supine positioning, but exercise and URIs are uncommon precipitating
factors for GER symptoms.
References:
Liu AH, Covar RA, Spahn JD, Leung DYM. Childhood asthma. In: Kleigman RM, Behrman RE,
Jenson HB, Stanton BF, eds. Nelson Textbook of Pediatrics. 18th ed. Philadelphia, Pa:
Saunders Elsevier; 2007:953-969
Weinberger M, Abu-Hasan M. Pseudo-asthma: when cough, wheezing, and dyspnea are not
asthma. Pediatrics. 2007;120:855-864. Available at:
http://pediatrics.aappublications.org/cgi/content/full/120/4/855
page 203
A 10-year-old boy comes to the office 2 days after falling off of his bicycle and injuring his
forehead. He denies vomiting or headache but complains of a runny nose. Physical examination
reveals a well-appearing boy who has a large ecchymotic swelling over the central portion of his
forehead with an overlying abrasion. The area is diffusely tender to palpation, and there is a
depression over the right lateral aspect of the swelling. Erythema around the abrasion is minimal,
and no purulent drainage is present. Clear fluid is draining from his right naris. The remainder of
his physical examination findings are normal. You order a computed tomography scan (Item
Q96).
Of the following, the MOST appropriate treatment of this boys injury should include
A. decongestants
B. nasal packing
C. no specific treatment
D. prophylactic antibiotics
E. surgical repair
page 204
Preferred Response: E
The history of trauma and the physical findings of forehead ecchymosis with clear, persistent
rhinorrhea described for the boy in the vignette should raise concern for a cerebrospinal fluid
(CSF) leak. Prompt radiologic imaging can aid in determining the cause. In most cases, a frontal
sinus fracture is the cause, as exhibited by the boy in the vignette, and head computed
tomography (CT) scan usually is adequate to identify the injury (Item C96). If an obvious
fracture is not seen on CT scan, further imaging with magnetic resonance imaging or
radionucleotide cisternography may be necessary to localize the source. Testing of the fluid for
glucose or protein is not diagnostically sensitive and should not be used to determine further
evaluation and treatment. The incidence of CSF leak following frontal sinus fracture is nearly
20% in children, twice the rate seen in adults. The primary concern in patients who have CSF
rhinorrhea is the development of central nervous system (CNS) infections such as meningitis or
brain abscess. Such complications are seen most commonly after trauma and in patients whose
leaks do not resolve spontaneously within 7 days.
Anterior wall frontal sinus fractures are repaired primarily for cosmetic reasons, but those
involving the posterior wall (which occurs in 70% of these injuries in children) require open
reconstruction to ensure that the communication between the sinus and brain is eliminated.
Decongestants and nasal packing are not indicated in the treatment of CSF rhinorrhea. The
use of prophylactic antibiotics for the prevention of CNS infection is controversial, although most
studies suggest that the risk of developing antibiotic resistance is greater than the potential
prevention of meningitis.
References:
Kellman RM. Maxillofacial trauma. In: Cummings CW, Flint PW, Haughey BH, Robbins KT,
Thomas JR eds. Cummings Otolaryngology: Head & Neck Surgery. 4th ed. Philadelphia, Pa:
Mosby Elsevier; 2005:chap 26
Kerr JT, Chu FW, Bayles SW. Cerebrospinal fluid rhinorrhea: diagnosis and management.
Otolaryngol Clin North Am. 2005;38:597-611. Abstract available at:
http://www.ncbi.nlm.nih.gov/pubmed/16005720
Kravitz PR, Koltai PJ. Pediatric facial fractures. In: Cummings CW, Flint PW, Haughey BH,
Robbins KT, Thomas JR eds. Cummings Otolaryngology: Head & Neck Surgery. 4th ed.
Philadelphia, Pa: Mosby Elsevier; 2005:chap 202
page 205
A 16-year-old boy in your practice has cystic fibrosis. As a complication of his illness, he has
developed cirrhosis and cholestasis. He now complains of shaky hands. Neurologic examination
demonstrates hyporeflexia and tremor with hands outstretched.
Of the following, the patients symptoms are MOST consistent with deficiency of
A. vitamin A
B. vitamin B1 (thiamine)
C. vitamin C
D. vitamin D
E. vitamin E
page 206
Preferred Response: E
Because the young man described in the vignette has chronic cholestasis, he is at risk for
developing deficiency of any of the fat-soluble vitamins, including vitamins A, D, E, and K. His
neurologic symptoms of tremor and hyporeflexia most strongly suggest vitamin E deficiency.
Vitamin E (tocopherol) is an important factor in stabilizing the lipid membrane of the red blood
cell and the lipids in the myelin sheath of neurons. Therefore, the most common presenting
features of hypovitaminosis E are hemolysis (primarily reported in preterm infants) and
peripheral neuropathy (identified in infants and children who have chronic cholestasis,
pancreatic insufficiency, or malabsorption).
Supplementation of formulas and parenteral nutrition with vitamin E has reduced
substantially the incidence of hemolysis in the vitamin E-deficient preterm infant. However,
patients who have cystic fibrosis or cholestatic liver disease require both monitoring of vitamin E
concentrations and supplementation with vitamin E. Because vitamin E is a fat-soluble vitamin,
those who have cholestasis may have difficulty absorbing alpha-tocopherol, the form of vitamin
E available in most dietary supplements. For this reason, d-alpha-tocopheryl polyethylene glycol
1,000 succinate, a water-soluble form of vitamin E, should be given to patients who have
significant cholestatic liver disease. The recommended dose for a patient who has cholestatic
liver disease is 15 to 25 IU/kg per day.
Deficiency of vitamin A, B1, C, or D would not be expected to cause such a clinical
presentation. Vitamin A deficiency causes impaired vision ("night blindness") and corneal ulcers;
vitamin B1 deficiency can cause myopathy and heart failure ("beriberi"); vitamin C deficiency
causes irritability, bone lesions, and bruising (scurvy); and vitamin D deficiency causes
osteopenia or rickets.
References:
Harmatz P, Burensky E, Lubin B. Nutritional anemias. In: Walker WA, Watkins JB, Duggan C,
eds. Nutrition in Pediatrics. 3rd ed. Hamilton, Ontario, Canada: BC Decker; 2003:830-847
Spinozzi NS. Hepatobiliary diseases. In: Hendricks KM, Duggan C. Manual of Pediatric Nutrition.
4th ed. Hamilton, Ontario, Canada: BC Decker; 2005:586-592
page 207
You are making rounds with medical students in the neonatal intensive care unit and examining a
2-kg, 34 weeks gestation newborn whose mother had gestational diabetes mellitus. The infant
has no respiratory distress. A medical student asks how to test for fetal lung maturity to predict
the risk of neonatal respiratory distress syndrome in the offspring of a pregnant woman who has
diabetes mellitus.
Of the following, the MOST appropriate test is
page 208
Preferred Response: E
The human fetus continues to develop lung maturity up until a term gestation, but the lungs
generally are sufficiently mature to maintain extrauterine respiration by 36 weeks' gestation.
Infants delivered prior to this time have variable degrees of pulmonary maturity as surfactant
composition, synthesis, and storage change with advancing fetal development. Tests for
determining fetal lung maturation depend on amniotic fluid analysis for surfactant presence and
composition because fetal surfactant leaves the lung as an effluent that ebbs and flows with fetal
breathing movement and is balanced by fetal swallowing.
In general, in the healthy fetus, growth, size, and maturation typically are linked with
gestational age. Fetal lung maturity has been assessed using the lecithin-to-sphingomyelin ratio
(L:S) for more than 30 years. This test is time-consuming and requires thin-layer
chromatography. It depends on fetal lung fluid flowing into the amniotic fluid and altering the
amniotic fluid phospholipid composition, with the results expressed as the ratio of a lecithin
(phosphatidylcholine) to sphingomyelin per milliliter of amniotic fluid. The sphingomyelin content
of amniotic fluid decreases after 32 weeks' gestational age, while the lecithin content, including
that portion that is disaturated, increases. The L:S ratio is designed to account for varying
amniotic fluid volumes that generally cannot be measured precisely. An L:S ratio of 2.0 typically
is achieved by 35 weeks' gestation. Clinically, respiratory distress syndrome (RDS) due to
surfactant deficiency is very unlikely if the L:S ratio is 2.0 or greater, it is indeterminate if the L:S
ratio is 1.5 to 2.0, and the incidence of RDS is high if the L:S ratio is less than 1.0. Of note, the
L:S ratio can increase over a period of several days or can be induced by the administration of
glucocorticoids to the mother.
Phosphatidylinositol (PI) is a pulmonary phospholipid that increases throughout gestation, as
does disaturated lecithin. PI concentrations decrease after 35 weeks' gestation. The mere
presence of PI or disaturated lecithin, therefore, does not indicate lung maturity.
Phosphatidylglycerol (PG) generally can be detected in the amniotic fluid at 36 weeks' gestation.
Due to its appearance relatively late in gestation, it has been used as an indicator of pulmonary
surfactant maturity.
In pregnancies complicated by maternal diabetes, a mature L:S ratio (>2.0) or the presence
of PG may not indicate pulmonary surfactant maturity. In this setting, fetal lung maturity is
determined best using a quantitative analysis of total surfactant activity. This rapidly determined,
automated fluorescence polarization assay measures all surfactant phospholipids in amniotic
fluid referenced as milligrams of surfactant per gram of albumin present. Recent data suggest
that a total surfactant activity of greater than 45 mg surfactant phospholipids per gram of albumin
is as good as, or better than, an L:S ratio of 2.0 in predicting fetal lung maturity, especially in the
presence of maternal diabetes.
References:
Jobe AH. Lung development and maturation. In: Martin RJ, Fanaroff AA, Walsh MC, eds.
Fanaroff and Martin's Neonatal-Perinatal Medicine. 8th ed. Philadelphia, Pa: Mosby Elsevier;
2006:1069-1086
Grenache DG, Gronowski AM. Fetal lung maturity. Clin Biochem. 2006;39:1-10. Abstract
available at: http://www.ncbi.nlm.nih.gov/pubmed/16303123
Winn-McMillan T, Karon BS. Comparison of the TDx-FLM II and lecithin to sphingomyelin ratio
assays in predicting fetal lung maturity. Am J Obstet Gynecol. 2005;193:778-782. Abstract
available at: http://www.ncbi.nlm.nih.gov/pubmed/16150274
page 209
At the end of the summer, you notice an increase in the number of preparticipation sports
examinations you are performing. You are pleased at the number of your patients who are
involved in sports activities but are reminded that many medical conditions preclude sports
participation and must be screened for during the preparticipation visit.
Of the following, the medical condition that is considered a CONTRAINDICATION for sports
participation is
A. a boy who has chronic leukemia and splenomegaly wishing to play golf
B. a boy who has insulin-dependent diabetes wishing to play tennis
C. a boy who has well-controlled seizures wishing to participate in basketball
D. a girl who has Marfan syndrome wishing to participate in gymnastics
E. a girl who has one ovary wishing to participate in softball
page 210
Preferred Response: D
Sports participation can play an important role in the social development of children and
adolescents by teaching leadership and team-building skills and encouraging physical fitness.
Most children can be cleared easily for such participation, but not all sports are safe for children
and adolescents, and some medical conditions warrant special consideration with regard to
participation in some sports. In all cases, the clinician should perform a complete preparticipation
sports physical examination and discuss the implications of the sports the patient plans to
pursue.
Sports can be classified by contact level (Item C99A). Children who have splenomegaly,
acute hepatomegaly, and contagious skin lesions should avoid contact sports but can
participate in noncontact sports. Children who have a single kidney should be advised to avoid
high-contact sports, but those who have other single organs, such as ovaries and testes, can
be cleared because the risk of injury is low or protective gear can be worn. Protective eyewear
can reduce the risk of injury in the case of a single functional eye. It is prudent for the clinician to
advise such patients of the risk to the remaining organ during sports participation, and some
advocate having the parents and the athlete sign a document that indicates their understanding
and acceptance of such risk.
Sports also may be classified by intensity (dynamic and static demand) (Item C99B). This
classification is especially important for children and adolescents who have cardiovascular
diseases. For example, children who have ventricular dysfunction should avoid high dynamic
sports; those who have significant essential hypertension, left heart obstructive disease, or an
increased risk of aortic dissection should avoid high static sports. Accordingly, a girl who has
Marfan syndrome should avoid gymnastics, a sport that has potentially high static intensity.
Several cardiac diseases, such as acute pericarditis or myocarditis, cardiomyopathy, severe
pulmonary hypertension, and right-to-left shunting, are absolute contraindications to participation
in sports. A full listing of these conditions is outlined in the 36th Bethesda Conference guidelines.
Patients who have well-controlled epilepsy can be cleared to participate in most sports, but
those whose seizures are poorly controlled should be advised to avoid sports in which they
could sustain significant injury should a seizure occur during participation, such as swimming,
diving, and riflery. Patients who have diabetes should be encouraged to participate in sports, but
they should pay attention to hydration and insulin therapy, especially with sports that last longer
than 30 minutes.
References:
36th Bethesda conference: eligibility recommendations for competitive athletes with
cardiovascular abnormalities. J Am Coll Cardiol. 2005;45:1312-1375. Available at:
http://content.onlinejacc.org/cgi/reprint/45/8/1312
American Academy of Pediatrics Committee on Sports Medicine and Fitness. Medical conditions
affecting sports participation. Pediatrics. 2001;107:1205-1209. Available at:
http://pediatrics.aappublications.org/cgi/content/full/107/5/1205
Metzl JD. Preparticipation examination of the adolescent athlete: part 1. Pediatr Rev.
2001;22:199-204. Available at: http://pedsinreview.aappublications.org/cgi/content/full/22/6/199
Metzl JD. Preparticipation examination of the adolescent athlete: part 2. Pediatr Rev.
2001;22:227-239. Available at: http://pedsinreview.aappublications.org/cgi/content/full/22/7/227
Singh A, Silberbach M. Consultation with the specialist: cardiovascular preparticipation sports
screening. Pediatr Rev. 2006;27:418-424. Available at:
http://pedsinreview.aappublications.org/cgi/content/full/27/11/418
page 211
A child presents to a clinic associated with a disaster relief shelter after a hurricane destroys the
community. The mother states that they have run out of the creams prescribed for her
daughters eczema before the storm, and the child is itchy. She is concerned because there are
some blisters and crusting in the antecubital fossae and popliteal fossa where the itching is
worst. Physical examination reveals erosions (Item Q100) and erythema surrounding areas of
lichenification, with a few vesicles both in clusters and scattered.
Of the following, the MOST likely pathogen involved in this pattern of infection is
page 212
Preferred Response: A
Children who have atopic dermatitis are prone to recurrent skin infections, particularly with
Staphylococcus aureus and herpes simplex virus (HSV), for several reasons. Exacerbations of
eczema disrupt the skin's protective barrier. The failure to produce endogenous antimicrobial
peptides has been offered as a reason for an increase risk for infection with S aureus.
Dissemination of cutaneous viral infections (eg, HSV and molluscum contagiosum) is believed to
be due to defects in cytokine production and T-cell function.
Secondary infection with S aureus may result in "weepy" or crusted erosions (Item C100A).
Use of topical and systemic antistaphylococcal antibiotics, with attention to the emergence of
community-acquired antibiotic-resistant strains, is indicated. Use of topical immune modulators
(eg, tacrolimus) should be interrupted during the infection because of the potential for inhibiting
local immune response and the development of irritation, including burning and stinging.
Eczema herpeticum results when areas of active dermatitis are infected by HSV. Initially,
there may be an increase in complaints of itching and scratching, with a noticeable increase in
erythema; fever may be present. As described for the child in the vignette, clustered, often
umbilicated vesicles appear; these rupture, forming ulcers that have a "punched-out"
appearance (Item C100B). Infection may become disseminated and sometimes is fatal. For
those who have extensive involvement, recommended treatment is parenteral acyclovir.
Sporotrichosis is a dimorphic fungus that can be isolated from soil and plants such as roses;
it also may be found in hay, straw, and decaying vegetation. It may cause infection at a site of
minor trauma that appears as a solitary violaceous nodule (Item C100C). Although infections
caused by streptococci (viridans and enterococci) may be observed in immunocompromised
hosts and in neonates, they do not cause the type of disease described for the child in the
vignette. Papules, not vesicles, are characteristic of human papillomavirus infection of the skin.
Primary infection with varicella-zoster virus results in varicella, with individual, not grouped,
vesicles that rupture, leaving shallow erosions, not "punched-out" ulcers.
References:
American Academy of Pediatrics. Non-group A or B streptococcal and enterococcal infections.
In: Pickering LK, Baker CJ, Long SS, McMillan JA, eds. Red Book: 2006 Report of the
Committee on Infectious Diseases. 27th ed. Elk Grove Village, Ill: American Academy of
Pediatrics; 2006:627-629
American Academy of Pediatrics. Sporotrichosis. In: Pickering LK, Baker CJ, Long SS, McMillan
JA, eds. Red Book: 2006 Report of the Committee on Infectious Diseases. 27th ed. Elk Grove
Village, Ill: American Academy of Pediatrics; 2006:595-597
Bunikowski R, Mielke M, Skarabis H, et al. Prevalence and role of serum IgE antibodies to the
Staphylococcus aureus-derived superantigens SEA and SEB in children with atopic dermatitis. J
Allergy Clin Immunol. 1999;103:119-124. Abstract available at:
http://www.ncbi.nlm.nih.gov/pubmed/9893195
Horii KA, Simon SD, Liu DY, Sharma V. Atopic dermatitis in children in the United States, 19972004: visit trends, patient and provider characteristics, and prescribing patterns. Pediatrics.
2007;120:e527-e534. Available at:
http://pediatrics.aappublications.org/cgi/content/full/120/3/e527
Knoell KA, Greer KE. Atopic dermatitis. Pediatr Rev. 1999;20:46-52. Available at:
http://pedsinreview.aappublications.org/cgi/content/full/20/2/46
Stanbury LR. Herpes simplex virus. In: Kliegman RM, Behrman RE, Jenson HB, Stanton BF,
eds. Nelson's Textbook of Pediatrics. Philadelphia, Pa: Saunders Elsevier; 2007:1360-1365
Waggoner-Fountain LA, Grossman LB. Herpes simplex virus. Pediatr Rev. 2004;25:86-93.
page 213
page 214
A 4-week-old infant who was born at term without any complications ate well and gained weight
for the first 3 weeks after birth. Over the last week, however, his mother reports that he appears
hungry but fatigues with feeding and now takes twice as long to complete his feeding as he did 1
week ago. He also breathes fast during his feedings and stops frequently to "catch his breath."
Of the following, the MOST likely explanation for the symptoms in this infant is
A. aspiration syndrome
B. congestive heart failure
C. gastroesophageal reflux disease
D. inborn error of metabolism
E. pneumonia
page 215
Preferred Response: B
The symptoms described for the child in the vignette are typical of progressive congestive heart
failure (CHF). The "hunger" described by the mother suggests that the problem results from the
infant's inability to take enough calories for satiation and growth. Such so-called poor feeding is
due to his inability to generate a prolonged suck while maintaining nasal breathing because of the
tachypnea that is caused by pulmonary congestion. Thus, the infant seems to stop sucking and
"catch his breath." Decreased caloric intake coupled with increased caloric expenditure caused
by tachypnea and tachycardia makes it difficult for the infant to gain weight. Indeed, weight loss
is common in infants who have CHF.
CHF is a clinical syndrome that reflects the inability of the myocardium to meet the metabolic
requirements of the body, including those for growth. Congenital heart defects are the most
common reason for pediatric heart failure, and the failure develops most frequently during early
infancy. CHF results from excessive workload imposed on the cardiac muscle that usually is
caused by the structural defects. The cardiac defects may impose an excessive volume load on
the left ventricle (eg, large ventricular septal defect, atrioventricular septal defect), an excessive
pressure load on the ventricle (eg, aortic stenosis), or a combination of volume and pressure
load (eg, ventricular septal defect with coarctation). Less commonly, CHF may be caused by an
intrinsic alteration in myocardial performance, which could result from an inflammatory or an
infectious process that directly affects the myocardium and depresses its contractile function.
Aspiration syndrome typically is an acute event resulting from the passage of
gastrointestinal contents (including food) into the lungs. The symptoms usually are acute and not
associated with poor feeding in spite of hunger. Similarly, gastrointestinal reflux presents more
acutely with signs of discomfort that might include arching during feedings. Pneumonia is
associated with other signs of infection, including tachypnea when at rest, fever, or change in
behavior. Inborn errors of metabolism, although less common, are important to consider in the
neonate and infant who is not feeding well. Most commonly, they are associated with symptoms
that are present both at rest and during feeding as well as other systemic signs.
References:
Balfour I. Management of chronic congestive heart failure in children. Curr Treat Options
Cardiovasc Med. 2004;6:407-416. Abstract available at:
http://www.ncbi.nlm.nih.gov/pubmed/15324616
Dreyer WJ, Fisher DJ. Clinical recognition and management of chronic congestive cardiac
failure. In: Garson A Jr, Bricker JT, Fisher DJ, Neish SR, eds. The Science and Practice of
Pediatric Cardiology. 2nd ed. Baltimore, Md: Williams & Wilkins, 1998:2309-2325
Silberbach M, Hannon D. Presentation of congenital heart disease in the neonate and young
infant. Pediatr Rev. 2007;28:123-131. Available at:
http://pedsinreview.aappublications.org/cgi/content/full/28/4/123
Talner NS, McGovern JJ, Carboni MP. Congestive heart failure. In: Moller JH, Hoffman JIE, eds.
Pediatric Cardiovascular Medicine. Philadelphia, Pa: Churchill Livingstone; 2000:817-829
page 216
A 10-year-old boy has double vision and drooping eyelids. On physical examination, he is
afebrile and has normal mentation. Pupillary responses are normal, but he has bilateral ptosis.
He cannot fully adduct his right eye. You note that his ptosis increases with sustained upward
gaze (Item Q102). Bedside forced vital capacity is normal.
Of the following, the test MOST likely to confirm the diagnosis is
page 217
Preferred Response: C
The child described in the vignette presents with progressive fatigable weakness affecting